You are on page 1of 293

‫بسمَّهللاَّالرمحنَّالرحمي‬

‫(فَأَمَّاَّالزبَدُ َّفَيَ ْذه َُبَّ ُجفَ ًاء ََّوأَماَّ َماَّي َ ْنفَ ُعَّالن َاسَّفَيَ ْم ُك ُث يَِّفَّ‬
‫َّاَّلل َّْاْلَ ْمث َا ََّل)‬
‫ْض ُب ُ‬ ‫ْاْلَ ْر يضَّكَ َذ ي َِلَّي َ ْ ي‬

‫قالَّرسولَّهللاَّصيلَّهللاَّعليهَّ"اذاَّماتَّابنَّأدمَّانقطعَّمعهلَّ‬
‫االَّمنَّثالث‪َّ:‬صدقةَّجارية‪َّ،‬أوَّعملَّينتفعَّبه‪َّ،‬أوَّودلَّصاحلَّ‬
‫يدعوَّهل"‬

‫اللهمَّاجعلَّهذاَّالعملَّخالصاَّلوهجكَّوانَّيرباانَّممنَّيس تغهلَّ‬
‫ملصاحلَّماديهَّاوَّمعنويهَّوانَّيرزقناَّحسنَّالعمل‪.‬‬
‫الَّنرجوَّمنمكَّسويَّدعوةَّصادقهَّمنَّالقلبَّعيسَّانَّتكونَّ‬
‫منَّنصيبناَّفاحبَّالاعاملَّايلَّهللاَّادلعاء‪.‬‬

‫نسألَّهللاَّالتوفيقَّوالنجاحَّوالسدادَّوانَّيرزقمكَّماَّتمتنونَّ‬
‫واس تغفرَّهللاَّيلَّولمك‬

‫(امللفَّالرئييس)‬

‫‪EMS‬‬
‫َّ‬
{1}

1( After subtotal thyroidectomy for patient. He complains restlessness the nurse is


assessing his vital signs.
BP is 150/100
Heart rate is 120.
Temperature 40
The nurse should expect that the cause for his condition is:
A. Hyper-metabolism due to increase in T3&T4
B. Hypo metabolism due to decrease T3&T4
C. Hypo metabolism due to increase in T3&T4
D. Hyper metabolism due to decrease in T3&T4.
Answer: A
2) Calculate Total intake and output Patient in OR
1500ml Ringer,50 ml antibiotic, Nasogastric tube feeding 50ml, Blood loss 500ml,
Urine out 120
Answer:
Total Intake = 1600 / Total Output = 620
3) Mother with hypothyroidism high risk for??
A. Preterm labor
B. Hemorrhage
C. Congenital anomalies
D. Eclampsia
Answer: A
4) Mother with hyperthyroidism high risk for??
A. Preterm labor
B. Pre-eclampsia
C. Hemorrhage
D. Congenital anomalies
Answer: B

ِ‫يِاْلَ أرض‬
‫أ‬ ُ ‫َم‬
ُ ‫ك‬
‫ثِِف‬ َ ‫اس‬
‫ِِفي أ‬َ َّ ‫ع‬
‫ِالن‬ ُ ‫ف‬
َ ‫ماِيَ أن‬ َّ َ‫وأ‬
َ ِ‫ما‬ َ
{2}

5) Post right-side mastectomy where to take blood pressure?


A. Right brachial
B. Left brachial.
C. Right radial
D. Left radial.
Answer: B
6) one-week short-term goal for a total hip replacement patient
A sits up by himself three times a day.
B. state pain control
Answer: B
7) Broken infusion machine - what to do
A. Put a sticker and report.
B. Put in the broken machines area.
Answer: A
8) ABG results - for an underweight girl 2 times?
A. one is ABG disorder - Metabolic Alkalosis.
B. uncompensated metabolic alkalosis
C. Compensated metabolic acidosis.
Answer: C
9) How to know it the left shoulder dislocated??
A. left arm longer than right arm
B. patient cannot move the left shoulder.
C. swelling in the left shoulder
Answer: A
10) A patient is admitted to the emergency department after sustaining abdominal
injuries and a broken femur from a motor vehicle accident. The patient is pale,
diaphoretic, and is not talking coherently. Vital signs upon admission are temperature
98.0 F (36.3 C), heart rate 130 beats/minute, respiratory rate 34 breaths/minute, blood

ِ‫يِاْلَ أرض‬
‫أ‬ ُ ‫َم‬
ُ ‫ك‬
‫ثِِف‬ َ ‫اس‬
‫ِِفي أ‬َ َّ ‫ع‬
‫ِالن‬ ُ ‫ف‬
َ ‫ماِيَ أن‬ َّ َ‫وأ‬
َ ِ‫ما‬ َ
{3}

pressure 50/40mmHg. The healthcare provider suspects which type of


shock?
A. Distributive
B. Neurogenic
C. Cardiogenic
D. Hypovolemic
Answer: D
11) Which of the following the arranged assessment for postoperative patient???
A. Cardiovascular, Respiration, Surgery site, Neurological signs.
B. Respiration, Cardiovascular, Neurological signs, Surgery site.
C. Neurological signs, Cardiovascular, Respiration, Surgery site.
D. Surgery site, Respiration, Cardiovascular, Neurological signs.
Answer: B
12) A woman at 24 weeks' gestational age has fever body ache and has coughing last
5 days she is sent to hospital with admission prescriptions for H1N1influenza. which
prescription has the highest priority?
A. Assign private room
B. Vital signs every 4 hours
C. Obtain specimens for culture
D. Ringer lactate IV
Answer: A
13) What is most appropriate blood test for patient who is going to surgery?
A. WBC
B. Albumin
C. Prothrombin time
D. Hemoglobin
Answer: C
14) Physician order three times the normal dose - when the nurse calls him, he insists on
giving the ordered medication - what does the nurse do?
A. Call pharmacy

ِ‫يِاْلَ أرض‬
‫أ‬ ُ ‫َم‬
ُ ‫ك‬
‫ثِِف‬ َ ‫اس‬
‫ِِفي أ‬َ َّ ‫ع‬
‫ِالن‬ ُ ‫ف‬
َ ‫ماِيَ أن‬ َّ َ‫وأ‬
َ ِ‫ما‬ َ
{4}

B. Call another doctor.


C. Call nurse supervisor
Answer: C
15) Doctor's order excessed the recommended dose what to do?
A. Call the doctor to clarify.
B. Administer the dose.
C. Hold the dose.
D. Talk with supervisor.
Answer: A
16) Major depression activities
A. Watch a movie with other patients.
B. Plan today's activities with the patient
Answer: B
17) Gum and tooth pain 一 what is the cranial nerve?

A. Third nerve
B. Fifth nerve
C. Seventh nerve
D. Ninth nerve
Answer: B
18) Patient is 6 cm dilated what stage of labor?
A. First stage
B. Second stage
C. Third stage
D. Fourth stage
Answer: A

19) The nurse was caring for patient wound staples after operation. While she was preparing the
patient sneezed and one staple fell down the floor. What should the nurse do?
A. Bring the staple from floor
B. Bring it and put back on wound
C. Apply dressing to the wound✅

Answer: C

ِ‫يِاْلَ أرض‬
‫أ‬ ُ ‫َم‬
ُ ‫ك‬
‫ثِِف‬ َ ‫اس‬
‫ِِفي أ‬َ َّ ‫ع‬
‫ِالن‬ ُ ‫ف‬
َ ‫ماِيَ أن‬ َّ َ‫وأ‬
َ ِ‫ما‬ َ
{5}

20) Recurrent urinary tract infection what advice to give?


A. Drink 2-3 liters of water
B. Wipe from back to front
C. Urinate only when you have the urge.
Answer: A
21) What is the complication that needs observation after the birth of the child for
diabetic mother?
A. Preterm labor
B. Term labor
C. Macrosomia
D. Hyperglycemia
Answer: C
22) After cardiac catheterization, the nurse should do first ?
A. Assess blood pressure

B. Assess apical pulse /distal site✅


C. Elevated the head of the bed

Answer: B
23) post- operative patient to be discharged and his temperature is 37.6 at 8:00 -
everything else is normal what to do?
A. informs the physician to delay discharge.
B. gives aspirin dose earlier than scheduled.
C. temperature is normal because of the time.
D. suspected infection
Answer: C
24) pregnant women Said to nurse I feel I depends on my husband and my family?
A-self concern
B-ambivalence
C-normal
D-esteem
Answer: A

ِ‫يِاْلَ أرض‬
‫أ‬ ُ ‫َم‬
ُ ‫ك‬
‫ثِِف‬ َ ‫اس‬
‫ِِفي أ‬َ َّ ‫ع‬
‫ِالن‬ ُ ‫ف‬
َ ‫ماِيَ أن‬ َّ َ‫وأ‬
َ ِ‫ما‬ َ
{6}

25) What is the incidence rate Definition?


A. All new cases of specific disease during giving time.
B. All old cases of specific disease during giving time.
C. All new and old cases of specific disease during giving time.
Answer: A
26) What is the definition of prevalence rate?
A. All new cases of specific disease during giving time.
B. All old cases of specific disease during giving time.
C. All new and old cases of specific disease during giving time.
Answer: C
27) Woman went to gynecological clinic for routine examination then she diagnosed
for positive Chlamydia. What is this considered?
A. Clinical
B. Sub clinical
C. Suspect
Answer: B
28) What is New-born examination after delivery?
A. Vital Signs. Inspection. Auscultation. Percussion.
Palpation.
B. Inspection. Auscultation. Percussion. Palpation. Vital
Signs
C. Palpation Inspection. Auscultation. Percussion. Vital
Signs
D. Auscultation. Inspection. Palpation. Percussion. Vital
Signs
Answer: A
29) The doctor order 0.2 g, the available 400 mg in quantity
10 ml, how many ml needed??
A-2.5ml
B-5ml

ِ‫يِاْلَ أرض‬
‫أ‬ ُ ‫َم‬
ُ ‫ك‬
‫ثِِف‬ َ ‫اس‬
‫ِِفي أ‬َ َّ ‫ع‬
‫ِالن‬ ُ ‫ف‬
َ ‫ماِيَ أن‬ َّ َ‫وأ‬
َ ِ‫ما‬ َ
{7}

C- 2ml
D- 4ml
Answer: B
30) A 28-year man admitted to orthopedic ward complaining of throbbing pain in
casted leg. Which of the following nursing intervention should be taken first?
A. Remove the cast
B. Notify a doctor
C. Assess pedal pulse.
D. Administer PRN medication
Answer: C
31) Doctor write new order restrain (PRN) for Psychiatric patient that he is overly
aggressive. Which of the following indicate that the nurse should do??
A. Call doctor while patient aggressive to do that by himself.
B. Apply the order one time only or if needed.
C. Close patient room and do not apply restrain.
D. Express that is not acceptable and complains about your supervisor.
Answer: B
32) The Oral contraceptives increase risk for any cancer for long-term using??
A. Cervical cancer
B. Ovarian cancer
C. Skin cancer
D. Breast cancer
Answer: D
33) Yemeni soldier caught in the war and he got many injuries. The patient anxious and
he is speaking about death for the nurse. What is the most appropriate action for the
nurse??
A -You say no or yes.
B. Why do you say you are dying?
C. Why you say you will die.

ِ‫يِاْلَ أرض‬
‫أ‬ ُ ‫َم‬
ُ ‫ك‬
‫ثِِف‬ َ ‫اس‬
‫ِِفي أ‬َ َّ ‫ع‬
‫ِالن‬ ُ ‫ف‬
َ ‫ماِيَ أن‬ َّ َ‫وأ‬
َ ِ‫ما‬ َ
{8}

D. Restating for patient wards


Answer: D
34) There is someone his father is dead. Then he turned his sadness and anger by
learning martial arts. What is the defense mechanism that he used?
A. Displacement
B. Sublimation
C. Projection
D. Denial
Answer: B
35) In teaching a female client who is HIV-positive about pregnancy, the nurse would
know more teaching is necessary when the client says:
A. The baby can get the virus from my placenta.”
B. “I’m planning on starting on birth control pills.”
C. “Not everyone who has the virus gives birth to a baby who has the virus.”
D. “I’ll need to have a C-section if I become pregnant and have a baby.”
Answer: D
36) The neonate is delivered by cesarean section the baby should be transported to
NICU. Which type of baby incubator should be used?
A - Closed box incubators
B - Portable incubators
C - Double - walled incubators
D- Servo - control incubators
Answer: B
37) Therapeutic communication is used by the nurse to create a beneficial outcome in
the delivery of quality nursing care.
Which of the following statement best describe the term ‘’therapeutic
communication’’?
A. interactive process of socialization.
B. Assessment component of nursing process

ِ‫يِاْلَ أرض‬
‫أ‬ ُ ‫َم‬
ُ ‫ك‬
‫ثِِف‬ َ ‫اس‬
‫ِِفي أ‬َ َّ ‫ع‬
‫ِالن‬ ُ ‫ف‬
َ ‫ماِيَ أن‬ َّ َ‫وأ‬
َ ِ‫ما‬ َ
{9}

C. Communication with patient through words and language


D. Process to encourage the expression of feelings and needs.
Answer: D
38) Click image to enlarge A 56 - year - old patient presents with psoriasis. Secondary
lesions are present on the
upper and lower extremities
(see image). Which of the
following best describes the
appearance of psoriasis?
A. Scales
B. Crusts
C. Scars
D. Fissures
Answer: A
‫” نركز في كل األنواع بشكل الصور ألنه‬
"‫بيجيب الصورة ويسأل على االسم‬
39) While taking the history from a new patient, the nurse densified that he had
hypomanic episode which was alternating with a mirror depressive\episode for the last
two years. what is the most likely diagnosis?
A. Bipolar I disorder
B. Bipolar II disorder
C. Dysthymic disorder
D. Cyclothymic disorder
Answer: D
40) talking with the parents of a child with Down syndrome, which of the Police
would the appropriate foal for the care of the child?
A. Encourage self-care skills in the child.
B. Teaching the child something new each day
C. Encourage more lenient behavior limits for the Cho.
D. Achieving age-appropriate social skills
Answer: D

ِ‫يِاْلَ أرض‬
‫أ‬ ُ ‫َم‬
ُ ‫ك‬
‫ثِِف‬ َ ‫اس‬
‫ِِفي أ‬َ َّ ‫ع‬
‫ِالن‬ ُ ‫ف‬
َ ‫ماِيَ أن‬ َّ َ‫وأ‬
َ ِ‫ما‬ َ
{ 10 }

41) 28-year-old married woman is admitted in the Gynecology Ward for the
observation after her miscarriage. She has not been able to sustain any pregnancy
since she got married for the past three years and she had three miscarriages during
this time.
What intervention is the most appropriate?
A. Family history
B. Past medical history
C. Physical examination
D. Laboratory investigations
Answer: B
42) It is a managerial function that indicates leading the staff in the most effective
method.
A: Planning
B: Directing
C: Organizing
D: Controlling
Answer: B
43) which of the following best describes the role of a case management?
A. promotes healthy lifestyle.
B. provides coordination of care to ensure continuity.
C. possesses highly skilled communication methods.
D. promotes the comfort of the patient.
Answer: B
44) A 30-year-old female patient went to the clinic for a consultation after her work
due to Pain no both legs during prolongs standing and sitting Doppler ultrasound was
cone and confirmed the initial diagnosis of varicose veins.
The patient asked what is the possible surgical procedure in relation to her case?
A. Amputation
B. Sclerotherapy
C. Thermal ablation
D. Ligation and stripping

ِ‫يِاْلَ أرض‬
‫أ‬ ُ ‫َم‬
ُ ‫ك‬
‫ثِِف‬ َ ‫اس‬
‫ِِفي أ‬َ َّ ‫ع‬
‫ِالن‬ ُ ‫ف‬
َ ‫ماِيَ أن‬ َّ َ‫وأ‬
َ ِ‫ما‬ َ
{ 11 }

Answer: D
45) Leaving the tourniquet for a long-time cause?
A- Low blood flow
B- Limb embolism
C- Hemoconcentration
Answer: C
46) Which drug would be used to treat a patient who has increased intracranial
pressure (ICP) resulting from head trauma after an accident:
a-Mannitol
b-Atropine sulphate
c- epinephrine hydrochloride
d-Sodium bicarbonate
Answer: A
47) Leaving the tourniquet for a long-time cause:
A. Embolism
B. Low blood flow
C. Ischemia
Answer: B
48) When the nurse on duty accidentally bumps the bassinet. the neonate throws out
its arms. hands opened. and begins to cry. The nurse interprets this reaction as
indicative of which of the following reflexes?
A. Startle reflex
B. Babinski reflex
C. Grasping reflex
D. Tonic neck reflex
Answer: A
49) Women come to ER with her husband and the husband demonstrate she not talk
or voluntary eat since the son died in accident, what the women need according to
Maslow?

ِ‫يِاْلَ أرض‬
‫أ‬ ُ ‫َم‬
ُ ‫ك‬
‫ثِِف‬ َ ‫اس‬
‫ِِفي أ‬َ َّ ‫ع‬
‫ِالن‬ ُ ‫ف‬
َ ‫ماِيَ أن‬ َّ َ‫وأ‬
َ ِ‫ما‬ َ
{ 12 }

A. loving and belonging


B. psychological needs
C. spiritual support
D. family support
Answer: A
50) Jizan province is poised to achieve malaria elimination. There is a need to change
from a policy of passive case detection to reactively and proactively detecting
infectious reservoirs that require new approaches to surveillance. For discovering health
problems and diseases inside the community. What is the priority of surveillance?
A. Social surveillance
B. Financial surveillance and previous health diseases.
C. Incidence and prevalence of the health diseases.
Answer: C
51) Jizan province is poised to achieve malaria elimination. There is a need to change
from a policy of passive case detection to reactively and proactively detecting
infectious reservoirs that require new approaches to surveillance. For discovering health
problems and diseases inside the community. What is the priority of surveillance?
A. Social surveillance
B. Public surveillance and prevent infection.
C. Financial surveillance and previous health diseases.
Answer: B
52) Jizan province is poised to achieve malaria elimination. There is a need to change
from a policy of passive case detection to reactively and proactively detecting
infectious reservoirs that require new approaches to surveillance. For discovering health
problems and diseases inside the community. What is the priority of surveillance?
A. Social surveillance
B. Surveillance for previous and new health problems
C. Financial surveillance and previous health diseases.
Answer: B
53) Why do we use “log rolling" Technique?
A) Facility good circulation

ِ‫يِاْلَ أرض‬
‫أ‬ ُ ‫َم‬
ُ ‫ك‬
‫ثِِف‬ َ ‫اس‬
‫ِِفي أ‬َ َّ ‫ع‬
‫ِالن‬ ُ ‫ف‬
َ ‫ماِيَ أن‬ َّ َ‫وأ‬
َ ِ‫ما‬ َ
{ 13 }

B) Avoid movement
C) To protect the spinal cord from moving
Answer: C
54) A 38-year- old patient is about to have lumber disk surgery. during
preoperative care, the nurse instructs the patient including the family.
members how to do "log rolling" to change patient position. One of the family
members ask why they must do such action in turning the patient.
postoperative.
Which of the following is the nurse best response?
A. Facility good circulation
B. Avoid movement
C. Prevent post-operative bed sore.
D. Makes changing of patient position easier.
Answer: C
55)The nurse is caring for 6 patients. Providing for them all care and activities. What is
the type of nurse according to nursing care delivery system?
A⁃ primary
B⁃ Case management
C⁃ Functional
Answer: A
56) The patient is receiving hemodialysis. The patient is experiencing disequilibrium
syndrome. What is the cause of the syndrome?
A. Increase urea and Na
B. Decrease urea and Na
Answer: B
57) The pregnant woman is experiencing muscle cramps in her leg. She has less
calcium intake. What should the nurse instruct her during spasm?
A. knee flexion

ِ‫يِاْلَ أرض‬
‫أ‬ ُ ‫َم‬
ُ ‫ك‬
‫ثِِف‬ َ ‫اس‬
‫ِِفي أ‬َ َّ ‫ع‬
‫ِالن‬ ُ ‫ف‬
َ ‫ماِيَ أن‬ َّ َ‫وأ‬
َ ِ‫ما‬ َ
{ 14 }

B. knee extension
Answer: B
58) Classical sign of adrenal insufficiency?
A. Hypernatremia
B. Hypotension
C. Hyperpigmentation
Answer: C
59) position after appendectomy?
A-high fowlers
B-semi fowler
C-sitting
D-Dorsal recumbent
Answer: B
60) How the nurse ensure that the mother is breast feeding well?
A. It is normal that suppose you know
B. Referral to breast feeding educator for one day
C. Check mother while she is breast feeding and progression✅

Answer: C
61) What is type of transport that carry oxygen and carbon dioxide from and to
placenta during pregnancy?
A. Simple diffusion
B. Active transport
C. Simple transport
Answer: A
62) Patient has difficulty in walking and lethargic, there decrease in capillary refill and
urine incontinence, has risk for?
A-aspiration
B-infection

ِ‫يِاْلَ أرض‬
‫أ‬ ُ ‫َم‬
ُ ‫ك‬
‫ثِِف‬ َ ‫اس‬
‫ِِفي أ‬َ َّ ‫ع‬
‫ِالن‬ ُ ‫ف‬
َ ‫ماِيَ أن‬ َّ َ‫وأ‬
َ ِ‫ما‬ َ
{ 15 }

C-skin breakdown
D-body disturbance
Answer: C
63) child with thalassemia pale and activity intolerance what the priority?
A-RBC transfusion
B-oxygen administration
Answer: B
64) The nurse is giving instruction for a pregnant woman. Which of the following should
be avoided during pregnancy?
A-raw uncooked met.
B-clean well fruit and vegetables
C-cooked fish and chicken
Answer: A
65) child has urine incontinence two day ago with WBC in urine.?
A-psychological abuse
B-urinary trac infection
C-genitalia defect
Answer: B
66) pt. has difficulty taking liquid medications from a cup. How should the nurse
administer the medications?
A. Request that the physician change the order to the IV route.
B. Administer the medication by the IM route.
C. Use a needleless syringe to place the medication in the side of the mouth.
D. Add the dose to a small amount of food or beverage to facilitate swallowing.
Answer: C
67) Patient diagnosed with appendicitis Which statement by patient indicates absolute
ruptured of appendicitis?
A -I feel like I want to vomit.

ِ‫يِاْلَ أرض‬
‫أ‬ ُ ‫َم‬
ُ ‫ك‬
‫ثِِف‬ َ ‫اس‬
‫ِِفي أ‬َ َّ ‫ع‬
‫ِالن‬ ُ ‫ف‬
َ ‫ماِيَ أن‬ َّ َ‫وأ‬
َ ِ‫ما‬ َ
{ 16 }

B- Pain in the epigastric

C - Painless in right quadrant / No hurts suddenly ✔️

Answer: C
68) Nursing supervisor newly elected Wants to use" disciplinary evaluation to nurses’
behavior " What type of power she is using?
A. Reward
B. Coercive
C. legitimate
D. Formal
Answer: Bbb
69) The patient weight 67 kg. He is receiving drug 30mg /kg/24hr. How many grams
should the nurse give for 24 hr.?
A.5 g
B. 1.5g
C. 1 g
D. 2g
Answer: D
70) Based upon the legal guidelines for documentation, which of following corrective
action is correct?
A Never erase entries do line under ward and write correct
B. erase the incorrect notes then write the correct
Answer: A

ِ‫يِاْلَ أرض‬
‫أ‬ ُ ‫َم‬
ُ ‫ك‬
‫ثِِف‬ َ ‫اس‬
‫ِِفي أ‬َ َّ ‫ع‬
‫ِالن‬ ُ ‫ف‬
َ ‫ماِيَ أن‬ َّ َ‫وأ‬
َ ِ‫ما‬ َ
{ 17 }

71) The nurse is caring for child regarding his


activity. Which of the following is consider fine?
motor activity?
A. Ride bicycle
B. Walking
C. Holding cup
Answer: C
72) When the Zygote enter to the uterus after
fertilization?
A. 2 day
B. 4 day
C. 6 day
D. 8 day
Answer: B
*Take care:
*Zygotes enter to the uterus 3 _ 5 day

* Implant 7 _ 10 day after fertilization

73) A nurse is assisting during a normal vaginal delivery on a 22-year diabetic patient.
The head was delivered without any complication head suddenly retracts against the
perineum prompting the physician to immediately ask for the nurse assistance with this
dystocia. Which of the following will be the nurse appropriate action to impacted
shoulders of the infant?
A. Fracture the infant’s clavicle.
B. Prepare patient for immediate caesarean section.
C. Apply fundal pressure to displace anterior shoulder.
D. Perform supra public pressure to release anterior shoulder.
Answer: D

ِ‫يِاْلَ أرض‬
‫أ‬ ُ ‫َم‬
ُ ‫ك‬
‫ثِِف‬ َ ‫اس‬
‫ِِفي أ‬َ َّ ‫ع‬
‫ِالن‬ ُ ‫ف‬
َ ‫ماِيَ أن‬ َّ َ‫وأ‬
َ ِ‫ما‬ َ
{ 18 }

74) The community nurse was providing home visit for a family to give health instruction.
The nurse observed while she entered the garage, she was finding the car's oil had
expired. What would you do?
A. get rid of oil and throw it away junk.
B. Call to the company and change it.
C. Use the oil
D. Give it to the neighbors.
Answer: B
75) What are the symptoms of the BCG vaccination side effect?
A. Diarrhea
B. Skin ulcer or scar
C. no symptoms
D. Seizure
Answer: B
76) How many times shower the baby?
A- twice a week with warm water
B- Every day with warm water
C- Every day with warm water and acidic soap
D- Every day with warm water and alkaline soap
Answer: A
77) A nurse is caring for a 72-year-old man patient, is unsteady. The patient requests the
nurse to help nurse asked the patient to wait few minutes' device to transfer him. Which
of the following transfer devices is the nurse to use?
A. Belt
B. Board
C. Handle
D. Mechanical lift
Answer: A

ِ‫يِاْلَ أرض‬
‫أ‬ ُ ‫َم‬
ُ ‫ك‬
‫ثِِف‬ َ ‫اس‬
‫ِِفي أ‬َ َّ ‫ع‬
‫ِالن‬ ُ ‫ف‬
َ ‫ماِيَ أن‬ َّ َ‫وأ‬
َ ِ‫ما‬ َ
{ 19 }

78) Neonate is near to cold window what is the type of heat loss?
A. Radiation
B. Convection
C. Conduction
D. Evaporation
Answer: A
79) The nurse has been teaching a new mother how to feed was born with a cleft lip
and palate before surgical
repair of Which of the following action from the mother indicate teaching has been
successful?

A. burping the baby frequently ✅✅

B. Prevent the infant from crying


C. Placing the baby flat during feeding
D. Keep the infant prone following feedings
Answer: A
80) The age of Cleft lip surgery is 2 – 6 months.
81) Definition of Word Salad:
words or phrases are connected meaninglessly being forced.
82) A 49-year-old patient was admitted in the Medical Ward to rule out his pale
appearance, loss of appetite, abdominal pain in the right upper quadrant, dark urine
and grey colored stools for the last few days. He was diagnosed with Hepatitis C and
was discharge from the hospital when his condition gets stable. He was given the
advice to take his medication regularly. What should be the nurse’s prime focus in the
discharge teachings for the patient?
A. Family support
B. Dietary counselling
C. Activity and exercise
D. Understanding of the disease
Answer: D

ِ‫يِاْلَ أرض‬
‫أ‬ ُ ‫َم‬
ُ ‫ك‬
‫ثِِف‬ َ ‫اس‬
‫ِِفي أ‬َ َّ ‫ع‬
‫ِالن‬ ُ ‫ف‬
َ ‫ماِيَ أن‬ َّ َ‫وأ‬
َ ِ‫ما‬ َ
{ 20 }

83) The nurse is assessing a 2 -years-old child with Wilms surgery Which of the following
should the nurse avoid?
A. Putting the child in lateral position.
B. Palpating the child's abdomen.
C. Putting the child in a private room.
D. Provide mouth hygiene 30 minutes after meal.
Answer: B
84) Mother came to the Outpatient Department with an infant having cleft and
palate. The infant was underweight, so the nurse has to consider Teaching the proper
way of feeding the child in the treatment plan. Which of the
following is the proper way of feeding
A. Use a non-squeezable bottle during feeding
B. Feed infant in an upright, sitting position ✅✅

C. Enlarge nipple holes of bottle to allow more milk to pass through


D. Feed infant longer than 45 minutes to allow more food to be small
Answer: B
85) What is the type of umbilical cord care for neonate?
A. Curing and treatment
B. Infection prevention
Answer: B
86) After lumbar puncture position?
A. Supine
B. Prone
Answer: A
87) Elderly Immunization - - - - - - Pneumonia, Influenza
88) what is the action to prevent cleft lip & palate?
A. Give immunizations
B. Advice pregnant folic acid

ِ‫يِاْلَ أرض‬
‫أ‬ ُ ‫َم‬
ُ ‫ك‬
‫ثِِف‬ َ ‫اس‬
‫ِِفي أ‬َ َّ ‫ع‬
‫ِالن‬ ُ ‫ف‬
َ ‫ماِيَ أن‬ َّ َ‫وأ‬
َ ِ‫ما‬ َ
{ 21 }

Answer: B
89) Which of the following vitamin supplements can decrease the incidence of Neural
tube defects such as anencephaly
and spina bifida new-born?
A. Vitamin A
B. Riboflavin
C. Folic Acid
D. Vitamin K
Answer: C
90) As identified by DR Elizabeth Kubler which stage of dying is characterized by the
transition from ‘NO’ not me to “yes, me but....”
A. Anger
B. Depression
C. Acceptance
D. Bargaining
Answer: D
91) The head nurse gives low evaluation to the nurse because of her late. What is the
type of her evaluation effect?
A. Halo effect
B. Horn effect
C. Central tendency
Answer: B
92) The head nurse is too busy to give evaluation to her staff. She gave to everyone
average evaluation. What is the evaluation type that the head nurse used?
A. Halo effect
B. Horn effect
C. Central tendency
Answer: C

ِ‫يِاْلَ أرض‬
‫أ‬ ُ ‫َم‬
ُ ‫ك‬
‫ثِِف‬ َ ‫اس‬
‫ِِفي أ‬َ َّ ‫ع‬
‫ِالن‬ ُ ‫ف‬
َ ‫ماِيَ أن‬ َّ َ‫وأ‬
َ ِ‫ما‬ َ
{ 22 }

93) nurse educator said to a new nurse" our hospital provides the highest level of
specialized healthcare in an integrated educational and research setting" which of the
following aspects of the hospital's strategic plan is indicated by this statement?
A. Vision
B. Policy
C. Values
D. Mission
Answer: D
94) New-born has small, whitish, pinpoint spots over the nose, which the nurse knows are
caused by retained sebaceous secretions. When charting this observation, the nurse
identifies it as:
A. Milia
B. Lanugo
C. Whiteheads
D. Mongolian spots
Answer: A
95) The community health care Facility observed low response for immunizations
activities. Then it decided to teach people about immunizations and the importance
for applying it. What would improve the effect of response?
A. Modify immunizations program schedule.
B. Provide more health teaching programs.
Answer: B

96) NORMAL FETAL HR 120-160


97) Which patient to see first:
A. Patient with Potassium level 7.5
B. Anemia client with pain level 6/10
Answer: A

ِ‫يِاْلَ أرض‬
‫أ‬ ُ ‫َم‬
ُ ‫ك‬
‫ثِِف‬ َ ‫اس‬
‫ِِفي أ‬َ َّ ‫ع‬
‫ِالن‬ ُ ‫ف‬
َ ‫ماِيَ أن‬ َّ َ‫وأ‬
َ ِ‫ما‬ َ
{ 23 }

98) The speed of transmission of the disease from person to person?

A. Infective✅

B. Infection
C. Infectivity
D. Precaution
Answer: A

99) Why should the community nurse know about the disease in other countries?
A-To give patient needs accordingly ✔️✔️.

B-To know the perveance of the disease in certain country.


C- To help countries for solutions
D- To make research and studies
Answer: A
100) HIV patient will be risk for which life threating disease?
A. Pneumonia
B. Tuberculosis
Answer: B
101) couple has a son and a daughter, and both moved out and got family on their
own
Which type of family structure?
A. Extended
B. Blended
Answer: A
102) Patient with cardiac tamponed muffled heart sound Which of the following the
nurse monitor?
A. Pulse and respiratory rate.
B. Paradoxical pulse 40 in inspiration.
Answer: B

ِ‫يِاْلَ أرض‬
‫أ‬ ُ ‫َم‬
ُ ‫ك‬
‫ثِِف‬ َ ‫اس‬
‫ِِفي أ‬َ َّ ‫ع‬
‫ِالن‬ ُ ‫ف‬
َ ‫ماِيَ أن‬ َّ َ‫وأ‬
َ ِ‫ما‬ َ
{ 24 }

103) Treatment of pyloric stenosis?


* Rehydration and pyloromyotomy
104) Which medicine prescription the nurse should question?
A. Lasix 40mg oral BID
B. Codeine oral PRN
Answer: B
105) Float nurse from medicine floor. Came to telemetry unite Which patient she
receives?
A. MI patient in heparin infusion
B. Hypertensive patient on Lasix.

C. Atrial Fibrational with anticoagulant drug.

Answer: C
106) Cause of death in Saudi Arabia?
*ischemic heart disease and road accident.
107) Diphtheria which type of isolation?
A. Droplet
B. Airborne
Answer: A
108) Patient with schizophrenia seen with which of the following signs?
A -Word salad
B -Associates lessness
Answer: B
109) Why hemorrhage occurs postpartum?
A-prolonged labor last 12 hours
B-excusive analgesics given during labor.
Answer: B

ِ‫يِاْلَ أرض‬
‫أ‬ ُ ‫َم‬
ُ ‫ك‬
‫ثِِف‬ َ ‫اس‬
‫ِِفي أ‬َ َّ ‫ع‬
‫ِالن‬ ُ ‫ف‬
َ ‫ماِيَ أن‬ َّ َ‫وأ‬
َ ِ‫ما‬ َ
{ 25 }

110) How measure Orthostatic hypotension?


* The Systolic is <20, and the Diastolic is <10.
111) A male client has received a prescription for orlistat for weight and nutrition
management. In addition to the medication, client states plan to take a multivitamin.
what teaching should a nurse provide?
A. Following a well-balanced diet is a much healthier approach to good nutrition
than depending on a multivitamin. Be sure to take the multivitamin and the
medication at least two hours apart.
B. As a nutritional supplement, orlistat contains all the recommended daily vitamins
and minerals.
C. Multivitamins are contraindicated during treatment with weight control
medications such.
D. as orlistat

Answer: A
*Take care:
Disfral or Orlistat is medication of (Beta-thalassemia) - - - - - take multiple vitamins and
this medicine 2 hour apart.
112) How to prevent cleft lip and palate from occurring?
A-eat green vegetables and citrus fruit.
B- Drink too much juice
Answer: A
113) IQ 45 what level of mental retardation?
A-moderate mental retardation.

B-severe mental retardation.


C-mild mental retardation.
Answer: A
114) IQ is mild retardation characteristic?
A-needs assistance all time.
B-can learn some of daily life activities but still needs helps.
Answer: B

ِ‫يِاْلَ أرض‬
‫أ‬ ُ ‫َم‬
ُ ‫ك‬
‫ثِِف‬ َ ‫اس‬
‫ِِفي أ‬َ َّ ‫ع‬
‫ِالن‬ ُ ‫ف‬
َ ‫ماِيَ أن‬ َّ َ‫وأ‬
َ ِ‫ما‬ َ
{ 26 }

115) Nurse plan community program to those who experienced a disease


And assess the recovery or rehabilitation or dis ability Which level of prevention?
A-primary
B-secondary
C-tertiary
Answer: C
116) What is an example of primary level of prevention?
A-how to be smoking cessation.
B-Given education on HIV to high school students.
Answer: B
117) During haji nurse is caring for non-Arabic speaker What is most appropriate action?
A-Use google translation.
B-ask family members to interpret.
C-call hospital Interpreter services
Answer: C
118) A5 - year - old child was brought to the Emergency Room with a fractured of
forearm. He had several bruises on his body but showed no signs of pain while
palpating them. He seemed scared and did not answer any questions asked. How
should the nurse initiate therapeutic communication with the child?
A. Start interviewing
B. Encourage him to speak.
C. Explain about the fracture
D. Greet and show gentleness - nurse gives health.
Answer: D
119) The nurse administered a prescribed intramuscular medication to a patient during
a home health visit. How should the nurse dispose of the used needle and syringe?
A) Recap the needle, then place the needle and syringe into a waterproof.
container until safe disposal can be made.
B) Bend the needle back towards the barrel of the syringe before putting.

ِ‫يِاْلَ أرض‬
‫أ‬ ُ ‫َم‬
ُ ‫ك‬
‫ثِِف‬ َ ‫اس‬
‫ِِفي أ‬َ َّ ‫ع‬
‫ِالن‬ ُ ‫ف‬
َ ‫ماِيَ أن‬ َّ َ‫وأ‬
َ ِ‫ما‬ َ
{ 27 }

the needle and syringe in a metal trash container.


C) Wrap the needle and syringe in disposable paper before putting the
needle and syringe into the dirty section of the nurse’s equipment bag.
D)Put the needle and syringe directly into a puncture-resistant plastic.
container that has a lid.
Answer: D
120) Patients eat contaminated food and the diagnosed with hepatitis A as result of
that food which of the following s related to hepatitis A disease: -
A-The disease is transmitted through the blood.
B- The disease is transmitted through sexual contact.
C- Incubation period 3-5weeks.
D- Incubation period 4-5 month
Answer: C
121) A 16-year-old girl developed an infection over the surface of the heart after
having her nose pierced to place jewelry on admission, she had developed pyrexia of
38C, a heart murmur and petechiae over the whole body. She was admitted to the
hospital and treated with intravenous antibiotics. The nurse explains to the patient that
she is at high-risk for re-infection and provides discharge teaching on preventive
measures. Which of the following would most likely require prophylactic treatment?
A. pelvic examination
B. dental. Care
C. bronchoscopy
D. urinary catheterization
Answer: B
122) 5-year-old child was admitted with Nephrotic Syndrome. A nurse noticed that the
child has slight facial puffiness with mild pitting edema on his hands and feet. there was
no distended abdomen diet the nurse should order for the child.
A. High protein, high salt diet
B. Low protein, low fiber diet
C. Low protein, normal salt diet
D. Normal protein, low salt diet
Answer: D

ِ‫يِاْلَ أرض‬
‫أ‬ ُ ‫َم‬
ُ ‫ك‬
‫ثِِف‬ َ ‫اس‬
‫ِِفي أ‬َ َّ ‫ع‬
‫ِالن‬ ُ ‫ف‬
َ ‫ماِيَ أن‬ َّ َ‫وأ‬
َ ِ‫ما‬ َ
{ 28 }

123) Managers create a new position patient educator work with head nurse, what
organizational relationship?
A. square
B. solid line
C. matrix
D. Dotted line
Answer: D
124) There is a new community nurse assigned to work in a Makkah, one of the most
multicultural cities in Saudi Arab has to interact with multicultural population every day.
What is the most important step the nurse should do before with the clients?
A. Asking the client about his/her cultural background
B. Conducting an appropriate culturological assessment.
C. Ensuring that the client has his/her a privacy.
D. Looking at the client's file to take history.
Answer: B
125) Doctor writes DNR on a patient with end stage cancer, the patient tells the nurse
and express his feelings he wants to be resuscitation, what is your nursing action?
A. explain to the patient to except DNR.
B. ignore the patient feeling.
C. Follow the hospital policy.
D. Report the doctor to Saudi Commission for Health Specialties
Answer: C
126) A nurse is taking care of a woman who is in active labor. She is multipara at 38
weeks gestation. Her cervix is 5 cm dilated at -2 station with ruptured membranes. On
the fetal heart rate tracing, the nurse observes a fetal heart rate baseline of 150 beats
per minute with Deceleration in fetal heart rate to as low as 100 beats per minute.
Deceleration in heart rate occurs both during and between contractions. With each
deceleration heart rate quickly returns to baseline after a few seconds. The nurse has
anticipated which of the following.?
A. Client will probably deliver quickly.
B. Client may need rapid intervention for a prolapsed cord.
C. Client has a normal fetal heart rate tracing for a multipara in active labor.
D. Client has a fetal heart rate tracing consistent with poor uteroplacental blood
flow.
Answer: B

ِ‫يِاْلَ أرض‬
‫أ‬ ُ ‫َم‬
ُ ‫ك‬
‫ثِِف‬ َ ‫اس‬
‫ِِفي أ‬َ َّ ‫ع‬
‫ِالن‬ ُ ‫ف‬
َ ‫ماِيَ أن‬ َّ َ‫وأ‬
َ ِ‫ما‬ َ
{ 29 }

127) Mother of nine children, three of them with congenital anomalies


and one down syndrome: she is a primary school graduate, with low.
financial status. She is not using any method of family planning. So, the
primary health care nurse has referred her for counseling. Which of the
following indicates that the summarizes the important points during
counseling?
A. Provision broad ideas to the client
B. Restatement for better understanding
C. Keeping silent while the client asking questions.
D. Interpreting feelings and resistance of the client
Answer: B
128) What percentage of patients with ectopic pregnancy will have
normally rising HCG levels?
A. 10%
B. 25%
C. 50%
D. 95%
Answer: B
129) An old patient admitted to ICU with signs and symptoms of Dyspnoea, cough,
expectoration, weakness, and edema. Which of the following conditions are correct
related these symptoms?
A. Pericarditis
B. Hypertension
C. Obliterative
D. Restrictive
Answer: A
130) The nurse prepares cardiac patient for the insertion of a pulmonary artery
catheter (Swan-Ganz catheter). The nurse teaches the patient that the catheter will be
inserted to provide information about.
A. Stroke volume
B. Venous pressure
C. Cardiac output
D. left ventricular functioning.
Answer: Ddd

ِ‫يِاْلَ أرض‬
‫أ‬ ُ ‫َم‬
ُ ‫ك‬
‫ثِِف‬ َ ‫اس‬
‫ِِفي أ‬َ َّ ‫ع‬
‫ِالن‬ ُ ‫ف‬
َ ‫ماِيَ أن‬ َّ َ‫وأ‬
َ ِ‫ما‬ َ
{ 30 }

131) Which of the following conditions would a nurse not administer erythromycin?
A. Campylobacteriosis infection
B. Multiple Sclerosis
C. Legionnaires disease
D. Pneumonia
Answer: B
132) A nurse is assessing a patient who just arrived in the emergency department after
a motor vehicle collision the patient has a strong smell of alcohol on breath is restless
and has a bluish discoloration on the abdomen by the umbilical. The patient vital signs
or temperature of 37.2 heart rate 120 beats per minute and blood pressure of 100/ 62
mmhg while other members of the team are evaluating the patient the nurse should
obtain.
A. A pair of elastic support stockings
B. a chest tube insertion tray
C. Supplies for peritoneal lavage
D. a Vial of Hydralazine
Answer: C
133) A patient is attending a primary health clinic for regular checkup. He complains of
constipation. After assessment, the nurse instructed him to consume bulk-forming foods.
Which of the following is the best bulk-forming foods?
A. Fruit juice
B. Raw meat
C. Whole grains
D. Milk products
Answer: C
134) A nurse in the postnatal ward is assigned for a multiparous patient has just
delivered a healthy new-born. When should the nurse plan to take the patient vital
signs?
A. Every hour for the first 2 hours
B. Every 30 minutes during the first hour
C. Every 15 minutes during the first hour
D. Every 5 minutes for the first 30 minutes
Answer: C

ِ‫يِاْلَ أرض‬
‫أ‬ ُ ‫َم‬
ُ ‫ك‬
‫ثِِف‬ َ ‫اس‬
‫ِِفي أ‬َ َّ ‫ع‬
‫ِالن‬ ُ ‫ف‬
َ ‫ماِيَ أن‬ َّ َ‫وأ‬
َ ِ‫ما‬ َ
{ 31 }

135) post-operative patient who underwent an abdominal procedure request pain


medication from the nurse and rates the pain at a level nine. There is a standing order
for narcotic administration. When the nurse opera the narcotic box and performs a
count, the number of pills remaining in the box is different than the number of pills.
recorded on the sheet. What is the most appropriate initial nursing action?
A. Notify the nursing supervisor.
B. Write the finding on the narcotic sheet.
C. Administer the patient's requested medication.
D. Identify the last nurse who used the narcotic.
Answer: A
136) Women should get their pap smear test each.
A-6-month
B-1year
C-3years
D-5 years
Answer: C
137) A woman was diagnosed gestational trophoblastic disease what is the lab
investigation was done to diagnose condition?
A. Cervical pap smear
B. Serum HCG levels
C. Serum estrogen level
D. Plasma thyroxin level
Answer: B
138) The community midwife has just finished an educational session with a group of
women regarding maternal health and annual gynecological examination along with
mammogram and Pap smear testing. What outcome is the most desired?
A. Disease prevention.
B. Increased healthy living.
C. Early detection of related issues.
D. Strengthened marital relationship.
Answer: C
139) Female patient has been advised that laboratory tests confirm herpes simplex virus
(HSV), type 2. The nurse should teach the patient that a Papanicolaou test (Pap smear)
is recommended:

ِ‫يِاْلَ أرض‬
‫أ‬ ُ ‫َم‬
ُ ‫ك‬
‫ثِِف‬ َ ‫اس‬
‫ِِفي أ‬َ َّ ‫ع‬
‫ِالن‬ ُ ‫ف‬
َ ‫ماِيَ أن‬ َّ َ‫وأ‬
َ ِ‫ما‬ َ
{ 32 }

A. Every 6 months if symptoms persist despite treatment.


B. Every year even if asymptomatic whenever symptoms recur.
C. Every 3 years if other Pap smears have been negative.
Answer: A
140) A female patient complains of abdominal discomfort. Watery stool has been
leaking from her rectum. This could be a sign of which of the following?
A. fecal impaction ✅

B. constipation
C. bowel incontinence
D. diarrhea
Answer: A
141) Which of the following religion is prohibited the blood transfusion?
A. Christian
B. Muslims
C. Jewish
D. Jehovah
Answer: D
142) A patient, who had abdominal surgery six days ago, has been ambulating in the
halls without much difficulty. however, on seventh postoperative day, the patient
complains of increased pain at incisional site and is walking bent over. What is the most
likely cause?
A. Intestinal inflammation
B. Pulmonary edema
C. Wound infection
D. Deep vein thrombosis
Answer: C
143) A 29-year-old woman had been diagnosed with a 3 cm ovarian cyst. Which of the
following is the appropriate step in management?
A. Cyst aspiration
B. Hormonal therapy
C. Cyst removal by laparoscopy

ِ‫يِاْلَ أرض‬
‫أ‬ ُ ‫َم‬
ُ ‫ك‬
‫ثِِف‬ َ ‫اس‬
‫ِِفي أ‬َ َّ ‫ع‬
‫ِالن‬ ُ ‫ف‬
َ ‫ماِيَ أن‬ َّ َ‫وأ‬
َ ِ‫ما‬ َ
{ 33 }

D. Examination after next menstruation


Answer: D
144) 25-year-old woman has a family history of breast cancer. The nurse reviews the
procedure for breast self- examination (BSE) and tells her that the best time for a
woman to perform a breast self-examination is:
A. a few days before her period.
B. during her menstrual period.
C. on the last day of menstrual flow.
D. 3- 7days after the beginning of her period
Answer: D
145) The nurse is instructing a female client how to do breast self-exam. Which of the
following is the best time to perform this exam?
A. After ovulation
B. After period
C. Two weeks after period
D. Three days before period
Answer: B
146) At labor room, a nurse assessed the condition of the patient and gathered the
following data. Cervical dilatation 2-5minutes lasting 40-60seconds increasing bloody
show leg discomfort with heaviness what is the significance of the data?
A. Patient on the first stage of labor
B. Patient on the third stage of labor
C. Patient on the second stage of labor
D. Patient is experiencing a prolonged labor.
Answer: A
147) Establishes baby's hydration A 11-month-old boy with cleft lip & palate was
discharged from the hospital after surgery. The nurse provided the parents with
teaching. Which of the following statement by the parents indicate appropriate
understanding of the teaching instruction?
A. I should avoid that my infant's cry as much as possible.
B. I will use a spoon to feed my child after the surgery.
C. I will put my child in his abdomen after feeding.

ِ‫يِاْلَ أرض‬
‫أ‬ ُ ‫َم‬
ُ ‫ك‬
‫ثِِف‬ َ ‫اس‬
‫ِِفي أ‬َ َّ ‫ع‬
‫ِالن‬ ُ ‫ف‬
َ ‫ماِيَ أن‬ َّ َ‫وأ‬
َ ِ‫ما‬ َ
{ 34 }

D. I will use lotion to ease my child's lip irritation.


Answer: A
148) Epidural anesthesia site?
A. L3 - L4
B. L1 - L2
C. T3 - T4
D. C1 - C2
Answer: A
149) The scale used by a nurse to assess the gestational age of a newborn is:
A. Bishop score
B. Ballard score ✅

C. Bel ward score


D. Apgar score
Answer: B

150) On assessing the client pressure


ulcer (see photo), the nurse would
document this as which stage? A
Stage II
B. Stage II
C. Stage III
D. Stage IV
Answer: A

151) Adult -15- years old hospitalized to relieve anxiety. What should the nurse do?
A. Increase numbers of staff during nursing care

ِ‫يِاْلَ أرض‬
‫أ‬ ُ ‫َم‬
ُ ‫ك‬
‫ثِِف‬ َ ‫اس‬
‫ِِفي أ‬َ َّ ‫ع‬
‫ِالن‬ ُ ‫ف‬
َ ‫ماِيَ أن‬ َّ َ‫وأ‬
َ ِ‫ما‬ َ
{ 35 }

B. Limited numbers of staff during nursing care✅

C. Allow parents visit him along daytime


Answer: B
152) Nurse is caring for a client with a nasogastric tube that is attached to low suction
The nurse monitors the client, knowing that the client is at risk for which acid-base
disorder?
A. Metabolic acidosis
B. Metabolic alkalosis
C. Respiratory acidosis
D. Respiratory alkalosis
Answer: B
153) Nurse is preparing to insert a nasogastric tube into a client. The nurse places the
client in which position for insertion:
A. Right side
B. Low Fowler's
C. High Fowler's
D. Supine with the head flat
Answer: C
154) Mother of nine children, three of them with congenital anomalies Down
syndrome. she is a primary school graduate, with low status.
She is not using any method of family planning. So, the health care nurse has
referred her. for counselling
Which of the following application the counsellor can help?
Mrs. regarding family planning
A. Prevention level
B. Gather model.
C. Group teaching
D. Rejection of Mrs. M expression
Answer: B

ِ‫يِاْلَ أرض‬
‫أ‬ ُ ‫َم‬
ُ ‫ك‬
‫ثِِف‬ َ ‫اس‬
‫ِِفي أ‬َ َّ ‫ع‬
‫ِالن‬ ُ ‫ف‬
َ ‫ماِيَ أن‬ َّ َ‫وأ‬
َ ِ‫ما‬ َ
{ 36 }

155) Mother of nine children, three of them with congenital anomalies and
down syndrome she is a primary school graduate, with low financial status.
She is not using any. method of family planning Accordingly, the primary
health care nurse has referred her for counselling the following.
terms describe any restriction or lack of ability to perform an activity?
A. Impairment
B. Abnormality
C. Handicapped
D. Disability
Answer: D
156) Mother of nine children, three of them with congenital anomalies and one down
syndrome; she is a primary school graduate, with low financial status. She is not using
any method of family planning. Accordingly, the primary health care nurse has referred
her for counselling.
Which of the following is a barrier facing the team responsible for providing health
services to handicapped individuals?
A. Sensory limitations
B. Rigid rules
C. Developmental disabilities
D. Deafness & hearing limitations
Answer: C
157) Mother of nine children, three of them with congenital anomalies Down syndrome;
she is a primary school graduate, with low status. She is not using any method of family
planning. According to primary health care nurse referred her for counseling Which of
the following types of home visits that the community her nurse should conduct for this
client?
A. Systematic routine
B. Selective
C. Follow up
D.Field trip
Answer: C

ِ‫يِاْلَ أرض‬
‫أ‬ ُ ‫َم‬
ُ ‫ك‬
‫ثِِف‬ َ ‫اس‬
‫ِِفي أ‬َ َّ ‫ع‬
‫ِالن‬ ُ ‫ف‬
َ ‫ماِيَ أن‬ َّ َ‫وأ‬
َ ِ‫ما‬ َ
{ 37 }

158) Mother of nine children, three of them with congenital anomalies and one down
syndrome; she is primary school graduate, with low financial status. She is not using any
method of family planning. So, the primary health care nurse has referred her for
counselling. Which of the following is the best health education method that can be
used?
A. community organization
B. individual counselling
C. group discussion
D. health class
Answer: B
159) Mother of nine children, three of them with congenital anomalies down syndrome;
she is a primary school graduate, with low first status. She is not using any method of
family planning. According to primary health care nurse has referred her counselling.
Which of the following phases of home visit accomplishes intervention?
A. Initial
B. Closing
C. Action
D. Terminal
Answer: A
160) Mother of nine children, three of them with congenital an down syndrome; she is a
primary school graduate, with status. She is not using any method of family planning. So
Health care nurse has referred her for counselling which of the following must be
focused on by the community nurse to provide an effective health education?
A. Educate regardless realistic objectives.
B. Use clear and concise language.
C. Use scientific terms during explanation.
D. Explain the negative consequences in the family.
Answer: B
161. A pregnant woman is 36 weeks pregnant was admitted to antenatal ward for
observation after being involved
in a car accident the refuses when the nurse tried to touch her abdomen to perform
examination saying it is
painful. She also present mild vaginal bleeding . Which of the following is the most
appropriate diagnosis
A. Placenta previa

ِ‫يِاْلَ أرض‬
‫أ‬ ُ ‫َم‬
ُ ‫ك‬
‫ثِِف‬ َ ‫اس‬
‫ِِفي أ‬َ َّ ‫ع‬
‫ِالن‬ ُ ‫ف‬
َ ‫ماِيَ أن‬ َّ َ‫وأ‬
َ ِ‫ما‬ َ
{ 38 }

B. Tubal pregnancy

C. Abruptio placenta ✅

D. Inevitable abortion
Answer: C
162) The nurse is assessing patient after craniotomy. The patient's blood pressure is
180/65 ICP is 25. What is the patient's cerebral perfusion pressure?
A. 72mmHg
B. 81mmHg
C. 78mmHg
D. 83mmHg
Answer: C
*Explanation Question 162:
CPP = MAP _ ICP
A
n
(Cerebral
s perfusion Pressure) = (Mean Arterial Blood Pressure) _ (Intracranial Pressure)
w 103 _ 25 So CPP = 78 mmHg
e
2(Diastolic BP)+ Systolic BP ) (2×65)+180 310
MAP=
r 3
MAP = 3
= 3 = 103
:
C
163) year-old woman patient in the Medical Ward is in a semiconscious. Her pancreatic
cancer is metastasized to her liver and lungs and she admitted for supportive
treatment. Her physicians discussed with the family that she will not be given the
Cardiopulmonary Resuscitation to save life if she goes into the cardiac arrest. Her two
sons agreed but the daughter is indecisive. Which of the following is the critical thinking
behind not providing discounted pulmonary resuscitation?
A. Unilateral judgment of health professionals
B. Refusal of patient’s right to treatment
C. Ethical dilemma and indecisiveness
D. Mercy killing to ease suffering.
Answer: D
164) Which of the following medication should the nurse clarify if given to post-delivery
client who had preeclampsia?
A-magnesium sulphate

ِ‫يِاْلَ أرض‬
‫أ‬ ُ ‫َم‬
ُ ‫ك‬
‫ثِِف‬ َ ‫اس‬
‫ِِفي أ‬َ َّ ‫ع‬
‫ِالن‬ ُ ‫ف‬
َ ‫ماِيَ أن‬ َّ َ‫وأ‬
َ ِ‫ما‬ َ
{ 39 }

B-methergine
C-Panadol
Answer: B
165) Which of the following is necessities for caesarean section?
A-preterm labor
B-severe preeclampsia
C-total placenta previa
D-partial placenta previa
Answer: C
166) Female patient will do the breast implant surgery and she tell the nurse not to tell
her family about the surgery. What is the nature of this action?
A-confidentiality
B-ethical dilemma
C-illegal nursing practice
D-medico-legal practice
Answer: A
167) Which of the following statement indicate nursing action during the first hour after
delivery of the placenta?

A. Monitor of mothers hemoglobin

B. Assess maternal vital signs every 15 minutes ✅

C. Ensure that the mother mobilize and empty her bladder

D. Administer 10 units of oxytocin via IV line to ensure uterus is well contracted.

Answer: B
168) A nurse is giving health education for a mother who has mastitis. Which of the
following if stated by the mother about what she needs to do, indicate the additional
education is needed?
A. Take antibiotics
B. Use analgesics

ِ‫يِاْلَ أرض‬
‫أ‬ ُ ‫َم‬
ُ ‫ك‬
‫ثِِف‬ َ ‫اس‬
‫ِِفي أ‬َ َّ ‫ع‬
‫ِالن‬ ُ ‫ف‬
َ ‫ماِيَ أن‬ َّ َ‫وأ‬
َ ِ‫ما‬ َ
{ 40 }

C. Wear a supportive bra.


D. Stop breast-feeding
Answer: D
169) A mother is in the midwife's clinic with the complaint of, redness and swelling in her
right breast. She is breast feeding her nine months-old baby. What should be immediate
intervention?
A. Start on antibiotics
B. Detailed investigations
C. Stop the breast feeding.
D. Wound cleaning and dressing
Answer: A
170) The nurse was planning care for a 25-year-old primigravida post-partum mother
who had engorgement due to poor feeding technique. the left breast appeared red
and swollen and was diagnosed as. Which of the following is the best education for the
mother?
A. Avoid wearing brassiere.
B. Begin suckling on the right breast.
C. Stop pumping milk from the left breast.
D. Take antibiotics till the soreness subsides.
Answer: D
171) A patient in surgical was transferred to isolation room after the wound swab
confirmed to have methicillin Resistant staphylococcus Atreus MRSA. Which of the
following measures should the nurse take to prevent infection in the ward?
A. Clean the Room three times a day.
B. Discard all soiled dressing into waste bag.
C. Instruct the patient to wash hands regularly.
D. Wear gloves and gown on every entry into the room
Answer: D
172) A nurse is providing instructions to a mother who has been diagnosed with mastitis.
Which of the following statements if made by the mother indicates a need for further
teaching?
A. I need to take antibiotics. And I should begin to feel better in 24-48 hours.

ِ‫يِاْلَ أرض‬
‫أ‬ ُ ‫َم‬
ُ ‫ك‬
‫ثِِف‬ َ ‫اس‬
‫ِِفي أ‬َ َّ ‫ع‬
‫ِالن‬ ُ ‫ف‬
َ ‫ماِيَ أن‬ َّ َ‫وأ‬
َ ِ‫ما‬ َ
{ 41 }

B. I can use analgesics to assist in alleviating some of the discomfort.


C. I need to wear a supportive bra to relieve the discomfort.
D. “I need to stop breastfeeding until this condition resolves.”
Answer: D
173) 33 old women presented to the ER with general weakness. The laboratory
investigation indicated VIT D deficiency. Which of the following nutrient should be
recommended as a good source of vitamin?
A. Rice
B. Green tea
C. Orange juice
D. Fish liver oils
Answer: D
174) Radon can cause any cancer.
A. skin
B. cervical
C. colon
D. prostatic
Answer: A
* Cause lung or pancreas cancer also.
175) What is the action of combined oral contraceptives?
A. induce thinning of endometrium and prevent ovulation.
B. Decrease the production of luteinizing hormone and follicle and prevent ovulation.
Answer: A
176) A patient who underwent a left foot amputation post-operative Care Unit and the
following assess (see lab results). Blood pressure 110/80 mmHg, Heart rate 65 /min,
Respiratory rate 13 /min Temperature 37.2C, Oxygen Saturation 98 % on room air
Test Result Normal Value
Hb 120 120-158 g/L
WBC 10.2 4-10.5 ×10
RBC 3.8 3.8-5.1 ×10
Fasting blood sugar 9.3 3.5-6.5 mm
LDL 6.5 <4.0 mm

ِ‫يِاْلَ أرض‬
‫أ‬ ُ ‫َم‬
ُ ‫ك‬
‫ثِِف‬ َ ‫اس‬
‫ِِفي أ‬َ َّ ‫ع‬
‫ِالن‬ ُ ‫ف‬
َ ‫ماِيَ أن‬ َّ َ‫وأ‬
َ ِ‫ما‬ َ
{ 42 }

Triglycerides 3.8 <2.16 mm


Which long-term complication is most likely?
A. Pain
B. Infection
C. Immobility
D. Bleeding
Answer: B
177) What is the management/treatment for woman who has gestational diabetes?
A. Diet management
B. Insulin
C. Oral diabetic drugs
Answer: A
178) A man is to be discharged from the General appendectomy. The precautionary
measures, plans are discussed with him. What is the most important desired outcome
after discharge?
A. Remain free of post-surgical complications.
B. Report fever, redness or drainage from the wound site
C. Use pain management techniques apropos.
D. Resume gradual activities and avoid weight.
Answer: A
179) A postpartum woman who was admitted for 24 hours, community nurse visited her
after 2 days of discharge. Which of these are abnormal findings?
A. Frequent urination
B. Lochia serosa
C. Uterus below umbilical level
D. Both breast full of milk
Answer: D
180) A postpartum woman who was admitted for 24 hours, community nurse visited her
after 2 days of discharge. Which of these are abnormal findings?
A. frequent urination
B. lochia serosa
C. uterus below symphysis pupils
D. breast full of milk

ِ‫يِاْلَ أرض‬
‫أ‬ ُ ‫َم‬
ُ ‫ك‬
‫ثِِف‬ َ ‫اس‬
‫ِِفي أ‬َ َّ ‫ع‬
‫ِالن‬ ُ ‫ف‬
َ ‫ماِيَ أن‬ َّ َ‫وأ‬
َ ِ‫ما‬ َ
{ 43 }

Answer: C
181) 6month-old boy with hydrocephalus is admitted to the pediatric surgical Ward for
ventriculoperitoneal Shunt (VPS) insertion. Which of the following findings should be of
the most concern when assessing the child postoperative?
A. Sunken fontanelle and irritability
B. decreased head circumference
C. poor feeding and pupillary change
D. headache and excessive sleepiness
Answer: C
182) 6month-old boy with hydrocephalus is admitted to the pediatric surgical Ward for
ventriculoperitoneal Shunt (VPS) insertion. Which of the following findings should be of
the most concern when assessing the child postoperative?
A. Plugging fontanelle
B. decreased head circumference
C. poor feeding and pupillary change.
D. headache and excessive sleepiness
Answer: A
183) 6 month-old boys with hydrocephalus is admitted to surgical Ward post
ventriculoperitoneal Shunt (VPS) What is the priority postoperative assessment???
A-Neurological Assessment
B. decreased head circumference
C. poor feeding and pupillary change.
D. headache and excessive sleepiness
Answer: A
184) A 65-year-old women visited the gynecological outpatient history reveals that she
had 3 pregnancies, one abortion gestational age, had 2 normal deliveries. She smokes
20 Her complaint is that she wets herself when she coughs embarrassing for her? Which
of the following can be considered as risk factors pelvic floor muscles?
A. Chronic coughing
B. Diabetes mellitus
C. Excessive spot

ِ‫يِاْلَ أرض‬
‫أ‬ ُ ‫َم‬
ُ ‫ك‬
‫ثِِف‬ َ ‫اس‬
‫ِِفي أ‬َ َّ ‫ع‬
‫ِالن‬ ُ ‫ف‬
َ ‫ماِيَ أن‬ َّ َ‫وأ‬
َ ِ‫ما‬ َ
{ 44 }

D. Sedentary lifestyle
Answer: D
185) After accessing patients' medical records, which behavior nurse shows that
patient’s confidentiality has been breached?
A. Reviews patients medical record
B. Read patients care plan.
C. Disclosing patient’s information.
D. Documents medication administered.
Answer: C
186) A 5-month-old boy has been vomiting green coloured vomit He has intermittent
abdominal pain during which he draws his chest, turns pale and cries forcefully. On
observation, the in the stool which has a jelly-like consistency. Abdominal pal along.
tube-like mass. There is no fever, rash nor diarrhoea are hyperactive in all quadrants.
Which is the most likely form of initial treatment?
A. Manual manipulation
B. Surgical resection
C. Normal saline enema
D. Laparoscopy
Answer: C
187) A 22-year-old gravida 2 para 1 with gestational age 38 week admitted to the
hospital. The chief complaint is decreased the fetal non-stress test revealed decreased
variability and fetal movement.
The next morning as part of the antenatal the nurse checks the fetal heart rate by
Doppler Sonic aid decreased the fetal heart rate to less than 100 /min. which of the
following action the nurse should do first?
A. Reassure the mother that the FHR is Ok.
B. immediately Notify the physician or midwife.
C. Reposition the patient to left lateral position.
D. Ask the mother about the pattern of fetal movement.
Answer: B

ِ‫يِاْلَ أرض‬
‫أ‬ ُ ‫َم‬
ُ ‫ك‬
‫ثِِف‬ َ ‫اس‬
‫ِِفي أ‬َ َّ ‫ع‬
‫ِالن‬ ُ ‫ف‬
َ ‫ماِيَ أن‬ َّ َ‫وأ‬
َ ِ‫ما‬ َ
{ 45 }

188) A child was admitted to the hospital three hours ago with an injury. The child
responds appropriately, but sluggishly to drifts in and out of sleep Which of the following
best describes this patient's level of?
A. Lethargic
B. Obtruded
C. Comatose
D. Semi-comatose
Answer: A
189) A patient was on a course of lithium carbonate drug. During the nurse found that
he complained from nystagmus visual hallucination, and oliguria Which of the following
drug related complications best symptoms?
A. Overdose
B. Mild toxicity
C. Severe toxicity
D. Moderate toxic
Answer: C
190) A nurse is planning to discharge a known HIV, the Isolation Unit after the recovery
from upper, which of the following nursing problem requires?
A. Risk of infection due to altered immune.
B. Fluid volume deficit due to frequent diarrhea
C. Anxiety due to disease, fear and social
D. Weight loss due to higher metabolism rate
Answer: A
191)An elderly patient who has an aortic aneurysm Intensive Care Unit to a Medical
Surgical Unit on day. While assessing the client, a nurse notes extremity and is unable to
palpate the pedal pulse.
Which intervention should the nurse implement?
A. Wrap the lower extremities with warm.
B. Use a Doppler ultrasound to reassess the
C. Elevate the extremities above heart level.

ِ‫يِاْلَ أرض‬
‫أ‬ ُ ‫َم‬
ُ ‫ك‬
‫ثِِف‬ َ ‫اس‬
‫ِِفي أ‬َ َّ ‫ع‬
‫ِالن‬ ُ ‫ف‬
َ ‫ماِيَ أن‬ َّ َ‫وأ‬
َ ِ‫ما‬ َ
{ 46 }

D. Place a bed cradle over the bed to levitate.


Answer: B
192) The head nurse of a Coronary Care Unit delegated the staff a senior nurse in that
unit What initial step must the head nurse implement before?
A. Check the hospital policies for delegating tasks.
B. Explain the task to the senior nurse.
C. Negotiate with the senior nurse.
D. Take the signature of the senior nurse.
Answer: A
193) 34-weeks-pregnant mother experiences a sudden gush o from her vagina and mild
uterine contractions. She informs about her condition and requests if she could wait
until the delivery. Which of the following is the best desired response for report to the
hospital?
A. Intravenous fluids and medicines need to be administered.
B. Observation is necessary to identify premature labour.
C. Pain and fluid flow both need to be controlled.
D. Fetal heart sound monitoring is necessary.
Answer: B
194) The nurse is assisting a patient to ambulate in hall. The patient has a history of
coronary artery disease (CAD) and had coronary artery bypass graft surgery (CABG) 3
days ago. The patient reports chest pain rated 3 on a scale of 0 (no pain) to 10 (severe
pain). The nurse should first:
A. Determine how long it has been since the patient's last dose of aspirin
B. Obtain a chair for the patient to sit down
C. Assess the patient's radial pulse
D. Ask the patient to take several slow, deep breaths
Answer: B
195) What is the benefit of Kegel exercise?
A- Increase strength of pelvic muscle
B- Induce cervix contraction.

ِ‫يِاْلَ أرض‬
‫أ‬ ُ ‫َم‬
ُ ‫ك‬
‫ثِِف‬ َ ‫اس‬
‫ِِفي أ‬َ َّ ‫ع‬
‫ِالن‬ ُ ‫ف‬
َ ‫ماِيَ أن‬ َّ َ‫وأ‬
َ ِ‫ما‬ َ
{ 47 }

Answer: A
196) Bioeffects report claims that obstetrical scanning may be harmful to a particular
group of patients. What should be the response of the medical community?
A. Perform the exams on all patients when the risks outweigh the benefits.
B. Stop all diagnostic exams.
C. Ignore the report
D. Perform exams on all patients when the benefits outweigh the risks.
Answer: D
*Beneficence: benefits out weight risks.
197) 45 years old Woman with uterine fibroid. The doctor prescribed for her
Gonadotropin-releasing hormone (GnRH) agonists. What is the side effect for hcg
hormone?
A. Depression ✅

B. Anorexia
C. Osteoarthritis
D. Menopause
Answer: A
198) The first step in the qualitative research process?
A. Data analysis
B. Review of literature
C. Sample
D. Study design
Answer: B
199) In an article’s abstract, which research process steps may be noted?
A-Literature review
B-Data-collection procedure
C-Research question & study purpose
D-Legal-ethical issues

ِ‫يِاْلَ أرض‬
‫أ‬ ُ ‫َم‬
ُ ‫ك‬
‫ثِِف‬ َ ‫اس‬
‫ِِفي أ‬َ َّ ‫ع‬
‫ِالن‬ ُ ‫ف‬
َ ‫ماِيَ أن‬ َّ َ‫وأ‬
َ ِ‫ما‬ َ
{ 48 }

Answer: C
200) During the research process, when should a hypothesis be developed by the
researcher?
A-Before any statistical analysis
B-After a research design is determined.
C-Before development of the research question
D-After development of the research question
Answer: D
201) All the following are considered steps in the qualitative research process, except?
A-Literature review
B-Data collection
C-Sample
D-Hypothesis
Answer: D
202) The doctor decided DNR for patient. The patient refused that. What should the nurse do?
A-discuss patient about DNR
B-comfort and palliative care ✅
C_ prepare spiritual equipment
D- stop all medication
Answer: Bbb
203) For old man patient with poor prognoses the health care provider decides to put
him DNR, Patient have 3 children 2 of them agree with DNR the 3rd one refused and ask
to continue treatment.
What the nurse should do:
A. provide care and comfort
B. provide palliative care
C. Discuss with family regarding DNR and patient needs✅

Answer: C

204) The discussion section addressing the…


A-Problem
B-Question

ِ‫يِاْلَ أرض‬
‫أ‬ ُ ‫َم‬
ُ ‫ك‬
‫ثِِف‬ َ ‫اس‬
‫ِِفي أ‬َ َّ ‫ع‬
‫ِالن‬ ُ ‫ف‬
َ ‫ماِيَ أن‬ َّ َ‫وأ‬
َ ِ‫ما‬ َ
{ 49 }

C-Objectives
D-All the above
Answer: D
205) The nurse is caring woman that has cancer, and she is under Chemotherapy. She is
complaining anorexia and the patient has low weight. What should the nurse instruct
her???
A. Eat small meals every day.
B. Eat large meals every day.
C. Eat if you are hungry.
D. Eat your favourite food.
Answer: A
206) 56-year-old present to the emergency department experiencing left sides eye
discomfort for the past 3 hours, left eye was blurred while vision in the right eye
remained examination showed increased intra-ocular pressure in the left eye pupil of
the left also reacted slowly to light. which is the most likely health problem?
A. Detached retina
B. Macular hole
C. Glaucoma
Answer: C
207)10 years' boy with polyuria and dysuria after assessment diagnosed with urinary
tract infections what should do to take urine sample?
A. Increase fluid intake
B. Decrease urine intake
C. Regular intake of fluid
D. Zero intake of fluid
Answer: A
208) After assessment of new-born nurse instruct parent to use kangaroo roll which of
the following should be useful
A. Heart rate 150
B. Respiration 55

ِ‫يِاْلَ أرض‬
‫أ‬ ُ ‫َم‬
ُ ‫ك‬
‫ثِِف‬ َ ‫اس‬
‫ِِفي أ‬َ َّ ‫ع‬
‫ِالن‬ ُ ‫ف‬
َ ‫ماِيَ أن‬ َّ َ‫وأ‬
َ ِ‫ما‬ َ
{ 50 }

C. Temperature 34.5
Answer: C
209) Nurse notes some students with anorexia nervosa what does she do??
A. Push them to eat.
B. One to one superior during eating
C. Let students eat with them the same meals.
Answer: C
210) A nurse is teaching a group of women about the side effects of different types of
contraceptives. What common side effect associated with the use of an intrauterine
device (IUD) should the nurse discuss during the teaching session?
A: Tubal pregnancy
B: Rupture of the uterus
C: Expulsion of the device
D: Excessive menstrual flow
Answer: D
211) What does the mechanism of action result in for Inhaled corticosteroids for
asthmatic patient??
A. Reduced inflammatory response and fewer exacerbations, reduced oedema
and mucus. production, Increase responsiveness to β-agonist.
B. Increase broncho dilatation inside lung.
Answer: A
212) The nurse is caring for patient with asthma. What is the appropriate treatment for
asthma?
A. Alpha antagonist, Corticosteroid
B. Beta antagonist, Corticosteroid
C. Alpha agonist, Corticosteroid
D. Beta agonist, Corticosteroid
Answer: D

ِ‫يِاْلَ أرض‬
‫أ‬ ُ ‫َم‬
ُ ‫ك‬
‫ثِِف‬ َ ‫اس‬
‫ِِفي أ‬َ َّ ‫ع‬
‫ِالن‬ ُ ‫ف‬
َ ‫ماِيَ أن‬ َّ َ‫وأ‬
َ ِ‫ما‬ َ
{ 51 }

213)The patient with sign of severe asthma. What is the treatment you will administer
first?
A. Corticosteroids.
B. Bronchodilator.
C. Beta agonist.
Answer: A
214) The nurse is caring for patient with deep vein thrombosis (DVT). The patient’s
heparin sodium infusion has been discontinued and the patient is receiving prescribed
warfarin sodium (Coumadin). The nurse should advise the patient that which of the
following needs to be continued?
A. Daily complete blood count (CBC)
B. Laboratory tests for partial thromboplastin time (PTT)
C. Strict bed rest
D. Wearing elasticized support
stockings.
Answer: C
215) Amenorrhea for 5 weeks and
breast tenderness?
A-Positive sign
B-presumptive sign
Answer: B

ِ‫يِاْلَ أرض‬
‫أ‬ ُ ‫َم‬
ُ ‫ك‬
‫ثِِف‬ َ ‫اس‬
‫ِِفي أ‬َ َّ ‫ع‬
‫ِالن‬ ُ ‫ف‬
َ ‫ماِيَ أن‬ َّ َ‫وأ‬
َ ِ‫ما‬ َ
{ 52 }

216) The nurse is reviewing the health care provider’s prescriptions for a child with a
streptococcal infection and notes that an antistreptolysin O titer is prescribed. Based on
this prescription, which disorder would the nurse suspect in the child?
A- Rheumatic Fever RF
B-Aortic valve diseases
C-pulmonic valve diseases
D-congestive heart failure
Answer: A
217) Patient with Rheumatic fever. what the accurate test for Rheumatic Fever?
A. Ant streptolysin Test
B. Blood cultures
C. Urine culture
Answer: A

218) How to prevent Rheumatic fever?


A. Give vaccines the pregnant women during pregnancy.
B. Isolation the children with tonsillitis
C. Give 9 months vaccination.
D. Treat the children with rheumatic fever antibiotics full course.
Answer: D
219) In determining the one-minute APGAR score of a male infant the nurse assesses a
heart rate of 120 beats per minute and respiratory rate of 44 per minute. He has flaccid
muscle tone with slight flexion and resistance to straightening. He has a loud cry with
colour is acrocyanotic What is the APGAR score for the infant?

ِ‫يِاْلَ أرض‬
‫أ‬ ُ ‫َم‬
ُ ‫ك‬
‫ثِِف‬ َ ‫اس‬
‫ِِفي أ‬َ َّ ‫ع‬
‫ِالن‬ ُ ‫ف‬
َ ‫ماِيَ أن‬ َّ َ‫وأ‬
َ ِ‫ما‬ َ
{ 53 }

A. 7
B. 8
C.9

D.10

Answer: B
220) year-old child was admitted with suspected appendicitis. A nurse was evaluating
the child’s condition and the mother stated that the child did have his bowel
movement for the past two days and requested for natives. What is the risk of giving
laxative to patient with appendicitis?
A. Pain
B. Fever
C. Rupture
D. Diarrhoea
Answer: C
221) What is The term that use to describe people communicate with same culture,
goals, values and ethical?
A. Intrapersonal communication
B. Interpersonal communication✅

C. Intergroup communication
D. Interorganization communication
Answer: B
222) A patient took his NPH insulin at 6 AM, when will he be hypoglycemia?
A-7am
B-8am
C-9am
D-10am
Answer: D
223) A 56-year-old man was admitted with complaint working for three days. The nurse
is preparing to administer infusion of saline 1000 ml over six hours. What is the hourly
infusion rate that the nurse needs to infusion therapy in mL/hour?

ِ‫يِاْلَ أرض‬
‫أ‬ ُ ‫َم‬
ُ ‫ك‬
‫ثِِف‬ َ ‫اس‬
‫ِِفي أ‬َ َّ ‫ع‬
‫ِالن‬ ُ ‫ف‬
َ ‫ماِيَ أن‬ َّ َ‫وأ‬
َ ِ‫ما‬ َ
{ 54 }

155
167
C.190

D.217

Answer: B
224) Suspect patient with TB what is the highest priority nursing action?
A. Isolate the patient in private negative pressure room.
B. Take nasal swab
Answer: A
225) At 6:00 PM while admitting a woman for a scheduled repeat C section1, a patient
tells a nurse that she drank a cup of coffee at 4:00OM because she wanted to avoid
getting a headache. Which of the following actions should the take first?
A. Inform anesthesia care provider.
B. Ensure preoperative lab results are available.
C. Start prescribed IV with lactated ringers.
D. Contact patient's obstetrician
Answer: A
226) A patient is transferred to the Intensive Care following a craniotomy. The Patient is
difficult to arouse, and the pupils are pinpoint and non-reactive. Blood pressure 118/70
mmHg, Heart rate 58/min, Respiratory rate 11/min, Temperature 37.2°C. Which
medication should the nurse prepare administer?
A- Adrenaline
B- Thiamine
C- Naloxone
D- Dextrose 50%
Answer: C
227) Patient with schizophrenia. He complains with anxiety episodes which needs is
highest priority in Maslow hierarchy?
A. Physiological needs C. Self esteem

ِ‫يِاْلَ أرض‬
‫أ‬ ُ ‫َم‬
ُ ‫ك‬
‫ثِِف‬ َ ‫اس‬
‫ِِفي أ‬َ َّ ‫ع‬
‫ِالن‬ ُ ‫ف‬
َ ‫ماِيَ أن‬ َّ َ‫وأ‬
َ ِ‫ما‬ َ
{ 55 }

B. Safety D. actualization
Answer: B
228) How can we prevent infection for preterm neonate?
A. Schedule for immunizations
B. Give influenza and pneumonia vaccine.
C. Wear gloves and gown during the care of neonate
Answer: C
229) A nurse who works in the surgical unit at one of the hospitals was asked by the
home health care nurse to make a home visit to a patient with colostomy, who had
been discharged the previous day in order to give him a follow-up care and education
which of the following nurses should do the assigned task?
A. Critical care nurse
B. Psychiatric nurse
C. Surgical nurse
D. Community nurse
Answer: D
230) A nurse is caring for child who is pot tonsillectomy and adenoidectomy the nurse
should plan to assess which of the following complication?
A. Pulmonary hypertension
B. Hemorrhage
C. Hearing loss
D. Orthopnea
Answer: B
231) The infant after circumcision. What are the non-pharmacology techniques to
relieve pain??
A. Breastfeeding decrease pain.
Answer: A
232) A conscious victim of a motor vehicle accident arrives at the emergency
department. The patient is Gasping for air, is extremely anxious, and has a deviated
trachea. What diagnosis should the nurse Anticipate?

ِ‫يِاْلَ أرض‬
‫أ‬ ُ ‫َم‬
ُ ‫ك‬
‫ثِِف‬ َ ‫اس‬
‫ِِفي أ‬َ َّ ‫ع‬
‫ِالن‬ ُ ‫ف‬
َ ‫ماِيَ أن‬ َّ َ‫وأ‬
َ ِ‫ما‬ َ
{ 56 }

A. pleural effusion.

B. tension pneumothorax✅

C. pneumothorax
D. cardiac tamponed
Answer: B
233) What is the position for adenoidectomy??
A. Semi Fowler
B. Sitting
C. prone
Answer: A
234) A 20 weeks pregnant, primary gravid woman visits the antenatal has sickle cell
anemia trait and worried this disease transmitted to her baby which of the following
should be initial intervention?
A. Plan for the fetal genetic screening
B. Educate mother that her disease is inactive.
C. Discuss the chances of genetic disease in the fetus.
D. Gather data about the other family members having the disease.
Answer: D
235) A 62-year-old women admitted to the emergency department for the fourth time
this year, each time the patient comes with severe injuries and bruises in the body.
What is your responsibility as a nurse to prevent such incident to happen again?
A. Reports the assault to the local police and write a report.
B. Provides information about safe shelter and support.
C. Instructs the women to move away from her home.
D. Discharge the patient to a safe shelter
Answer: A
236) An 82-year-old woman with Alzheimer’s disease had moved into a long-term care
facility two weeks previously. Since then, the staff has found her wondering in the
hallways in middle of the night. When approached, she is confused and frustrated,

ِ‫يِاْلَ أرض‬
‫أ‬ ُ ‫َم‬
ُ ‫ك‬
‫ثِِف‬ َ ‫اس‬
‫ِِفي أ‬َ َّ ‫ع‬
‫ِالن‬ ُ ‫ف‬
َ ‫ماِيَ أن‬ َّ َ‫وأ‬
َ ِ‫ما‬ َ
{ 57 }

often forgetting where she is. Which intervention would most likely decrease the
patient's confusion?
A. Administer a sleeping sedative.
B. Provide full-time nursing care.
C. Place a nightlight in the room
D. Provide a large meal before bed
Answer: C
237) Nurse is completing the preoperative checklist for one of the patients who ring is
wearing a ring. What is the most appropriate action?
A. Give ring to security office.
B. Lock ring with patient's valuables
C. Call patient's family to give them the ring.
D. Respect patient's choice and leave ring on patient's finger.
Answer: B
238) An Indian patient, who is vegetarian, is being discharged from after an elective
surgical procedure. The unit nurse is teaching using visual aids and pictures about food
combinations complete protein. Which of the following food items should the nurse
recommended dietary list of the patient?
A. Lentils B. Potatoes
C. Macaroni D. Green salad
Answer: A

239) which of the following indication for patient with MI: -


A. ST depression
* ST elevation = MI
B. ST elevation
*ST depression = ischemia
C. short ST
D. none of the above
Answer: B

ِ‫يِاْلَ أرض‬
‫أ‬ ُ ‫َم‬
ُ ‫ك‬
‫ثِِف‬ َ ‫اس‬
‫ِِفي أ‬َ َّ ‫ع‬
‫ِالن‬ ُ ‫ف‬
َ ‫ماِيَ أن‬ َّ َ‫وأ‬
َ ِ‫ما‬ َ
{ 58 }

240) The nurse is preparing to transfer a 55-year-old patient on the CT-scan unit. The
patient is too heavy for the nurse to transport, nurse went to get an assistive device to
transfer the patient. Which of the following transfer device is the most appropriate for
the nurse to use?
A. Board B. Handle
C. Trapeze D. Mechanical lift
Answer: D
241) During meningitis outbreak in one of the hospitals, the nun-immune staff were
given an immunoglobulin in order to prevent them from the infection. What is the type
of immunity that will be developed by these staff?
A. active B. long-term
C. natural D. passive
Answer: D
242) patient is being weaned off from the mechanical ventilator is about to hook the
endotracheal tube to oxygen at FiO2 of 40 Which of the following oxygen
administration device is the best the nurse in this situation?
A. Ambo bag
B. Ventura mask
C. Tracheostomy collar
D. T-piece/ Briggs
Answer: D
243) Basal skull fracture signs: Racoon eyes
244)For physical assessment steps for the foot I Start by:
A. Inspection, palpitation
B. Palpation, auscultation
C. Percussion, palpation
Answer: A
245) A 43-year-old man in the post-surgical area complains of abdominal pain radiating
to the naval which is increasing with examination his abdomen is guarded with marked
tender lower quadrant. What is the immediate goal of care to do?
A. Teach abdominal splinting during coughing.

ِ‫يِاْلَ أرض‬
‫أ‬ ُ ‫َم‬
ُ ‫ك‬
‫ثِِف‬ َ ‫اس‬
‫ِِفي أ‬َ َّ ‫ع‬
‫ِالن‬ ُ ‫ف‬
َ ‫ماِيَ أن‬ َّ َ‫وأ‬
َ ِ‫ما‬ َ
{ 59 }

B. Administer pain medication as ordered.


C. Assess pain and report immediately.
D. Position on the left lateral side
Answer: C
246) A 21-year-old in oversized clothing presents to the hospital with of felling dizzy and
faint. The hair and nail appear thin and dry. The skin appears pale, and she has sunken
eye sockets and tenting skin. Her body mass index is 16. She often induces vomiting
after eating blood is collected for analysis (see lab results). *Blood pressure 90/52
mmHg, Heart rate 118 /min, Respiratory rate 26/min, * Temperature 37.2 ͦC, Oxygen
saturation 97%.
ABG Test Result Normal value
HCO₃ 31 22 – 88mmol/L
PCO₂ 10.3 4.7-6.0 kPa
PH 7.50 7.36-7.45
Which nursing problems stem is the most appropriate?
A. Impaired nutrition
B. Decreased cardiac output.
C. Infective airway clearance
D. Ineffective breathing pattern
Answer: B
247) Patient has heart failure and suddenly has edema. What is the lung sound will be
hearing?
A. Rales
B. Rhonchi
C. Wheezing
Answer: A
248) Patient is receiving oxygen therapy by face mask. What is the oxygen saturation
should be achieved?
A. 85% and above
B. 95% and above

ِ‫يِاْلَ أرض‬
‫أ‬ ُ ‫َم‬
ُ ‫ك‬
‫ثِِف‬ َ ‫اس‬
‫ِِفي أ‬َ َّ ‫ع‬
‫ِالن‬ ُ ‫ف‬
َ ‫ماِيَ أن‬ َّ َ‫وأ‬
َ ِ‫ما‬ َ
{ 60 }

Answer: B
249) What is the normal range for oxygen saturation in patient who is receiving oxygen
therapy?
A- 60-69% B- 70-79%
C- 80-89% D- 94- 98%
Answer: D
250) Soldier was brought to triage area after being exposed to chemical weapons.
Signs and symptoms of nerve gas exposure were noticed. A nurse prepares for medical
management. Which medication should the nurse prepare for the patient?
A. Atropine B. Adrenaline
C. Sodium nitrate D. Sodium thiosulphate
Answer: A
251) A patient comes to the emergency unit with cough and severe dyspnea. The
patient’s medical history revealed a diagnosis of chronic heart failure and chronic
obstructive pulmonary disease. Blood pressure 110/70 mmhg Heart rate 87/min
Respiratory rate 23/min Temperature 37.3 C
Which of the following diagnostic tests will be most beneficial to a nurse to figure out if
there is an exacerbation of heart failure?
A. B-type natriuretic peptide (BNP)
B. arterial blood gas (ABG)
C. cardiac enzymes (CK-MB)
D. chest x-ray
Answer: A
252) 20 years old came to ER department in motor accident with one of his relative, all
medical team attempts failed and the PT died, what is the best action to his relative ?
A-Ask the relative sitting down
B-Assess the relative vital signs
C-Let the relative see the PT✅
D-Ask the relative to be patient
Answer: C
253) A patient returned to the Surgical Unit from the thyroidectomy. The nurse observed
that the arousable. Blood pressure 90/60 mmHg Heart rate 108 /min What immediate
action should the nurse take?

ِ‫يِاْلَ أرض‬
‫أ‬ ُ ‫َم‬
ُ ‫ك‬
‫ثِِف‬ َ ‫اس‬
‫ِِفي أ‬َ َّ ‫ع‬
‫ِالن‬ ُ ‫ف‬
َ ‫ماِيَ أن‬ َّ َ‫وأ‬
َ ِ‫ما‬ َ
{ 61 }

A. Recheck pulse and blood pressure.


B. Administer intravenous fluids as ordered.
C. Place client in modified Trendelenburg's
D. Assess the back of neck surgical dressing for bleeding.
Answer: D
254) new-born has small, whitish, pinpoint spots over the nose, which the nurse knows
are caused by retained sebaceous secretions. When charting this observation, the
nurse identifies it as:
A. Milia
B. Lanugo
C. Whiteheads
D. Mongolian spots
Answer: A
255) A nurse is caring for a client with syndrome of inappropriate antidiuretic hormone
(SIDH). Which of the following should be a priority intervention for the client?
A. Monitoring hourly intake and output
B. Pressure ulcer prevention strategies
C. Encourage client to eat foods rich in potassium.
D. Restricting fluid intake of the client to less than 1000ml per day
Answer: D
256) A postdate pregnant woman is admitted for the induction of labour. Her fetal
heart rate and vital signs are within normal range her intravenous line is maintained and
she is to be started on low doses of labour inducing medication. Which of the following
medication the mother is likely to receive intravenously?
A. Oxytocin
B. Cervidil
C. Cytotec
D. Cytoxan
Answer: A

ِ‫يِاْلَ أرض‬
‫أ‬ ُ ‫َم‬
ُ ‫ك‬
‫ثِِف‬ َ ‫اس‬
‫ِِفي أ‬َ َّ ‫ع‬
‫ِالن‬ ُ ‫ف‬
َ ‫ماِيَ أن‬ َّ َ‫وأ‬
َ ِ‫ما‬ َ
{ 62 }

257) A 40-year-old man was admitted to a male medical department with pneumonia.
A nurse in the department explained and obtained informed consent for a chest CT
scan procedure with contrast. with of the following ethical principles underpinning
informed consent.?
A. Autonomy
B. Non maleficence
C. Beneficence
D. Respect
Answer: A
258) While a nurse is assessing vital signs of new-born infant first hour of delivery. HR 170
RR 70 TEM 36 the nurse would interpret these finding as in the discharge instruction.
A. Anaemia B. Cold distress
C. Heart defects D. Hyperglycemia
Answer: B
259) Which of the following statements describe the function of Immunoglobulin (IgG)?
A. It does not cross the placenta and is present in colostrum and breast milk.
B. It protects against gastrointestinal and respiratory system infections.
C. It provides temporary immunity to bacteria and toxins to which the mother has
developed immunity
D. It protects against gram negative bacteria and can be found cord blood.
Answer: C
260) Fertilization occurs in the fallopian tube, egg and sperm united to form which
moves towards the uterus in a long journey to implant in uterus, how long the zygote
takes to implant in the uterus?
A )3-8 day
B) 3-7
C) 7-10
D) 10-14
Answer: C

ِ‫يِاْلَ أرض‬
‫أ‬ ُ ‫َم‬
ُ ‫ك‬
‫ثِِف‬ َ ‫اس‬
‫ِِفي أ‬َ َّ ‫ع‬
‫ِالن‬ ُ ‫ف‬
َ ‫ماِيَ أن‬ َّ َ‫وأ‬
َ ِ‫ما‬ َ
{ 63 }

261) A lot of theories took about early skin contact in 3rd stage of labour what is the
purpose from early skin to skin contact between mother and child?
A-at warm baby
B- early breast feeding
C- improve uterine contraction.
D-decrease maternal pulse rate
Answer: B
262) 22 years old girl went her that voice is bad she in music institute which defends
mechanism she used?
A) Denial B) displacement
C) projection D) repression
Answer: C
263) Patient immobile 3days science total hip replacement. she will ambulate for the
first time after surgery the nurse told her to descent her leg and sit at the side of the bed
before bearing her weight in foot. The nurse instructs her to do this to avoid which of the
following?
A- hypotension
B- dislocation
C-hypertension
D-headache
Answer: A
264) Pt when entered ER he said, my heart will get out of my chest I fell that I will die he
diagnosed with panic attack what is the medical problem that will be developed if
panic not controlled?
A) Respiratory acidosis
B) Respiratory alkalosis
C) Metabolic acidosis
D) Metabolic alkalosis
Answer: B
265) If You lose an ampule of morphine what you will do?

ِ‫يِاْلَ أرض‬
‫أ‬ ُ ‫َم‬
ُ ‫ك‬
‫ثِِف‬ َ ‫اس‬
‫ِِفي أ‬َ َّ ‫ع‬
‫ِالن‬ ُ ‫ف‬
َ ‫ماِيَ أن‬ َّ َ‫وأ‬
َ ِ‫ما‬ َ
{ 64 }

A) write an incident report


B) call the nurse manager and told her to deal with this situation
C) call the primary nurse who was in the previous shift
D) counselling the nurse who count and sign
Answer: B
266) Nurses meet another nurse college after failed in resuscitation of child, she was
crying and depressed. What is the suitable response in this situation?
A) you need to take vacation some days until you feel better
B) just return to your home and hugging your children
C)there was nothing in your hand to do.
D) let her crying and talk about the matter
Answer: D
267) A gravida 3 para 2 presents to the Maternity Triage Unit after the amniotic
membranes ruptured at home. The fluid is noted to be clear. The neonates head is
engaged into the pelvis and the patient is having contractions every 5 to 7 minutes.
Each contraction lasts for 60-90 seconds. An examination of the cervix finds 4
centimeters dilatation and 90% effacement. She is. uncomfortable during contractions
and rates the pain at a level 7, on pain scale of 1-10.
Which of the following is most indicative that she is in true labour?
A. Level of pain
B. Cervical dilatation and effacement
C. Engagement of presenting part
D. Frequency and length of contractions
Answer: B
268) The nurse is teaching 32-week pregnant women how to distinguish between pre
labor (false) contraction and true labor contractions. Which statement about pre labor
contractions accurate?
A. they are regular and increase gradually.
B. they are felt in the abdomen.
C. they start at the back and radiate to the abdomen.

ِ‫يِاْلَ أرض‬
‫أ‬ ُ ‫َم‬
ُ ‫ك‬
‫ثِِف‬ َ ‫اس‬
‫ِِفي أ‬َ َّ ‫ع‬
‫ِالن‬ ُ ‫ف‬
َ ‫ماِيَ أن‬ َّ َ‫وأ‬
َ ِ‫ما‬ َ
{ 65 }

D. they become more intense during walking.


Answer: B
269) A patient was complaining of depression, muscle weakness and continuous
fatigue. which of the following
deficiencies is suspected?
A. Deficiency of vitamin B.
B. Deficiency of vitamin D.
C. Deficiency of vitamin E
D. Deficiency of vitamin K
Answer: B
270) The nurse is assessing a child who has Tetralogy of Fallot observed that the child is
having clubbing in his fingernails Which of the flowing might be the reason for this
clubbing?
A. Prolonged tissue hypoxia
B. Delayed physical growth.
C. Inactive bone marrow
D. Pulmonary fibrosis
Answer: A
271) Which of the following related to Low pitched breath sound??
A. Crackles
B. Rhonchi
C. Wheezing
D. Vascular
Answer: B
272) Which of the following related to High pitched breath sound??
A. Crackle
B. Rhonchi
C. Wheezing

ِ‫يِاْلَ أرض‬
‫أ‬ ُ ‫َم‬
ُ ‫ك‬
‫ثِِف‬ َ ‫اس‬
‫ِِفي أ‬َ َّ ‫ع‬
‫ِالن‬ ُ ‫ف‬
َ ‫ماِيَ أن‬ َّ َ‫وأ‬
َ ِ‫ما‬ َ
{ 66 }

D. Bronchial
Answer: C
273) Which of the following related to harsh breath sound??
A. Crackles
B. Bronchial
C. Wheezing
D. Stridor
Answer: D
274) Which of the following related to harsh breath sound??
A. Crackles
B. Tracheal
C. Wheezing
D. Rhonchi
Answer: B
275) A newly graduated nurse is inserting an intravenous cannula into the mid-cephalic
vein of a patient who is being admitted to the unit. As she withdraws the needle, a
nurse calls out for help from another patient’s room. The new nurse rushes to help. She
secured the intravenous catheter and threw the needle into the waste basket instead
of the sharp’s container. When asked who had thrown the needle into the bin, the new
nurse admits that she had made that mistake. Which professional act best describes
the newly graduated nurse’s response?
A. Responsibility
B. Accountability
C. Assertiveness
D. Leadership
Answer: B
276) He wants to influence the customary way of thinking and behaving that is shared
by the members of the department. Which of the following terms refer to this?
A. Organizational chart
B. Cultural network

ِ‫يِاْلَ أرض‬
‫أ‬ ُ ‫َم‬
ُ ‫ك‬
‫ثِِف‬ َ ‫اس‬
‫ِِفي أ‬َ َّ ‫ع‬
‫ِالن‬ ُ ‫ف‬
َ ‫ماِيَ أن‬ َّ َ‫وأ‬
َ ِ‫ما‬ َ
{ 67 }

C. Organizational structure
D. Organizational culture
Answer: D
277) After a code blue, a doctor announced the death of a patient. A signature
other begins to show signs of decreased level of consciousness. What is the priority
intervention of the nurse at this time?

A. Offer a meal

B. Ensure safety ✅

C. Set up an IV line

D. Offer a glass of water

Answer: B
278) Stephanie is a new Staff Educator of a private tertiary hospital. She conducts
orientation among new staff nurses in her department. Joseph, one of the new staff
nurses, wants to understand the channel of communication, span of control and lines of
communication. Which of the following will provide this information?
A. Organizational structure
B. Policy
C. Job description
D. Manual of procedures
Answer: A
279) Stephanie is often seen interacting with the medical intern during coffee breaks
and after duty hours. What type of organizational structure is this?
A. Formal
B. Informal
C. Staff
D. Line
Answer: B
280) She takes pride in saying that the hospital has a decentralized structure. Which of
the following is NOT compatible with this type of model?

ِ‫يِاْلَ أرض‬
‫أ‬ ُ ‫َم‬
ُ ‫ك‬
‫ثِِف‬ َ ‫اس‬
‫ِِفي أ‬َ َّ ‫ع‬
‫ِالن‬ ُ ‫ف‬
َ ‫ماِيَ أن‬ َّ َ‫وأ‬
َ ِ‫ما‬ َ
{ 68 }

A. Flat organization
B. Participatory approach
C. Shared governance
D. Tall organization
Answer: D
281) you find a victim arrested in the street according to car accident you found a
clear fluid dropping from his ear, and you must give him rescues breathing, what you
will do?
A. chest thrust
B. jaw thrust ✅
C- Head tilt chin lift
Answer: B
282) Patient has Colostomy we transfer the Pt:
A-Belt abdomen
B- Belt chest ✅

C- Restrain
Answer: B

283) A 4-month-old infant returned immediately from OR room post cleft lip repair which
of the following nursing
intervention should be considered.?
A. Apply elbow restrain ✅
B. Apply suction when needed
C. Measure temperature
D. Put infant in prone position
Answer: A

284) Which of the following guidelines should be least considered in formulating


objectives for nursing care?

ِ‫يِاْلَ أرض‬
‫أ‬ ُ ‫َم‬
ُ ‫ك‬
‫ثِِف‬ َ ‫اس‬
‫ِِفي أ‬َ َّ ‫ع‬
‫ِالن‬ ُ ‫ف‬
َ ‫ماِيَ أن‬ َّ َ‫وأ‬
َ ِ‫ما‬ َ
{ 69 }

A. Written nursing care plan


B. Holistic approach
C. Prescribed standards
D. Staff preferences
Answer: D
285) Stephanie considers shifting to transformational leadership. Which of the following
statements best describes this type of leadership?
A. Uses visioning as the essence of leadership.
B. Serves the followers rather than being served.
C. Maintains full trust and confidence in the subordinates.
D. Possesses innate charisma that makes others feel good in his presence.
Answer: A
286) As a manager, she focuses her energy on both the quality of services rendered to
the patients as well as the welfare of the staff of her unit. Which of the following
management styles does she adopt?
A. Country club management
B. Organization man management
C. Team management
D. Authority-obedience management
Answer: C
287) The statement, “The Holy Spirit Medical Centre aims to provide patient-centered
care in a total healing environment” refers to which of the following?
A. Vision
B. Goal
C. Philosophy
D. Mission
Answer: B
288) Joey plans to revisit the organizational chart of the department. He plans to
create a new position of a Patient Educator who has a coordinating relationship with

ِ‫يِاْلَ أرض‬
‫أ‬ ُ ‫َم‬
ُ ‫ك‬
‫ثِِف‬ َ ‫اس‬
‫ِِفي أ‬َ َّ ‫ع‬
‫ِالن‬ ُ ‫ف‬
َ ‫ماِيَ أن‬ َّ َ‫وأ‬
َ ِ‫ما‬ َ
{ 70 }

the head nurse in the unit. Which of the following will likely depict this organizational
relationship?
A. Box
B. Solid line
C. Broken line (Dotted)
D. Matrix
Answer: C
289) He likewise stresses the need for all the employees to follow orders and instructions
from him and not from anyone else. Which of the following principles does he refer to?
A. Scalar chain
B. Discipline
C. Unity of command
D. Order
Answer: C
290) Joey orients his staff on the patterns of reporting relationship throughout the
organization. Which of the following principles refer to this?
A. Span of control
B. Hierarchy
C. Esprit d’ corps
D. Unity of direction
Answer: B
291) He emphasizes to the team that they need to put their efforts together towards
the attainment of the goals of the program. Which of the following principles refers to
this?
A. Span of control
B. Unity of direction
C. Unity of command
D. Command responsibility
Answer: B

ِ‫يِاْلَ أرض‬
‫أ‬ ُ ‫َم‬
ُ ‫ك‬
‫ثِِف‬ َ ‫اس‬
‫ِِفي أ‬َ َّ ‫ع‬
‫ِالن‬ ُ ‫ف‬
َ ‫ماِيَ أن‬ َّ َ‫وأ‬
َ ِ‫ما‬ َ
{ 71 }

292)What is drug that prevent recurrence of rheumatic fever?

A-Penicillin ✅

B- Corticosteroids
C. Salicylates
Answer: A
293) He discusses the goal of the department. Which of the following statements is a
goal?
A. Increase the patient satisfaction rate.
B. Eliminate the incidence of delayed administration of medications.
C. Establish rapport with patients.
D. Reduce response time to two minutes.
Answer: A
294) Ms. Caputo is newly promoted to a patient care manager position. She updates
her knowledge on the theories in management and leadership in order to become
effective in her new role. She learns that some managers have low concern for services
and high concern for staff. Which style of management refers to this?
A. Organization Man
B. Impoverished Management
C. Country Club Management
D. Team Management
Answer: C
295) Her former manager demonstrated passion for serving her staff rather than being
served. She takes time to listen, prefers to be a teacher first before being a leader,
which is characteristic of
A. Transformational leader
B. Transactional leader
C. Servant leader
D. Charismatic leader
Answer: C

ِ‫يِاْلَ أرض‬
‫أ‬ ُ ‫َم‬
ُ ‫ك‬
‫ثِِف‬ َ ‫اس‬
‫ِِفي أ‬َ َّ ‫ع‬
‫ِالن‬ ُ ‫ف‬
َ ‫ماِيَ أن‬ َّ َ‫وأ‬
َ ِ‫ما‬ َ
{ 72 }

296) She reads about Path Goal theory. Which of the following behaviours is
manifested by the leader who uses this theory?
A. Recognizes staff for going beyond expectations by giving them citations.
B. Challenges the staff to take individual accountability for their own practice.
C. Admonishes staff for being laggards.
D. Reminds staff about the sanctions for non-performance.
Answer: A
297) One leadership theory state that “leaders are born and not made,” which refers
to which of the following theories?
A. Trait
B. Charismatic
C. Great Man
D. Situational
Answer: C
298) She came across a theory which states that the leadership style is effective
dependent on the situation. Which of the following styles best fits a situation when the
followers are self-directed, experts and are matured individuals?
A. Democratic
B. Authoritarian
C. Laissez faire
D. Bureaucratic
Answer: C
299) The nurse was giving sufficient information for parent regarding health for their
baby. What is this considering??
A. Positive denial
B. Detachment
C. Empowerment✅

Answer: C

ِ‫يِاْلَ أرض‬
‫أ‬ ُ ‫َم‬
ُ ‫ك‬
‫ثِِف‬ َ ‫اس‬
‫ِِفي أ‬َ َّ ‫ع‬
‫ِالن‬ ُ ‫ف‬
َ ‫ماِيَ أن‬ َّ َ‫وأ‬
َ ِ‫ما‬ َ
{ 73 }

300) A nurse manager in medical unit is not satisfied with the way things in her unit.
Patient satisfaction rate seductive months and staff morale is at its lowest. He decides to
plan and initiate changes that will push for a turnaround in the condition of the unit.
Which of the following actions is a priority for Henry?
A. Call for a staff meeting and take this up in the agenda.
B. Seek help from her manager.
C. Develop a strategic action on how to deal with these concerns.
D. Ignore the issues since these will be resolved naturally.
Answer: A
301) He knows that there are external forces that influence changes in his unit. Which
of the following is NOT an external force?
A. Memo from the CEO to cut down on electrical consumption.
B. Demands of the labour sector to increase wages.
C. Low morale of staff in his unit
D. Exacting regulatory and accreditation standards
Answer: C
302) After discussing the possible effects of the low patient satisfaction rate, the staff
started to list down possible strategies to solve the problems head-on. Should they
decide to vote on the best change strategy, which of the following strategies is referred
to this?
A. Collaboration
B. Majority rule
C. Dominance
D. Compromise
Answer: B
303) One staff suggests that they review the pattern of nursing care that they are using,
which is described as a:
A. job description
B. system used to deliver care.
C. manual of procedure

ِ‫يِاْلَ أرض‬
‫أ‬ ُ ‫َم‬
ُ ‫ك‬
‫ثِِف‬ َ ‫اس‬
‫ِِفي أ‬َ َّ ‫ع‬
‫ِالن‬ ُ ‫ف‬
َ ‫ماِيَ أن‬ َّ َ‫وأ‬
َ ِ‫ما‬ َ
{ 74 }

D. rules to be followed.
Answer: B
304) Which of the following is TRUE about functional nursing?
A. Concentrates on tasks and activities
B. Emphasizes use of group collaboration.
C. One-to-one nurse-patient ratio
D. Provides continuous, coordinated, and comprehensive nursing services.
Answer: A
305) Elderly non-Saudi patient has COPD Stay in KSA. He admitted hospital for caring
and treatment, but he needed to stay long time to receive health services. The doctor
informed his family about that. They decided to discharge the patient because they
cannot pay hospital cost period. It was awfully expensive. What is the most common
cause that force the family to take this decision specially with elderly patient?
A. Because the patient is elderly he will die and no need to pay too much money
B. There is no private health care facility for geriatric patient with less expensive.
C. There are too many geriatric patients over 80 years old in hospitals
D. There is no hospital focus more on long term care for Non Saudi patients
Answer: B
306) He raised the issue on giving priority to patient needs. Which of the following offers
the best way for setting priority?
A. Assessing nursing needs and problems.
B. Giving instructions on how nursing care needs are to be met.
C. Controlling and evaluating the delivery of nursing care.
D. Assigning safe nurse: patient ratio
Answer: A
307) Which of the following is the best guarantee that the patient’s priority needs are
met?
A. Checking with the relative of the patient
B. Preparing a nursing care plan in collaboration with the patient.

ِ‫يِاْلَ أرض‬
‫أ‬ ُ ‫َم‬
ُ ‫ك‬
‫ثِِف‬ َ ‫اس‬
‫ِِفي أ‬َ َّ ‫ع‬
‫ِالن‬ ُ ‫ف‬
َ ‫ماِيَ أن‬ َّ َ‫وأ‬
َ ِ‫ما‬ َ
{ 75 }

C. Consulting with the physician


D. Coordinating with other members of the team
Answer: B
308) When Henry uses team nursing as a care delivery system, he and his team need to
assess the priority of care for a group of patients, which of the following should be a
priority?
A. Each patient as listed on the worksheet.
B. Patients who need least care
C. Medications and treatments required for all patients.
D. Patients who need the most care.
Answer: D
309) He is hopeful that his unit will make a big turnaround in the succeeding months.
Which of the following actions of Henry demonstrates that he has reached the third
stage of change?
A. Wonders why things are not what it used to be.
B. Finds solutions to the problems
C. Integrate the solutions to his day-to-day activities.
D. Selects the best change strategy.
Answer: C
310) Saudi Arabia has noticed the spread of a disease osteoporosis among people,
low attitude towards the problem and lack of commitment of people, what do we do ?
A. Apply strict decisions
B. Provide educational program for population
C. Cure all affected
Answer: B

311) The nurse is performing an admission assessment for a client who has
schizophrenia. The nurse notices that the client's appearance is unkempt, and he
appears to be actively hallucinating. Which of the following should be the nurse's
priority assessment?
A. Perception of reality
B. Ability to follow directions.

ِ‫يِاْلَ أرض‬
‫أ‬ ُ ‫َم‬
ُ ‫ك‬
‫ثِِف‬ َ ‫اس‬
‫ِِفي أ‬َ َّ ‫ع‬
‫ِالن‬ ُ ‫ف‬
َ ‫ماِيَ أن‬ َّ َ‫وأ‬
َ ِ‫ما‬ َ
{ 76 }

C. Physical needs
D. Mental status
Answer: C
312) A child has an abdominal pain and Hepatomegaly, what should the nurse
expect?
A- Hepatitis
B- Cancer
C- Liver damage
D- Low level of hemoglobin
Answer: A
313) Nurse A want to take a break and delegate her task to Nurse B, during that time
the patient falling. As a nurse, who is responsible and be accountable for fall.
A) Head nurse
B) Nurse A
C) Nurse B
D) Nurse A and B
Answer: D
314) which of the following the role of informatic nurse?
A) software
B) design system
Answer: B
315) Community Nursing nurse is planning a follow-up visit to a family after their firstborn
18) child died from Sudden Infant Death-Syndrome (SIDS). Which action is the most
important for the nurse to include in the initial visit??
A. Help the family in planning for future children.
B. Make a referral for genetic counselling and education.
C. Allow time for the parents to express their anger an
D. Educate the family on the causes of SIDS.
Answer: C

ِ‫يِاْلَ أرض‬
‫أ‬ ُ ‫َم‬
ُ ‫ك‬
‫ثِِف‬ َ ‫اس‬
‫ِِفي أ‬َ َّ ‫ع‬
‫ِالن‬ ُ ‫ف‬
َ ‫ماِيَ أن‬ َّ َ‫وأ‬
َ ِ‫ما‬ َ
{ 77 }

316) After the pericardiocentesis, the doctor inserted a polyethylene catheter left it in
the pericardial sac. The patient asked the nurse about the response of the catheter.
What should be the nurse's response?
A. Monitor consistency of drainage
B. Prevent movement of pericardial sac.
C. Prevent recurrence of cardiac tamponed.
D. Prevent increase in venous and blood pressure.
Answer: A
317) After tonsillectomy, a child begins to vomit bright red blood. the initial nursing
action is to?
A. Notify the physician
B. Turn the child to the side.
C. Maintain an NPO status
D. Administer the prescribed antiemetic.
Answer: B
318) A 17-year-old arrived at the emergency room complaining abdominal pain on
right lower quadrant. Pain was rated as 9 numeric scale with positive rebound
tenderness over the pain.
Blood pressure 120/70
Heart rate 100
Respiratory rate 22
Which of the following interventions has the highest priority?
A. keep NPO.
B. secure an IV access.
C. prepares for ultrasound.
D. prepares for abdominal surgery.
Answer: A
319) A 32-year-old prim pare attended the postnatal clinic 4days post – she says she is
keen to breastfeed but the baby to the painful. The nurse examined the breasts and

ِ‫يِاْلَ أرض‬
‫أ‬ ُ ‫َم‬
ُ ‫ك‬
‫ثِِف‬ َ ‫اس‬
‫ِِفي أ‬َ َّ ‫ع‬
‫ِالن‬ ُ ‫ف‬
َ ‫ماِيَ أن‬ َّ َ‫وأ‬
َ ِ‫ما‬ َ
{ 78 }

found that the red and cracked. Which will be nurse advice to her to help the women
situation?
A. apply antibiotic noodle cream to prevent infection.
B. use correct positioning of the infant to latch on nipple.
C. use the same position when feeding not to confuse the
D. use breast pads with plastic lining to prevent leaking of
Answer: B
320) Patient diagnosed with nephrotic syndrome his lab results showed that he had
protein in urinalysis Which of the following medication should give.
A. Immune spirits
B. Cortisone
C. Diuretic
Answer: B
321) How do I collect sputum? Collect your sputum in the early morning
322) Bacteria: The bacteria Streptococcus pneumoniae and Neisseria meningitides are
responsible for 80% of cases of meningitis in adults
323) Vitamin k: Intramuscular injection 0.5-1mg IM at birth (preferred route)
324. A nurse prepares to administer a vitamin K injection to a full term the mother wants
to know the
importance of the injection Which of the following is the best nurse response to the
mother
A. needed for blood clotting to prevent hemorrhage ✅

B. accelerate the growth and development of infants


C. help in maintain healthy gut and passage of meconium
D. protect the infant from developing sever respiratory distress
Answer: A
325) SIDH\ Nursing management is: Fluid restriction
326) A 24-year-old woman diagnosed as having hemolytic anent physician ordered to
transfuse one unit of packet red blood transfusion, the patient start flushing and
complained of dyspnea, generalized body itching (see lab results)

ِ‫يِاْلَ أرض‬
‫أ‬ ُ ‫َم‬
ُ ‫ك‬
‫ثِِف‬ َ ‫اس‬
‫ِِفي أ‬َ َّ ‫ع‬
‫ِالن‬ ُ ‫ف‬
َ ‫ماِيَ أن‬ َّ َ‫وأ‬
َ ِ‫ما‬ َ
{ 79 }

TEST Result Normal values


Hb 130 142-176 g/L
WBC 4.5 4-10.5 x10^2
RBC 3.0 3.8-5.1 x 10^2 /L
Which of the following nursing diagnosis is related to blood an occludes the patient’s
problem?
A. Bacteremia
B. Fluid overload
C. Hypovolemic shock
D. Transfusion reaction
Answer: D
327) A post-operative patient who underwent an abdominal procedure requests
medication from the nurse and rates the pain at a level nine. They are standing order
for narcotic administration. When the nurse opera the narcotic box and preforms a
count, the number of pills recorded on the sheet. What is the most appropriate initial
nursing action?
A. Notify the nursing supervisor.
B. write the finding on the narcotic sheet.
C. Administer the patient requested medication.
D. Identify the last nurse who used the narcotic box.
Answer: A
328) A mother of 8 months' child wants to give him an egg to eat what is the kind of
egg she should give?
A. Egg yolk
B. Egg whites (> 1yrs)
Answer: A
329) What are the 3 stages of Separation Anxiety in order?
A. Protest, despair, detachment.
B. Despair, protest detachment

ِ‫يِاْلَ أرض‬
‫أ‬ ُ ‫َم‬
ُ ‫ك‬
‫ثِِف‬ َ ‫اس‬
‫ِِفي أ‬َ َّ ‫ع‬
‫ِالن‬ ُ ‫ف‬
َ ‫ماِيَ أن‬ َّ َ‫وأ‬
َ ِ‫ما‬ َ
{ 80 }

Answer: A
330) Patient will start TPN and should started gradually to avoid which:
A-hyperinsulinemia B-hyperinsulinemia
C-hypoglycemia D-hyperglycemia
Answer: D
331) A nurse has an order to transfuse a unit of packed red blood cells to a client who
does not currently have an intravenous (IV) line inserted. When obtaining supplies to
start the IV infusion, the nurse selects an Angio catheter with a size of:
A. 18-gauge
B. 21 gauge
C. 22-gauge
D. 24 gauge
Answer: A
332) A nurse develops a plan of care for a child at risk for tonic-colonic seizures. In the
plan of care, the nurse identifies seizure precautions and documents that which item(s)
need to be placed at the child's bedside?
A. Emergency cart
B. Tracheotomy set
C. Padded tongue blade
D. Suctioning equipment and oxygen
Answer: D
333) A client with myocardial infarction suddenly becomes tachycardic, shows signs of
air hunger, and begins coughing frothy, pink-tinged sputum. Which of the following
would the nurse anticipate when auscultating the client's breath sounds?
A. Stridor
B. Crackles
C. Scattered rhonchi
D. Diminished breath sounds
Answer: B

ِ‫يِاْلَ أرض‬
‫أ‬ ُ ‫َم‬
ُ ‫ك‬
‫ثِِف‬ َ ‫اس‬
‫ِِفي أ‬َ َّ ‫ع‬
‫ِالن‬ ُ ‫ف‬
َ ‫ماِيَ أن‬ َّ َ‫وأ‬
َ ِ‫ما‬ َ
{ 81 }

334) The physician prescribes captopril (Capo ten) 25mg po tid for hypertension. Which
of the following adverse reactions can o Capo ten?
A. Tinnitus
B. Persistent cough
C. Muscle weakness
D. Diarrhea
Answer: B
335) The nurse wants to assess growth of baby 9 month what is the most appropriate
measure?
A. Weight
B. Height
C. Development
Answer: A
336) The nurse wants to assess nutritional status of baby 9 month what is the most
appropriate measure?
A. Head circumference
B. Arm circumference
C. Chest circumference
Answer: B
337) The supervisor observes a new graduate nurse suctioning a client. Which of the
following techniques requires an intervention?
A. Suction is applied when the catheter is withdrawn.
B. Suction is applied when the catheter is inserted.
C. Suction is applied for 10 seconds.
D. Suction is applied while rotating the catheter 360 degrees.
Answer: A
338) Which of these following indicates signs of severe COPD?
A. High p02 and high pC02
B. Low p02 and low pC02

ِ‫يِاْلَ أرض‬
‫أ‬ ُ ‫َم‬
ُ ‫ك‬
‫ثِِف‬ َ ‫اس‬
‫ِِفي أ‬َ َّ ‫ع‬
‫ِالن‬ ُ ‫ف‬
َ ‫ماِيَ أن‬ َّ َ‫وأ‬
َ ِ‫ما‬ َ
{ 82 }

C. Low p02 and high pC02


D. High p02 and low pC02
Answer: C
339) Multiple myeloma is a neoplastic proliferation of:
A- lymphocytes
B- Granulocytes
C- Plasma cells
D- Monocytes
Answer: C
340) How much of blood volume for patient has a 10% blood loss and his weight is 50kg
A- 300ml
B- 400ml
Formula of Blood Transfusion = 10 ml/ kg
C- 3000ml
D- 4000ml
Answer: B
341) Community health nurse in any country should know the statistical morbidity and
mortality rates for the most common disease for 4th data about national and
international levels. What is the main reason for this need?
A. Identify patient’s needs accordingly.
B. Offer a few accessible services to the people.
C. Compare the prevalence of disease in the communities.
D. Publicize previous health problems and suggest appropriate action.
Answer: D
342) Nursing staff If they take Saudi commission for health specialties. Which is the
Ministry is responsible for that ?
A. Ministry of Health✅
B. Saudi nursing Authority
Answer: A

ِ‫يِاْلَ أرض‬
‫أ‬ ُ ‫َم‬
ُ ‫ك‬
‫ثِِف‬ َ ‫اس‬
‫ِِفي أ‬َ َّ ‫ع‬
‫ِالن‬ ُ ‫ف‬
َ ‫ماِيَ أن‬ َّ َ‫وأ‬
َ ِ‫ما‬ َ
{ 83 }

343) Head nurse of an Intensive Care Unit oriented a new staff nurse about the
hospital's flat organizational structure. Which of the following expectations will the nurse
have about this type of organization?
A- Hierarchy of authority
B- Limit authority for workers
C- Low level of formalization
D- Centralized decision making
Answer: C
344) When developing a plan care for a hospitalized child, nurse Mica knows that
children in which age group are most likely to view illness as a punishment for
misdeeds?
A. Infancy
B. Preschool age
C. School age
D. Adolescence
Answer: B

345) Which hormone that detect the gender of the fetus??


A. Progesterone
B. testosterone
C. Genetics
D. Estrogenic
Answer: B
346) A nurse is starting the morning shift and has received the patient’s handover from
night shift nurses and completed the morning rounds. The nurse is preparing to perform
physical assessment for the patients Which of the following patients should be assessed
first?
A. A postoperative patient with total hip replacement
B. A patient with a stroke who is on nasogastric tube.
C. A patient with congestive heart failure

ِ‫يِاْلَ أرض‬
‫أ‬ ُ ‫َم‬
ُ ‫ك‬
‫ثِِف‬ َ ‫اس‬
‫ِِفي أ‬َ َّ ‫ع‬
‫ِالن‬ ُ ‫ف‬
َ ‫ماِيَ أن‬ َّ َ‫وأ‬
َ ِ‫ما‬ َ
{ 84 }

D. A patient scheduled for appendectomy who was admitted.


Answer: A
347) Rayed is provided new wheelchair program needs. Which of the following Priority
should include in the program??
A. Elderly
B. Psychiatric
C. Pre-eclampsia
D. Measles
Answer: A
348) Rayed is provided framework wheelchair program needs. Which of the following
Priority should include in the program??
A. School age
B. Psychiatric
C. Pre-eclampsia
D. Measles
Answer: C
349) Which of the following is consider the type of leadership in X theory?
A. Authoritaria
B. Autocratic
C. Democratic
Answer: B
350) Which of the following is consider the type of leadership in Y theory?
A. Authoritarian
B. Autocratic
C. Democratic
Answer: C

ِ‫يِاْلَ أرض‬
‫أ‬ ُ ‫َم‬
ُ ‫ك‬
‫ثِِف‬ َ ‫اس‬
‫ِِفي أ‬َ َّ ‫ع‬
‫ِالن‬ ُ ‫ف‬
َ ‫ماِيَ أن‬ َّ َ‫وأ‬
َ ِ‫ما‬ َ
{ 85 }

351) Which of the following is consider the type of leadership of HR (Human


Relationship)?
A. Authoritarian
B. Autocratic
C. Democratic
Answer: A
352) The emotions that is happened during pregnancy and allow pregnant to cry
without any cause it is called which of the following??
A. Sadness
B. Mood swing
C. Ambivalence
Answer: B
353) When the employees assess the performance of their managers. They give positive
feedback for them. What is the type of the evaluation?
A. Supervisor evaluation
B. Subordinate evaluation
C. Peer evaluation
D. Team evaluation
Answer: B
354) What is the most common Complication of NGT??
A. Hemorrhage
B. Aspiration
C. Infection
Answer: B
355) What is rational of insertion therapeutic NGT of pt. in the hospital?
A. lavage
B. decompression
C. juice suction

ِ‫يِاْلَ أرض‬
‫أ‬ ُ ‫َم‬
ُ ‫ك‬
‫ثِِف‬ َ ‫اس‬
‫ِِفي أ‬َ َّ ‫ع‬
‫ِالن‬ ُ ‫ف‬
َ ‫ماِيَ أن‬ َّ َ‫وأ‬
َ ِ‫ما‬ َ
{ 86 }

Answer: A
356)What is the goal for NGT insertion after cholecystectomy?
A. Prevent aspiration
B. Compress stomach
C. Suction gastric content
Answer: B
357) PKU screen test for what?
A-blood disorders
B-glucose
C-metabolism disorders
Answer: C
358) What is the authority that issues licenses for workers in the health sector in Saudi
Arabia?
A. Saudi commission for health specialties (SCFHS)
B. Saudi hospitals
Answer: A
359) Saudi Arabia neonatal screening for?

A. Hypothyroidism diseases✅

B. Congenital diseases
C. Immunodeficiency diseases
Answer: A
360) How take Urine sample from patient with catheter?
A-with syringe
B-clamp for 30 minutes then take it
C-Early morning
Answer: B
361) Subinvolution medication?
A-methergine

ِ‫يِاْلَ أرض‬
‫أ‬ ُ ‫َم‬
ُ ‫ك‬
‫ثِِف‬ َ ‫اس‬
‫ِِفي أ‬َ َّ ‫ع‬
‫ِالن‬ ُ ‫ف‬
َ ‫ماِيَ أن‬ َّ َ‫وأ‬
َ ِ‫ما‬ َ
{ 87 }

B-oxytocin
C-tramadol iv
Answer: A

362) The pregnant 24 week came to antenatal clinic with spot of blood from vagina
for 3 days. She was worried that Maybe she need curettage because she had 3
previous abortions before. What should the nurse do?
A. Call the doctor
B. Observation and make more investigation
C. Prepare for immediate curettage and dilatation
Answer: B
363) Meningocele in the ICU what we should Check?
A-infection
B-dehydration
Answer: A
364) What is the type of isolation to Pt with Meningitis?
A-Negative pressure
B-Positive pressure ✅✅

C-Reserve
Answer: B
365) Pregnant woman in Supine position what is the effect?
A. Decrease oxygen to placenta
Answer: A
366) Position post vulva procedure (vulvectomy)??
A. Recumbent
B. Semi Recumbent
C. Supine
Answer: A
367)Position of flatulent??
A. Flat

ِ‫يِاْلَ أرض‬
‫أ‬ ُ ‫َم‬
ُ ‫ك‬
‫ثِِف‬ َ ‫اس‬
‫ِِفي أ‬َ َّ ‫ع‬
‫ِالن‬ ُ ‫ف‬
َ ‫ماِيَ أن‬ َّ َ‫وأ‬
َ ِ‫ما‬ َ
{ 88 }

B. Side lying
C. Knee chest
Answer: C
368) Dexamethasone function? - promote lung Maturation
369)The nurse was giving health education for elderly patient. The patient asked her to
repeat again many times. What is the cause?
A. Patient does not care.
B. Patient confused
C. Sensory defect
D. Mental problem
Answer: C
370) Neonate in well nursery. They suspect that he has diaphragmatic hernia. What
does the Diagnosis confirm diaphragmatic hernia?
A. Ultrasound
*Prenatal - - - - - ultrasound
B. MRI
C. Radiography *Postnatal - - - - - Radiography

Answer: C

371 ) Pregnant complain vaginal blood spots happened 3 days back and repeat
again today. What should the nurse instruct her?
A. Rest on her back and elevate the leg
B. Prepare her for vaginal examination
Answer: A
372) A head nurse of an intensive care unit wrote a report about the needs of her
department and gave it to the nursing supervisor. Which of the following types of
communication is this?
A. Upward
B. Horizontal
C. Diagonal
D. Downward

ِ‫يِاْلَ أرض‬
‫أ‬ ُ ‫َم‬
ُ ‫ك‬
‫ثِِف‬ َ ‫اس‬
‫ِِفي أ‬َ َّ ‫ع‬
‫ِالن‬ ُ ‫ف‬
َ ‫ماِيَ أن‬ َّ َ‫وأ‬
َ ِ‫ما‬ َ
{ 89 }

Answer: A
373) The nurse is assigned to care for an 8-year-old child with a diagnosis of a basilar
skull fracture. The nurse reviews the health care provider's (HCP's) prescriptions and
should contact the HCP to question which prescription?
A. Obtain daily weight.
B. Provide clear liquid intake.
C. Nasotracheal suction as needed.
D. Maintain a patent intravenous line.
Answer: C
374) What precautions are necessary when caring for a patient with Hepatitis A to
protect family from infected?
A. wash dish and do not share.
B. not sharing the toilet.
D. Placing the patient in a private room.
Answer: A
375) child with rheumatic fever how can prevent for recurrent?
A- isolate the infected child.
B- give vaccine for mother.
C- let them complete the antibiotic course.
D- give vaccine in 9 months.
Answer: C
376) What is Thyroxine side effect?
A. Depression
B. Fever
C. Weight gain
D. Increase appetite ✅
Answer: D
377) A neonatal is admitted to the NICU with a meningomyelocele. HR130 .. RR 28…..
TEM 36.7 which of the

ِ‫يِاْلَ أرض‬
‫أ‬ ُ ‫َم‬
ُ ‫ك‬
‫ثِِف‬ َ ‫اس‬
‫ِِفي أ‬َ َّ ‫ع‬
‫ِالن‬ ُ ‫ف‬
َ ‫ماِيَ أن‬ َّ َ‫وأ‬
َ ِ‫ما‬ َ
{ 90 }

following action the nurse should perform to prevent infection of the


meningomyelocele sac ?
A. Wash the sac with betadine every shift
B. Expose the defect to the room air
C. Apply antibiotic cream every 24 hours
D. Cover he sac with moist sterile saline dressing ✅✅

Answer: D
378) The nurse has problem in her schedule then she contacts with manager she follows
chain of command. She has meeting with her director. Which the appropriate
approach of manager to deal with this situation??

A. Assertive
B. Calming

C. Reflective ✅✅
Answer: C
379) Child with Attention deficit hyperactivity disorder (ADHD). The parent complains
that child hyperactive and too much moving. What is your action?
A. Instruct parent to provide private school for child.
B. Instruct parent to provide normal school for child.
C. Instruct parent to allow child to share in activities that need more movement. ✅✅

Answer: C
380) What is the fetus’s name from 3 weeks to 8 weeks?

Embryonic

ِ‫يِاْلَ أرض‬
‫أ‬ ُ ‫َم‬
ُ ‫ك‬
‫ثِِف‬ َ ‫اس‬
‫ِِفي أ‬َ َّ ‫ع‬
‫ِالن‬ ُ ‫ف‬
َ ‫ماِيَ أن‬ َّ َ‫وأ‬
َ ِ‫ما‬ َ
{ 91 }

381) The patient diagnosed with syphilis and He complains a sore (chancre) that is
usually painless. Which phase of syphilis for patient?
A. Primary phase
B. Secondary phase
C. Tertiary phase
Answer: A
382) MR Ahmed admitted to ICU with congestive heart disease his vital signs BP 110/60
HR 120. stroke volume 80 the nurse expected cardiac output to be.
A. 180 B. 1700
C. 2400 D. 9600
Answer: D

COP (cardiac output) = HR (Heart Rate) × STROKE VOLUME


=120 × 80 = 9600

383) Nurse was providing research study for 200 people, the research is answered by
120 people only, calculate how many people that shared in study in percentage?
A-20 %
B-40 %
C-80 %
D-60 % ✅✅

Answer: D
384) The director is planning to develop hospital system. They will improve weakness
point in the system. What is the type of system?
A. Swot analysis system

ِ‫يِاْلَ أرض‬
‫أ‬ ُ ‫َم‬
ُ ‫ك‬
‫ثِِف‬ َ ‫اس‬
‫ِِفي أ‬َ َّ ‫ع‬
‫ِالن‬ ُ ‫ف‬
َ ‫ماِيَ أن‬ َّ َ‫وأ‬
َ ِ‫ما‬ َ
{ 92 }

B. Delphi analysis system

Answer: A
385)infant who weighs 9 kg (19.8 lbs.) requires 900ml of fluids per day for maintenance
fluids. The infant typically consumes 120ml during each feeding. The infant must have
how many feedings per day to meet the fluid maintenance needs?
A. 4
B.8
C.10
D.12
Answer: B
386) After tonsillectomy / thyroidectomy which of the following should be provided for
the patient??
A. Orange juice
B. Apple juice
C. Tomato juice
D. Strawberry juice
Answer: B
387) Most common cause for acute renal failure?
A. pyelonephritis
B. Tubular destruction

ِ‫يِاْلَ أرض‬
‫أ‬ ُ ‫َم‬
ُ ‫ك‬
‫ثِِف‬ َ ‫اس‬
‫ِِفي أ‬َ َّ ‫ع‬
‫ِالن‬ ُ ‫ف‬
َ ‫ماِيَ أن‬ َّ َ‫وأ‬
َ ِ‫ما‬ َ
{ 93 }

C. Urinary tract obstruction


D. Dehydration
Answer: D
388) Qualitative research with evidence
A. survey with ranking
B. interview one to one
C. collect data from system.
Answer: B
389) In the middle of bathing a client, the unit secretary notifies the nurse that there is
an emergency telephone call. Which action should the nurse implement to best assure
client safety?
A. Quickly finish the bath before answering the call.
B. Immediately leave the client’s room and answer the call.
C. Cover the client, place the call light within reach, and then leave to answer.
the call.
D. Leave the door open and ask staff to monitor the client, and then leave to
answer the call.
Answer: C
390) A nurse is caring for 14 - month - old immediately after a surgical repair of cleft
palate. In which of the following positions should the nurse put the child?

A prone ✅✅
B. lateral
C. Supine
D. lithotomy
Answer: A
391) What is the function of breathing exercise before anesthesia and preoperative?
A. To prevent respiratory complication after operation.
B. To prevent Aspiration.

ِ‫يِاْلَ أرض‬
‫أ‬ ُ ‫َم‬
ُ ‫ك‬
‫ثِِف‬ َ ‫اس‬
‫ِِفي أ‬َ َّ ‫ع‬
‫ِالن‬ ُ ‫ف‬
َ ‫ماِيَ أن‬ َّ َ‫وأ‬
َ ِ‫ما‬ َ
{ 94 }

C. To prevent atelectasis.
Answer: A
392) When put baby with high temp. In cold water to lower his temp.
A) convention
B) evaporation
C) conduction
Answer: B
393) What is COPD complication?
A. Cardiac tamponade
B. Pneumothorax
C. Cor pulmonale✅

Answer: C
394) The nurse forgot to raise side rails for patient.?
A. Breach in duty
B. Nature in duty
C. Assault
Answer: A
395) patient is to receive a prescribed blood transfusion using a subclavian catheter.
Catheter was inserted about 30 minutes ago. Which of the following is the most
appropriate step before starting blood transfusion?
A. Check patency of catheter by flushing with normal saline solution.
B. Position patient with the head of the bed elevated 30 degrees.
C. Review result of chest radiograph completed 15 minutes ago.
D. Obtain patient's most recent complete blood count result.
Answer: C
396) What is the best measure to ensure privacy and protect patient information during
uploading password??
A. Write the password in a paper and keep it.

ِ‫يِاْلَ أرض‬
‫أ‬ ُ ‫َم‬
ُ ‫ك‬
‫ثِِف‬ َ ‫اس‬
‫ِِفي أ‬َ َّ ‫ع‬
‫ِالن‬ ُ ‫ف‬
َ ‫ماِيَ أن‬ َّ َ‫وأ‬
َ ِ‫ما‬ َ
{ 95 }

B. Use alphabetic and numbers for password✅

C. Expose patient information record and don't share password


D. Refrain use of the name and Date of birth for the patient
Answer: B
397) Patient habitually worries about any symptoms, which is defense mechanism?
A. Conversion defense mechanism.
Answer: A
398) Which of the following can lead to infertility in adult males?
A. Orchitis
B. German measles
C. Rubella
D. Chicken pox
Answer: A
399) What is the most common cause of fetal hypoxia??
A. fetal descent
B. Full Dilation of cervical
C. Contraction of uterine
D. cervical effacement
Answer: C
400) Which of the following the most common cause of dissociative disorder?
A. Family history
B. Drug abuse
C. Traumatic Event
D. Behavioral changes
Answer: C
401) Cause of hemodilution in pregnancy?
A. Anemia

ِ‫يِاْلَ أرض‬
‫أ‬ ُ ‫َم‬
ُ ‫ك‬
‫ثِِف‬ َ ‫اس‬
‫ِِفي أ‬َ َّ ‫ع‬
‫ِالن‬ ُ ‫ف‬
َ ‫ماِيَ أن‬ َّ َ‫وأ‬
َ ِ‫ما‬ َ
{ 96 }

B. Decrease RBC
C. Increase WBC
D. Increase plasma
Answer: D
402) During suction of child, ensure that each pass of the suction catheter take no more
than?
A-5 Sec✅

B-10 Sec
C-15 Sec
Answer: A
403) What is the food should be limited during pregnancy??
A. Pasteurized Milk
B. Processed Cheese
C. Soft cheese
D. Yogurt
Answer: C
404) Patient after cardiac surgery Recovery return back to hospital with complication.
He stated After I have discharged from hospital to my house, I had thrown all
medication because I felt that I improved no need for drug. Which of the following
action should the nurse take?
A. Assess patient reasons for his action
B. Add new nursing diagnosis of None compliance✅

C. Give Instruction to the patient


Answer: B
405) who is pt. high risk for electrolytes imbalance?
A. 2 years pt. with gastrostomy feeding.
B. Pt with ileostomy
C. Pt with colostomy
Answer: B

ِ‫يِاْلَ أرض‬
‫أ‬ ُ ‫َم‬
ُ ‫ك‬
‫ثِِف‬ َ ‫اس‬
‫ِِفي أ‬َ َّ ‫ع‬
‫ِالن‬ ُ ‫ف‬
َ ‫ماِيَ أن‬ َّ َ‫وأ‬
َ ِ‫ما‬ َ
{ 97 }

406) when caring for a patient with an ostomy the nurse knows that extra skin
protection for the personal skin is MOST important for those patients with a (an):
a- Ileostomy
b- Ascending colostomy
c- Transverse colostomy
d- Sigmoid colostomy
Answer: A
407) a patient with long-standing diabetes mellitus (type I) is scheduled for surgical
amputation of 4 gangrenous toes on the right foot. Which surgical intervention would
this be classified as?
A. Palliative
B. Curative
C. Reconstructive
D. Diagnostic
Answer: A
408) the nurse is teaching a group of nursing students how provide care to clients with
anger management and aggression problem. Which statement made by the nurse is
most indicative of effective teaching?
A. Avoid taking the client's anger personally.
B. Defend yourself from the client's anger.
C. Take responsibility for the client's anger.
D. Respond firmly to the client to relieve your own stress.
Answer: A
409) mother asked the nurse that while she was changing the diaper for her female
newborn, she noticed a brick red stain on it. What is the best response by the nurse?
A. It is a sign of low iron excretion.
B. It is expected in female newborn.
C. It is due to medication given to the mother.
D. it due to medication given to the newborn.

ِ‫يِاْلَ أرض‬
‫أ‬ ُ ‫َم‬
ُ ‫ك‬
‫ثِِف‬ َ ‫اس‬
‫ِِفي أ‬َ َّ ‫ع‬
‫ِالن‬ ُ ‫ف‬
َ ‫ماِيَ أن‬ َّ َ‫وأ‬
َ ِ‫ما‬ َ
{ 98 }

Answer: B
410) A 18-year-old girl was under weight and malnourished her height was 163 cm, her
body weight was 45 kg and her BMI was 16.9. She was admitted in the Female Medical
Word for severe epigastric pain. The nurse observed that on the second day she
induced vomiting after having her dinner. On asking, she replied that inducing vomiting
from time to time helps clear the stomach bacteria.
Which aspect of the nursing care area is altered?
o A. Self-concept
o B. Health perception
o C. Value-belief system
o D. Nutrition management
Answer: B
411) Which of the following action is correct when college a urine specimen
from a client’s indwelling urinary catheter?
A. Collect urine from the drainage collecting bag using sterile.
technique.
B. Disconnect the catheter from the drainage tubing to collect urine.
using clean technique.
C. Remove the indwelling catheter and insert a sterile straight catheter.
to collect urine.
D. Aspirate specimen from the tubing draining port using needle and
syringe with sterile technique
Answer: D
412) A patient with chronic obstructive pulmonary disease (COPD)
Experiencing frequent dyspnea which of the following exercise would
teach the patient how to BETTER control breathing?
a. Lower side rib
b. Segmental
c. Pursed lip

ِ‫يِاْلَ أرض‬
‫أ‬ ُ ‫َم‬
ُ ‫ك‬
‫ثِِف‬ َ ‫اس‬
‫ِِفي أ‬َ َّ ‫ع‬
‫ِالن‬ ُ ‫ف‬
َ ‫ماِيَ أن‬ َّ َ‫وأ‬
َ ِ‫ما‬ َ
{ 99 }

d. Diaphragmatic
Answer: C
413) Pregnant mothers in 9 weeks she came for ER.
with vaginal bleeding and lower abdominal
cramp ...on U/S ...the concept products in
lower uterus
A. Missed abortion
B. Threaten abortion
C. Inevitable abortion
D. Complete abortion
Answer: C
414) How can the nurse detect the poor oxygenation for fetus??
A. Absent Deceleration
B. Absent acceleration
C. Non-reactive fetal heart ctg tracing
D. Breathing movement
Answer: B
415) A 29-year-old woman patient was brought to the Outpatient for the removal of
stitches on her left cheek which was treated nine days back after being involved in
road traffic accident. She covers her face completely and requests to be seen by a
female doctor. The site of the wound was red, swollen and some pussy points were
visible. She states that she did not wash her face since her accident and kept her face
covered all the time as she did not want anyone to see it. Which of the following should
the nurse do to elevate the patient's self-esteem?
A. Involve her husband in her care.
B. Explain that stitches marks will heal soon.
C. Ask her to wash face and wear light make up.
D. Discuss that it is normal to feel low temporarily.
Answer: D

ِ‫يِاْلَ أرض‬
‫أ‬ ُ ‫َم‬
ُ ‫ك‬
‫ثِِف‬ َ ‫اس‬
‫ِِفي أ‬َ َّ ‫ع‬
‫ِالن‬ ُ ‫ف‬
َ ‫ماِيَ أن‬ َّ َ‫وأ‬
َ ِ‫ما‬ َ
{ 100 }

416)-year-old woman patient was brought to the Outpatient for the removal of stitches
on her left cheek which was treated nine days back after being involved in road traffic
accident. She covers her face completely and requests to be seen by a female doctor.
The site of the wound was red, swollen and some pussy points were visible. She states
that she did not wash her face since her accident and kept her face covered all the
time as she did not want anyone to see it. What is the most appropriate nursing
diagnosis?
A. Hopelessness
B. Social isolation
C. Anxiety
D. Powerlessness
Answer: B

417) Pregnant complain from heart burn. What should the nurse advise her?
A. Drink water after ever meal
B. Elevate head of the bed✅
Answer: B

418) The head nurse notice conflict between the


stuff nurses and try to solve this conflict.
She told them that one side give in the pt.
satisfaction What type of this solution??
a- avoidance
B- accommodation
C. Compromise
D. Collaboration
Answer: B
419) Which model is most useful in examining the cause of disease in an individual,
based upon external factors?
A. The preventive health problem model
B. The Agent-Host-Environment Model
Answer: B

ِ‫يِاْلَ أرض‬
‫أ‬ ُ ‫َم‬
ُ ‫ك‬
‫ثِِف‬ َ ‫اس‬
‫ِِفي أ‬َ َّ ‫ع‬
‫ِالن‬ ُ ‫ف‬
َ ‫ماِيَ أن‬ َّ َ‫وأ‬
َ ِ‫ما‬ َ
{ 101 }

420) Recurrent Urinary Tract infection by Escherichia coli bacteria. Which of the
following external factor is the cause?
A. Host
B. Agent
C. Environment
Answer: B
421) A patient is admitted to the hospital for a right total knee replacement. during the
preoperative assessment the patient reports no known during allergies, but the patient
does.
report a severe allergic reaction to shellfish. Why is this allergic reaction to shellfish of
special?
importance?
Shellfish allergies could indicate a negative reaction to anesthesia.
This patient will not be able to receive donor packed red blood cells if needed.
A seafood allergy can be an indication of allergy to provide one-iodine.
A seafood allergy can be precursor to malignant hyperthermia.
Answer: C
422) An 84-year-old bedridden patient with Alzheimer's disease lives in a long-term care
facility. A nurse enters the patient's room at 7:30 and finds the patient lying towards the
foot of the bed on wet she draw sheet beneath t. Another nurse is called to the room
to.
assist. They place a patient and prepare to first move him upwards in the bed which of
the
following is the best explanation for using the sheet?
A- provide material for grasping.
B- Keep the patient dry.
C- prevent skin rubbing.
D- Minimize patient confuse.
Answer: C

ِ‫يِاْلَ أرض‬
‫أ‬ ُ ‫َم‬
ُ ‫ك‬
‫ثِِف‬ َ ‫اس‬
‫ِِفي أ‬َ َّ ‫ع‬
‫ِالن‬ ُ ‫ف‬
َ ‫ماِيَ أن‬ َّ َ‫وأ‬
َ ِ‫ما‬ َ
{ 102 }

423) Patient came to a scheduled appointment for follow up when the nurse checks
the vital signs BP: 130/92 RR: 18 HR: 86T: 37Which of the vital signs is abnormal?
A- Temperature
B- Heart rate
C- Respiratory rate
D- Blood pressure
Answer: D Pre hypertensive
424(Which hormones is responsible for uterine wall change and breast enlargement
during pregnancy, leucorrhea and hyperemesis. Which of the following placental
hormone is responsible??
A. Progesterone

B. Estrogen ✅✅
C. HCG
Answer: B
425) Which of the following hormones are responsible for implantation of ovum inside
Uterine wall??

A. Progesterone and estrogen ✅✅

B. LH and. FSH Hormones


C. Oxytocin
Answer: A
426) There is crisis in hospital. Which of the following category of patients should be
evacuated first during fire???
A. Infant

B. Ambulatory✅

C. elderly
D. Critically ill
Answer: B
427) – Patient came to ER complain from blood in stool. What is the most common
diagnosis??

ِ‫يِاْلَ أرض‬
‫أ‬ ُ ‫َم‬
ُ ‫ك‬
‫ثِِف‬ َ ‫اس‬
‫ِِفي أ‬َ َّ ‫ع‬
‫ِالن‬ ُ ‫ف‬
َ ‫ماِيَ أن‬ َّ َ‫وأ‬
َ ِ‫ما‬ َ
{ 103 }

A. GERD
B. Gastroenteritis
C. BID bowel inflammatory disease ✅✅

Answer: C
428) Pregnant complain nausea too much early morning. The nurse should advise her?
A. Instruct her to increase activity

B. Advise her to get biscuits and cracker before getting out bed✅

Answer: B
429) Patient has cystic fibrosis and the nurse wants to perform postural drainage when is
the best time for posture drainage?
A-Before meals

B-After meals after half an hour of meals✅

C-Before sleeping
D. Afternoon
Answer: B

430) nurses are caring for a client who has just returned from having a right-sided renal
biopsy. The nurse should intervene if a nursing assistant is observed doing which of the
following?
A. Obtaining the client's vital signs
B. Positioning the client on the left side
C. Positioning the client on the right side
D. Providing the client with reading materials
Answer: B
431) A nurse is a member of the Nursing Research Council of the hospital. His first
assignment is to determine the level of patient satisfaction on the care they received
from the hospital. He plans to include all adult patients admitted from April to May, with

ِ‫يِاْلَ أرض‬
‫أ‬ ُ ‫َم‬
ُ ‫ك‬
‫ثِِف‬ َ ‫اس‬
‫ِِفي أ‬َ َّ ‫ع‬
‫ِالن‬ ُ ‫ف‬
َ ‫ماِيَ أن‬ َّ َ‫وأ‬
َ ِ‫ما‬ َ
{ 104 }

average length of stay of 3-4 days, first admission, and with no complications. Which of
the following is an extraneous variable of the study?

A. Date of admission

B. Length of stay

C. Age of patients ✅

D. Absence of complications

Answer: C
432) A nurse is caring for a child who sustained a head injury after falling from a tree. On
assessment of the child, the nurse notes the presence of a watery discharge from
the child's nose. The nurse will immediately test the discharge for the presence of which
of the following substance?

A. Glucose✅

B. b. Protein
C. c. White blood cells
D. d. Neutrophils
Answer: A
433) Pregnant 36 weeks gestational with irregular painless contraction this signs
called ?
A-Hegar's sign
B- Chadwick sign
C-Braxton Hicks (false labor) ✅
D-True labor
Answer: C
434) The nurse is preparing to insert peripheral Cannula for patient. Which of the
following patients avoid insert cannula in lower extremity for them it is risky?

A. DM✅ B. Renal failure


C. COPD D. Heart failure
Answer: A
435) To make cornea reflux you should use?
A. Straily gloves B. Swap cotton✅

ِ‫يِاْلَ أرض‬
‫أ‬ ُ ‫َم‬
ُ ‫ك‬
‫ثِِف‬ َ ‫اس‬
‫ِِفي أ‬َ َّ ‫ع‬
‫ِالن‬ ُ ‫ف‬
َ ‫ماِيَ أن‬ َّ َ‫وأ‬
َ ِ‫ما‬ َ
{ 105 }

C. Cotton tap D. Sterile tongue depressor


Answer: B
436) Alkhurma hemorrhagic fever??

Flaviviridae virus ✅
437) Patient with TB going on Isoniazid medication. To prevent peripheral Neuropathy
What should be included in care plan advice?
A. Instruct low protein diet
B. Avoid sun exposure too much

C. Provide vitamin B6 intake✅


D. Increase fluid intake
Answer: C
438) Which of the following circumstances is most likely to cause uterine atony and lead
to postpartum hemorrhage?
A- endometriosis
B- urine retention
C- cervical and vaginal tears
D- hypertension
Answer: B
439) COMPLICATION OF Aortic Aneurysm - - - - - - Bleeding and hypotension
440) Aortic aneurysm - - - - - - - - - Emergency Care

441) Patient diagnosed MI then suffered from cardiogenic shock. What should the
nurse observe in symptoms?
A. Hypertension and hyperventilation
B. Hypotension and shortness of breath
Answer: B
442) Risks of MI - - - - - - hyperlipidemia

443) The nurse receives ECG for patient via Internet. What is consider?
A. Telehealth

ِ‫يِاْلَ أرض‬
‫أ‬ ُ ‫َم‬
ُ ‫ك‬
‫ثِِف‬ َ ‫اس‬
‫ِِفي أ‬َ َّ ‫ع‬
‫ِالن‬ ُ ‫ف‬
َ ‫ماِيَ أن‬ َّ َ‫وأ‬
َ ِ‫ما‬ َ
{ 106 }

B. Telecommunication
C. Telemedicine
Answer: B
444) Group of health care givers provide care for group of patients have same
diagnosis at same setting. What is this considering?
A. Team
B. Case management
C. Primary
D. Functional
Answer: B
445) The nurse received patient from Recovery room PACU. What is the first assessment
should the nurse observe?
A. Oxygen
B. Level of consciousness
Answer: A

446) The disease divided According to three factors host ،Environment ،Agent. The
patient complete antibiotics course. Then the disease returns again. What is the
causative factor ??
A. Agent
B. Environment
C. Host
D. Reaction between host and environment
Answer: D
447) The patient with central venous line. CVP measure 1 mmhg. The patient
hemoglobin 9.8 g/dl. What should give first ?
A. Give blood transfusion
B. Give 2 L N/S
Answer: B

448) Why should the nurse not keep drainage bag at the body level of the patient
during transporting him?
A. Prevent entry of air

ِ‫يِاْلَ أرض‬
‫أ‬ ُ ‫َم‬
ُ ‫ك‬
‫ثِِف‬ َ ‫اس‬
‫ِِفي أ‬َ َّ ‫ع‬
‫ِالن‬ ُ ‫ف‬
َ ‫ماِيَ أن‬ َّ َ‫وأ‬
َ ِ‫ما‬ َ
{ 107 }

B. Prevent entry of fluid to patient


C. Prevent tension pneumothorax
Answer: B
449) The patient admitted hospital for surgical procedure. The preoperative preparing
phase started. What is the first action in this phase?
A. Obtain informed consent
B. Surgical procedure explanation ✅

Answer: B
450)Patient with COPD uses pursed lip. What is the function for using this maneuver?
A. Relax respiratory muscle ✅✅

B. Facilitate inhalation to medication.


Answer: A
451) Nursing diagnosis done and the nurse set priorities for all NANDA. Which step in
nursing process the nurse doing??
A. Planning ✅✅

B. Diagnosis
C. Evaluation
D. Implementation
Answer: A
452) The nurse is teaching a patient about spironolactone (Aldactone).
Which of the following instructions should review with the patient?
A. Increasing the intake of foods that are high in potassium.
B. Taking the medication right before going to sleep.
C. Avoid seasonings that are labeled as salt substitutes.
D. Scheduling the medication so that a multivitamin is taken an hour later.
Answer: C

placenta ‫👇👇 سؤال االمتحان بييجي بالشكلين دول ده سؤال وظيفة ال‬

ِ‫يِاْلَ أرض‬
‫أ‬ ُ ‫َم‬
ُ ‫ك‬
‫ثِِف‬ َ ‫اس‬
‫ِِفي أ‬َ َّ ‫ع‬
‫ِالن‬ ُ ‫ف‬
َ ‫ماِيَ أن‬ َّ َ‫وأ‬
َ ِ‫ما‬ َ
{ 108 }

453) A nurse explains some of the purposes of the placenta to a client during a prenatal
visit. The nurse determines that the client understands some of these purposes when the
client states that the placenta:
Cushions and protects the baby.
Maintains the temp of the baby.

Is the way the baby gets food and oxygen✅✅?

Prevents all antibodies and viruses from passing to the baby.


Answer: 3
454 (Which purposes of placental functioning should the nurse include in a prenatal
class? Select all that apply.
it cushions and protects the baby.
it maintains the temperature of the baby.
it is the way the baby gets food and oxygen✅✅
it prevents all antibodies and viruses from passing to the baby.
it provides an exchange of nutrients and waste products between the mother and
developing fetus✅✅

Answer: C+E

Amniotic fluid ‫هي مش بتحمي البيبي ✋ اللي بيحمي البيبي‬

455)

Answer: premature ventricular complex


456) A 67-year-old man is admitted to the Post-Anesthesia Recovery unit following
chest surgery. The patient has a right chest tube that is attached to low suction.
Three hours after admission to the unit, the nurse observes the drainage output.
from the chest tube is 300 milliliters.

ِ‫يِاْلَ أرض‬
‫أ‬ ُ ‫َم‬
ُ ‫ك‬
‫ثِِف‬ َ ‫اس‬
‫ِِفي أ‬َ َّ ‫ع‬
‫ِالن‬ ُ ‫ف‬
َ ‫ماِيَ أن‬ َّ َ‫وأ‬
َ ِ‫ما‬ َ
{ 109 }

What is the most appropriate initial intervention?


Notify the doctor.
Reduce IV infusion rate.
Strip tube with roller device
Re-position in left lateral decubitus
Answer: A
457) 21-You are assisting a MD with the removal of a chest tube. What activity may the
MD have the patient perform while the chest tube is being remove??
A. Valsalva maneuver.
B. Leopold Maneuver
C. Chest Physiotherapy
D. Huff Cough Technique
Answer: A

458) The nurse wittiness for another nurse while entering to the isolated pt room with
airborne precautions and she wear only gown and gloves, what is the appropriate
action from the wittiness nurse to do?
1. Talk with the nurse about wearing mask
2. Talk with the education nurse to introduce lecture for staff about PPE
3. Inform the head department about the nurse
4. Ignored the nurse action
Answer: A

459) Which of the following should be included in the nursing care to a woman during
2nd stage of labor?
A. Shave the perineum
B. Administer enema to the woman.
C. Careful evaluation of prenatal history
D. Watch breathing, bear down with each contortion.

ِ‫يِاْلَ أرض‬
‫أ‬ ُ ‫َم‬
ُ ‫ك‬
‫ثِِف‬ َ ‫اس‬
‫ِِفي أ‬َ َّ ‫ع‬
‫ِالن‬ ُ ‫ف‬
َ ‫ماِيَ أن‬ َّ َ‫وأ‬
َ ِ‫ما‬ َ
{ 110 }

Answer: D
460) MD writes an order for Ibuprofen 3 mg/kg by mouth every 6 hours for pain for a
child. The child weighs 73 lb. Pharmacy dispenses you with 50 mg/2 ml. How many ml
will you administer per dose? *
A. 1 ml/dose
B. 3.9 ml/dose
C. 2.9 ml/dose
D. 6 ml/dose
Answer: B
461) A patient’s wound is draining thick yellow material. The nurse correctly describes
the
drainage as which of the following?

A. Serous
B. Purulent
C. Sanguineous
D. Serous-sanguineous
Answer: B
462) patient came to ER with complain abdominal patient history appendicitis
patient now with change pass stool distention abdominal and diharria
which most diagnosis?
A. colitis
B. peritonitis
C. gastritis
D. none
Answer: A
463) Schizophrenic patient with auditory hallucination. What should the nurse do?
A. Accept the patient
B. Put patient in isolated room
C. Let him share in activity
D. Say Don't hallucinate

ِ‫يِاْلَ أرض‬
‫أ‬ ُ ‫َم‬
ُ ‫ك‬
‫ثِِف‬ َ ‫اس‬
‫ِِفي أ‬َ َّ ‫ع‬
‫ِالن‬ ُ ‫ف‬
َ ‫ماِيَ أن‬ َّ َ‫وأ‬
َ ِ‫ما‬ َ
{ 111 }

Answer: C
464) Patient has cast on his arm. The patient complaining sever pain. The nurse not
palpating the pulse. What is the first action?
A. It is normal
B. Continue observation
C. Report the physician
D. It is not normal
Answer: C
465) The doctor decided DNR for patient. The patient refused that. What should the
nurse do?
A. Follow hospital policy and respect his desire
B. Give the care, continue treatment
C. Ignore the patient need
D. Stop all medication
Answer: A
466) An older-adult patient has developed acute confusion. The patient has been on
tranquilizers for the past week. The patient’s vital signs are normal. What should the
nurse do?
A. Take into account age-related changes in body systems that affect
pharmacokinetic activity.
B. Increase the dose of tranquilizer if the cause of the confusion is an infection.
C. Note when the confusion occurs and medicate before that time.
D. Restrict phone calls to prevent further confusion.
Answer: A
467) The doctor decided DNR for patient. The patient refused that. He insisted to do
CPR. What should the nurse do?
A. Continue to produce care and reduce pain
B. Discontinue the treatment
Answer: A
468) what complications after cleft palate repair?
A) Teeth pain

ِ‫يِاْلَ أرض‬
‫أ‬ ُ ‫َم‬
ُ ‫ك‬
‫ثِِف‬ َ ‫اس‬
‫ِِفي أ‬َ َّ ‫ع‬
‫ِالن‬ ُ ‫ف‬
َ ‫ماِيَ أن‬ َّ َ‫وأ‬
َ ِ‫ما‬ َ
{ 112 }

B) Deviated septum.
C) speech difficulty
D)recurrent tonsillitis
Answer: C
469) what predisposing factor of child with autism
A) family history
B) child abuse
C) history of AHDH and mental illness
Answer: A
470) The nurse is caring for Postpartum mother has DVT. What is the most appropriate
action for her??
A. Ambulation
B. Elevate left leg.
C. Apply ice back.
D. Give prescribed anticoagulant
Answer: B
471) The nurse should avoid the use of the dorsolateral site for
an intramuscular injection in children because of the risk of
injury to which of the following nerves?
A. Vigus
B. Sciatic
C. Ilioinguinal
D. Lumbar plexus
Answer: B

ِ‫يِاْلَ أرض‬
‫أ‬ ُ ‫َم‬
ُ ‫ك‬
‫ثِِف‬ َ ‫اس‬
‫ِِفي أ‬َ َّ ‫ع‬
‫ِالن‬ ُ ‫ف‬
َ ‫ماِيَ أن‬ َّ َ‫وأ‬
َ ِ‫ما‬ َ
{ 113 }

472) Persons try to stop smoking after that smoking cessation done successfully and
continue in cessation which stage of change the patient achieved?
A. He stop applying change process in action stage.
B. He stop applying change process in preparation stage.
C. He stop applying change process before maintenance stage complete.
D. He stop applying change process after Maintenance complete.
Answer: D
473) Nurse manger float RN nurse from surgical ward to CCU which patient should be
assigned to this nurse
A. Patient just transferred after coronary angiography.
B. Patient postoperative after open heart surgery
C. Patient on I. V Lasix have congestive heart failure✅✅

D. Patient had ventricular tachycardia and connected to cardiac monitor.


Answer: C
474) Nurse mangers pull out (float) nurse from medical ward to CCU which of the
following Patients should be assigned to the nurse:
A. Patient just transferred from cardiac catheterization✅✅

B. Patient just received with unstable angina on heparin infusion.


C. Patient need discharge education about coronary stenting.
D. Patient on Lasix I. V have acute left ventricular failure.
Answer: A

ِ‫يِاْلَ أرض‬
‫أ‬ ُ ‫َم‬
ُ ‫ك‬
‫ثِِف‬ َ ‫اس‬
‫ِِفي أ‬َ َّ ‫ع‬
‫ِالن‬ ُ ‫ف‬
َ ‫ماِيَ أن‬ َّ َ‫وأ‬
َ ِ‫ما‬ َ
{ 114 }

RN ‫لو ساب السؤال ومحددليش‬


LPN‫يبقى هختار على اساس‬
‫ هيحددلي في السؤال ويكتبه‬RN ‫لكن لو عايز‬
‫ دول الناس اللي بيتفوضلهم بس‬As a general
‫نفهم ونختار عموما الن مش دول السؤالين اللي هيجوا بس هيجيبو أسئلة تانيه مش عايزين نستقبل‬
‫✋✋اإلجابة الصح وخالص‬
475) A newly nursing director assigned to a hospital. That is the first should he do?
A. Evaluates the staff
B. Change the head nurses.
C. Change the roles
D. Nothing to do
Answer: A
476) A patient is admitted to the hospital with klebsiella pneumoniae. During the initial
intravenous dose of Amikacin (amikacin sulfate), the patient develops severe
respiratory distress. This is most likely:
A) A side effect
B) An indication of drug tolerance
C) A drug allergy

ِ‫يِاْلَ أرض‬
‫أ‬ ُ ‫َم‬
ُ ‫ك‬
‫ثِِف‬ َ ‫اس‬
‫ِِفي أ‬َ َّ ‫ع‬
‫ِالن‬ ُ ‫ف‬
َ ‫ماِيَ أن‬ َّ َ‫وأ‬
َ ِ‫ما‬ َ
{ 115 }

D) A toxic effect
Answer: D
477)A 47-year patient received spinal anesthesia five hours ago in the operation
theatre and was transferred to the surgical unit. Few hours later the patient complains
of incisional pain. The surgical site is dry and intact but the patient looks pale irritated.
BP 175/90 HR 112 RR 27 TEM 37.6
What is the most appropriate nursing intervention?
A. Take a 12 lead ECG
B. Administer pain medication
C. Notify treating physician ✅
D. Strat IV infusion and place patient in supine
Answer: C
479) A patient with impaired hearing, He is wearing a hearing aid. The nurse is preparing
him to operation, He asked the nurse to keep his hearing aid while he is going to
operation.
A. Leave patient wear his hearing aid.
B. Ask Operation Nursing staff
C. Say it is forbidden. You have to remove it.
D. He does not need it because he will be given general anesthesia.
Answer: A
480) Patient is taking streptokinase. Suddenly he is complaining fever, shortness of
breathing. What should the nurse expect that effect?
A. Side effect
B. Allergic reaction
C. Normal response
Answer: B
481) Cause of decrease the natural immunity of reproductive system after
menopause??
A. Decrease cervical and vaginal mucus

ِ‫يِاْلَ أرض‬
‫أ‬ ُ ‫َم‬
ُ ‫ك‬
‫ثِِف‬ َ ‫اس‬
‫ِِفي أ‬َ َّ ‫ع‬
‫ِالن‬ ُ ‫ف‬
َ ‫ماِيَ أن‬ َّ َ‫وأ‬
َ ِ‫ما‬ َ
{ 116 }

B. Decrease genital tissue and subcutaneous fats


C. Thinning of pubic hair
Answer: B
482) According to Erickson, the psychosocial task of adolescence is the development
of a sense of identity. The nurse can best promote the development of a hospitalized
adolescent by:
A. Emphasizing to follow the facility regime.
B. Allowing the parents and siblings to visit frequently.
C. Arranging for tutoring in schoolwork.
D. Encouraging peer visitation
Answer: C
483) One day a male client with the diagnosis of borderline personality disorder
describes a situation that happened at work when his immediate supervisor
reprimanded him for not completing an assignment. He explains that it was not his fault
and states, "people get angry and take it out on me." Which defense mechanism
identified by the nurse was the client using in this situation?
1. Denial
2. Projection
3. Displacement
4. Intellectualization
Answer: B
484) Some of Narcotic group arranged to C1-C5. That is According to:
A. Dose
B. Abuse ✅

C. Severity
Answer: B
485) Pregnant complain leg cramp. What should the nurse advise her?
A. Elevate the leg above hip level

B. Change her position continuously✅

ِ‫يِاْلَ أرض‬
‫أ‬ ُ ‫َم‬
ُ ‫ك‬
‫ثِِف‬ َ ‫اس‬
‫ِِفي أ‬َ َّ ‫ع‬
‫ِالن‬ ُ ‫ف‬
َ ‫ماِيَ أن‬ َّ َ‫وأ‬
َ ِ‫ما‬ َ
{ 117 }

C. Apply foot support during setting


D. Wearing slipper
Answer: B
486) Postpartum mother was complaining of mastitis on the Right breast, and she told
the nurse that it's safe feeding by my left side. What should The nurse response?
A. Breastfeed from both sides it is safe
B. The left side is only safe
C. the Right side is the Safe
Answer: A
487) Extrapyramidal adverse effects and symptoms are most often associated with
which of the following drug
classes?
A. Antidepressants
B. Antipsychotics
C. Antihypertensives
D. Antidysrhythmic
Answer: B
488) Sudden infant death syndrome (SIDS) is one of the most common causes of
death in infants. To prevent this syndrome, the nurse should keep infant inside
incubators in which position??
A. Left side
B. Back✅✅
C. Prone
D. Right side
Answer: B
489) Sudden infant death syndrome (SIDS) is one of the most common causes of death
in infants. Which of the following position should the nurse avoid for infant that cause
SIDS??
A. Left side

ِ‫يِاْلَ أرض‬
‫أ‬ ُ ‫َم‬
ُ ‫ك‬
‫ثِِف‬ َ ‫اس‬
‫ِِفي أ‬َ َّ ‫ع‬
‫ِالن‬ ُ ‫ف‬
َ ‫ماِيَ أن‬ َّ َ‫وأ‬
َ ِ‫ما‬ َ
{ 118 }

B. Back

C. Prone✅✅

D. Right side
Answer: C
490) After 3 days of breast feeding a post partial patient reports nipple soreness. To
relieve her discomfort the nurse
should suggest that she:
A. lubricate her nipples with expressed milk before feeding.
B. dry her nipples with soft towel after feeding.
C. Apply warm compresses to her nipples just before feeding.
D. Apply soap directly to her nipples, and then rinse.
Answer: A

491) What is the primary sign for ARF acute renal failure?
A. Hypertension
B. Oliguria
C. Face edema
Answer: B
492) Infant 11 month On ECG heart rate 240 b/m. Embedded in the QRS complexes
and absent p wave. What is the expected diagnosis for the infant???
A. Ventricular tachycardia (VT)
B. Bradycardia

ِ‫يِاْلَ أرض‬
‫أ‬ ُ ‫َم‬
ُ ‫ك‬
‫ثِِف‬ َ ‫اس‬
‫ِِفي أ‬َ َّ ‫ع‬
‫ِالن‬ ُ ‫ف‬
َ ‫ماِيَ أن‬ َّ َ‫وأ‬
َ ِ‫ما‬ َ
{ 119 }

C. Supraventricular tachycardia (SVT)


D. Atrial fibrillation
Answer: C
493) The nurse is caring for a client who has had a right modified radical
mastectomy this morning. Which exercise should the nurse encourage the?
client to perform this evening.
a- Hair combing exercises with the right arm.
b- Wall climbing exercises with the right arm.
c- Movement of the fingers and wrists of the right arm
d- Exercises of the left arm only
Answer: C
494) There is lecture about health model and illness model, and in the lecture included
the agent / host / environment model.
What is considered that model to be classified as:
A. Stage of illness

B. Risk factor✅
Answer: B
495) Patient is complaining from hypostatic hypotension
Before that, his blood pressure was 110/70, the pulse was 76 b/m
What is the expected vital signs?
A. BP 90/60 HR 69
B. BP 88/60 HR 100✅

Answer: B
496) A 9 -year-old child is admitted to the Emergency Department injury. The child is
oriented to the place, person and time, spontaneously, obeys commands. The nurse is
doing a ped Coma Scale (PGCS). Which of the following score the nurse should
record?
A. 3
B. 8

ِ‫يِاْلَ أرض‬
‫أ‬ ُ ‫َم‬
ُ ‫ك‬
‫ثِِف‬ َ ‫اس‬
‫ِِفي أ‬َ َّ ‫ع‬
‫ِالن‬ ُ ‫ف‬
َ ‫ماِيَ أن‬ َّ َ‫وأ‬
َ ِ‫ما‬ َ
{ 120 }

C. 12
D. 15
Answer: D
497) Pregnant women in16 week sever hyperemesis garvidurm what is the
complication for pregnant?
A. Preeclampsia
B. Electrolyte imbalance
C. Miscarriage
Answer: B
498) The nurse is caring for a client who has had a chest tube inserted and connected
to water seal drainage. The nurse determines the drainage system is functioning
correctly when which of the following is observed:
A. Continuous bubbling in the water seal chamber

B. Fluctuation in the water seal chamber✅


C. Suction tubing attached to a wall unit.
D. Vesicular breath sounds throughout the lung fields.
Answer: B
499) The nurse is caring for a client who has just had a chest tube attached to a water
seal drainage system. To ensure that the system is functioning effectively the nurse
should:

A. Observe for intermittent bubbling in the water seal chamber✅

B. Flush the chest tubes with 30-60 ml of NSS every 4-6 hours
C. Maintain the client in an extreme lateral position.
D. Strip the chest tubes in the direction of the client
Answer: A
500) client chest tube is connected to a chest tube drainage system with a water seal.
The nurse noted that the water seal c is fluctuating with each breath that client takes.
The fluctuation means that.
A. There is an obstruction in the chest tube.
B. The client is developing emphysema.

ِ‫يِاْلَ أرض‬
‫أ‬ ُ ‫َم‬
ُ ‫ك‬
‫ثِِف‬ َ ‫اس‬
‫ِِفي أ‬َ َّ ‫ع‬
‫ِالن‬ ُ ‫ف‬
َ ‫ماِيَ أن‬ َّ َ‫وأ‬
َ ِ‫ما‬ َ
{ 121 }

C. The chest tube system is functioning properly ✅

D. There is leak in the chest tube system.


Answer: C
501) What the purpose of a late vaginal_rectal culture during ante natal screening?
A. To screen for group B streptococci
B. To detect a Chlamydia infection.
C. To screen for gonorrhea
D. To screen for syphilis
Answer: A
502) What is the Noninvasive test for Hirschsprung disease?
A. Ultrasound
B. Anorectal manometry
C. X- ray
Answer: B
503) Nurse is watching the cardiac monitor, and a client’s rhythm suddenly changes.
There are no P waves; instead, there are wavy lines. The QRS complexes measure 0.08
second, but they are irregular, with a rate of 120 beats a minute. The nurse interprets this
rhythm as:
A-Sinus tachycardia
B-Atrial fibrillation
C-Ventricular tachycardia
D-Ventricular fibrillation
Answer: B
504) The patient is postoperative cardiac surgery. The pulse rate is changed to
bradycardia. What is the most common cause of decrease Heart rate after operation?
A. Pain
B. Stimulate Vegas nerve
C. Anxiety
Answer: B

ِ‫يِاْلَ أرض‬
‫أ‬ ُ ‫َم‬
ُ ‫ك‬
‫ثِِف‬ َ ‫اس‬
‫ِِفي أ‬َ َّ ‫ع‬
‫ِالن‬ ُ ‫ف‬
َ ‫ماِيَ أن‬ َّ َ‫وأ‬
َ ِ‫ما‬ َ
{ 122 }

505) child with iron deficiency complains of feeling tired all the Times. The nursing
diagnosis of fatigue is related to:
A- Decreased ability of the blood to transparent oxygen to the Tissues
B-An increased paroxysmal abdominal pain and distension to the stomach
C-A decreased anxiety level during hospitalization.
D-A decreased nutritional intake with mal absorption of
Answer: A
506) 35 years old female patient after taking breast biopsy she diagnosed with 3rd
stage breast cancer
She asked what is the purpose of chemotherapy?
A-it make no need for surgery.
B- destroy cancer cells and prevent it from spreading.
C- prevent recurrence of cancer.
Answer: B
507) An old Man 72 years old with insomnia which most likely If patient said will need
further instructions?
A-Avoid nap
B-Read book before sleep
C- Put the temperature room cold.
D-avoid drink caffeine after the dinner.
Answer: c
508) When should stop performing sponges bathing for febrile infant?
A. Shivering
B. Tachycardia
C. Mettled skin
D. Increase respiratory rate
Answer: A

509) The doctor is allow woman to go in with her husband during CPR ressustation.
What should the nurse tell her?

ِ‫يِاْلَ أرض‬
‫أ‬ ُ ‫َم‬
ُ ‫ك‬
‫ثِِف‬ َ ‫اس‬
‫ِِفي أ‬َ َّ ‫ع‬
‫ِالن‬ ُ ‫ف‬
َ ‫ماِيَ أن‬ َّ َ‫وأ‬
َ ِ‫ما‬ َ
{ 123 }

A. Try to stand away in seperate place to prevent stoping work because the work
at this time very fast✅
B. Don't cry or move during CPR and don't try to share
Answer: A

510) A nurse is reviewing blood chemistry result of a client magnesium 1.7 0.7-1.2 With of
the following
symptoms the nurse be monitored.
A. Trousseau sign
B. Depressed respirations
C. Elevated blood pressure
D. Increased tendon reflex
Answer: B

511) What is the priority nursing assessment before administering methergine for
management of postpartum
hemorrhage?
A. Blood pressure
B. Uterine atony
C. Amount of lochia
D. Deep tendon reflex
Answer: A
512) A pregnant woman informed a nurse that she was never vaccinated against
rubella. Which of the following is the best?

ِ‫يِاْلَ أرض‬
‫أ‬ ُ ‫َم‬
ُ ‫ك‬
‫ثِِف‬ َ ‫اس‬
‫ِِفي أ‬َ َّ ‫ع‬
‫ِالن‬ ُ ‫ف‬
َ ‫ماِيَ أن‬ َّ َ‫وأ‬
َ ِ‫ما‬ َ
{ 124 }

nursing advice?
A. No need for her to be distress, rubella is not harmful to the fetus.
B. The vaccine can be administered any time during her pregnancy.
C. She can get pregnancy any time after receiving the vaccine.
D. She should be vaccinated after delivery of the baby she get discharge.
Answer: D
513) The antenatal clinic nurse was assessing a 32yaers old gravida 2, para1 pregnant
mother on the fundal height
at 36 weeks. What is the expected fundus position?
A. Umbilicus
B. Xiphoid process
C. Symphysis pubis
Answer: B
514) A mother is asking the nurse how sickle cell is could be deducted. Which of the
following is at test detected?
sickle cell disease of traits?
A. Hemoglobin electrophoresis
B. Bone marrow aspiration
C. Complete blood count
D. Free erythrocyte protoporphyrin
Answer: A
515) A child with thalassemia was given deferoxamine (Deferral); which of the following
should alert the nurse to notify the physician?
A. Decreased hearing
B. Hypertension
C. Red urine
D. Vomiting
Answer: A

ِ‫يِاْلَ أرض‬
‫أ‬ ُ ‫َم‬
ُ ‫ك‬
‫ثِِف‬ َ ‫اس‬
‫ِِفي أ‬َ َّ ‫ع‬
‫ِالن‬ ُ ‫ف‬
َ ‫ماِيَ أن‬ َّ َ‫وأ‬
َ ِ‫ما‬ َ
{ 125 }

516) The nursing assistant with 20 years of experience approaches a recently graduated
nurse who recently passed the
licensing examination. The nursing assistant states “the only difference between you
and me is the size of our pay.
checks”. Which of the following is the most appropriate?
response for the newly graduated nurse?
A. assert a hierarchical position.
B. emphasize the additional education received.
C. explain the legal difference in the scope of practice.
D. focus on the need to work together for quality client care.
Answer: D

517) During clinical assessment, the nurse suspected that a child has been
psychological abuse. Which statement
by the child is most likely support the suspicion.
A. My parents tell me that I am stupid.
B. My parents hurt me to get attention from the doctors.
C. My mother forces do not give meals on times.
D. My uncle shows me picture of nude people.
Answer: A
518) A nurse accidentally dropped a medication ampule, informed the charge nurse
and complicated an incident.
report form. Which of the following best describes the nurse role?

ِ‫يِاْلَ أرض‬
‫أ‬ ُ ‫َم‬
ُ ‫ك‬
‫ثِِف‬ َ ‫اس‬
‫ِِفي أ‬َ َّ ‫ع‬
‫ِالن‬ ُ ‫ف‬
َ ‫ماِيَ أن‬ َّ َ‫وأ‬
َ ِ‫ما‬ َ
{ 126 }

A. Responsiveness
B. Timely decision making
C. Profession accountability
D. Abiding by moral obligation
Answer: C
519) What is the Priority of patient with schizophrenia?
A. Self esteem
B. Medication compliance
C. Family support
D. Impaired thought process
Answer: B

520) Which of the following is the leading cause of injury for primary school?
A. accidental suffocation
B. motor vehicle✅

C. riding bicycle
D. drowning
Answer: B

‫👇👇 صيغة السؤال بطريقة تانيه‬

ِ‫يِاْلَ أرض‬
‫أ‬ ُ ‫َم‬
ُ ‫ك‬
‫ثِِف‬ َ ‫اس‬
‫ِِفي أ‬َ َّ ‫ع‬
‫ِالن‬ ُ ‫ف‬
َ ‫ماِيَ أن‬ َّ َ‫وأ‬
َ ِ‫ما‬ َ
{ 127 }

521) which of the following is the leading cause of injury for children who are more than
five years old?
A. accidental suffocation
B. motor vehicle✅

C. congenital anomalies
D. drowning
Answer: B

522) What is the importance of vitamin K to stop bleeding. How it act ??


A. Converting prothrombin to thrombin✅
Answer: A

___Action of vitamin K increases the release of clotting factor by forming


prothrombin to secrete factors D VII, IX, X, seventh, ninth and tenth that
prevent bleeding.

523) Postpartum mother used the IUD as a contraceptive method. The nurse should
instruct her ?
A. Check for strips every month follow up✅
B. Avoid intercourse for 1 week
Answer: A

524) Child has urine incontinence two day ago with increase WBC in urine. What is the
cause??
A. Psychological abuse
B. Urinary tract infection
C. Genitalia defect
Answer: B
525) Which of the following methods need to use calendar in Natural method of family
planning??
A. 14 day from beginning of the period.

ِ‫يِاْلَ أرض‬
‫أ‬ ُ ‫َم‬
ُ ‫ك‬
‫ثِِف‬ َ ‫اس‬
‫ِِفي أ‬َ َّ ‫ع‬
‫ِالن‬ ُ ‫ف‬
َ ‫ماِيَ أن‬ َّ َ‫وأ‬
َ ِ‫ما‬ َ
{ 128 }

B. Body TEMP measure daily at morning


Answer: A

526) What is the benefit of skin to skin after delivery?


A- Control mother and baby temperature
B- Initiate and facilitate early breast-feeding during 3rd stage of labor
C- Improve circulation of baby.
D. Improve Uterine Contraction
Answer: B Control baby temperature ONLY NOT mother temperature
527) A 2-year-old child is admitted to pediatric ward. The mother cannot stay the child
and she will visit him in the
weakened only. Which of the following nursing action indicates an understanding of
the emotional needs this?
child?
A. Give her a warm bath to calm down.
B. Allow the child to suck on a pacifier.
C. Ask the parents to bring the child favorite toy.
D. Tell he parents that frequently visiting is unnecessary.
Answer: C
528) Full term Newborn was delivered 2 hour ago. The nurse checks him observed.
mild peripheral cyanosis. What is the first action?
A. Call for help
B. Notify the doctor
C. Check temperature
Answer: C
529) A community Nursing nurse is planning to conduct prenatal teaching and
community assembly for pregnant
adolescents. Which teaching strategy would be most effective?

ِ‫يِاْلَ أرض‬
‫أ‬ ُ ‫َم‬
ُ ‫ك‬
‫ثِِف‬ َ ‫اس‬
‫ِِفي أ‬َ َّ ‫ع‬
‫ِالن‬ ُ ‫ف‬
َ ‫ماِيَ أن‬ َّ َ‫وأ‬
َ ِ‫ما‬ َ
{ 129 }

A. Offering open sessions for pregnant adolescents and anyone else who wants to
attend.
B. Designing posters that girls can view individually in community Nursing centre.
C. Preparing group class sessions for teaching pregnant adolescents together.
D. Conducting one to one teaching sessions for both mothers and daughters.
Answer: C
530) Child 2 years old has acute otitis media. What is the sign while the nurse observing
child, confirm the diagnosis??
A. Otorrhea
B. Roll head side to side
Answer: B

531) Pregnant women are very tired. She is feeling anxious and anxiety for her baby.
What is describe woman situation according to Maslow hierarchy??

A. Self esteem

ِ‫يِاْلَ أرض‬
‫أ‬ ُ ‫َم‬
ُ ‫ك‬
‫ثِِف‬ َ ‫اس‬
‫ِِفي أ‬َ َّ ‫ع‬
‫ِالن‬ ُ ‫ف‬
َ ‫ماِيَ أن‬ َّ َ‫وأ‬
َ ِ‫ما‬ َ
{ 130 }

B. Safety
C. Physiological needs
D. Love and belonging
Answer: B
532) An 11-year-old child has been diagnosed with type 1 diabetes mellitus. which of
the following education?
should the nurse explain about the exercise?
A. Extra insulin is requiring during exercise.
B. Extra snacks are needed before exercise.
C. Exercise will increase blood glucose.
D. Exercise should be restricted.
Answer: B
533) A nurse is preparing a teaching session for elementary school students about the
safety measures of
hazards. Which of the following is the most appropriate topics for safety measure for
children at this age?
A. Sport activities
B. Riding bicycle
C. Swimming
D. Driving
Answer: B

ِ‫يِاْلَ أرض‬
‫أ‬ ُ ‫َم‬
ُ ‫ك‬
‫ثِِف‬ َ ‫اس‬
‫ِِفي أ‬َ َّ ‫ع‬
‫ِالن‬ ُ ‫ف‬
َ ‫ماِيَ أن‬ َّ َ‫وأ‬
َ ِ‫ما‬ َ
{ 131 }

534) A 28-week pregnant mother is in the antenatal clinic with the complaint of vaginal
irritation itching and thick
white secretion. she is diagnosed as having vaginal candidiasis she had been having
the same infection before
the pregnancy for which she was prescribed the vaginal cream she is using the same
cream for the past few days.
but her symptoms are not relieved what need to be emphasized to mother.
A. change clothes daily use sanitary napkins
B. Avoid self-treatment and seek doctor advise.
C. Use medicated bubble baths and reduce activates.
D. Was with warm water every two hours.
Answer: B
535) A prim gravida woman who is pregnant at 30 weeks' gestation told the nurse that
she is worried that anything
happens to her baby. Which of the following should be the proper nurse's response?
A. Ask the woman not to worry.
B. Ask the woman to express her concerns.
C. Attract the woman's attention to other issue.

ِ‫يِاْلَ أرض‬
‫أ‬ ُ ‫َم‬
ُ ‫ك‬
‫ثِِف‬ َ ‫اس‬
‫ِِفي أ‬َ َّ ‫ع‬
‫ِالن‬ ُ ‫ف‬
َ ‫ماِيَ أن‬ َّ َ‫وأ‬
َ ِ‫ما‬ َ
{ 132 }

D. Reassure the woman about the baby condition.


Answer: B
536) The nurse is caring for an adult patient who is admitted to the hospital for fever and
chills. The nurse repeatedly finds a temperature of 40° C (104° F) in the morning and
38.9° C (102°F) at night. What does the nurse infer about the fever pattern?
A. Intermittent Fever

B. Remittent fever✅✅

C. Relapsing fever
D. Continuous fever
Answer: B
537) The nurse notes that the patient has been experiencing febrile episodes lasting
more than 24 hours interrupted by periods of normal body temperature that also last
than 24 hours. What does the nurse infer about the patient's fever pattern?
A. Intermittent Fever
B. Remittent fever
C. Relapsing fever✅✅

D. Continuous fever
Answer: C
538)A 10-year-old child is brought to the hospital with high fever and chills. The nurse
records the vital signs and finds that her temperature is 104° F (40° C), blood pressure is
130/85 mm Hg, and pulse rate is 120/min. The fever remains mostly high but is
interspersed with periods of normal body temperature. What pattern of fever does the
child have?
A. Sustained

B. Intermittent✅✅

C. Remittent
D. Relapsing
Answer: B
539) Medical definition of Otic route for drug?
A. In the nose

ِ‫يِاْلَ أرض‬
‫أ‬ ُ ‫َم‬
ُ ‫ك‬
‫ثِِف‬ َ ‫اس‬
‫ِِفي أ‬َ َّ ‫ع‬
‫ِالن‬ ُ ‫ف‬
َ ‫ماِيَ أن‬ َّ َ‫وأ‬
َ ِ‫ما‬ َ
{ 133 }

B. In the ear✅

C. In the eye
Answer: B
540) The patient with CRF should perform peritoneal dialysis at home. The nurse is
learning patient and instructing the client to warm the dialyzing solution to 37 degrees
Celsius. Why should the patient warm the solution before using??
A. Dilate the peritoneal blood vessels✅✅

B. Maintain a constant body temperature.


C. Remove toxins from the body's cells.
D. Relax the abdominal muscles.
Answer: A
541) Patient with chronic renal failure before performing peritoneal dialysis the nurse
should warm the solution before administering to:
A. Promote abdominal muscle relaxation.
B. Maintain extra body warmth temperature.
C. Encourage the removal of serum urea ✅✅

D. Stimulate potassium back into body cells.


Answer: C
542)The nurse is preparing a preoperative client for transfer to the operating room. The
nurse should take which of the following actions in the care of this client at this time?
a) Ensure that the client has voided
b) Administer all the daily medications
c)Have the client practice postoperative breathing exercises.
d)Verify that the client has not eaten for the last 24 hours.
Answer: A
543) A 6-month-old infant receives a diphtheria, tetanus, and acellular pertussis (DTAP)
immunization at a well-baby clinic. The mother returns home and calls the clinic to
report that the infant has developed swelling and redness at the site of injection. A
nurse tells the mother.
a) Monitor the infant for a fever

ِ‫يِاْلَ أرض‬
‫أ‬ ُ ‫َم‬
ُ ‫ك‬
‫ثِِف‬ َ ‫اس‬
‫ِِفي أ‬َ َّ ‫ع‬
‫ِالن‬ ُ ‫ف‬
َ ‫ماِيَ أن‬ َّ َ‫وأ‬
َ ِ‫ما‬ َ
{ 134 }

b) Bring the infant back to the clinic


c) Apply a hot pack to the injection site
d) Apply an ice pack to the injection site
Answer: D
544) A nurse hears a client calling out for help, hurries down the hallway to the client's
room, and finds the client lying on the floor. The nurse performs a thorough assessment,
assists the client back to bed, notifies the physician of the incident, and completes an
incident report. Which of the following should the nurse document on the incident
report?
A) The client fell out of bed
B) The client climbed over the side rails
C) The client was found lying on the floor
D) The client became restless and tried to get out of bed
Answer: C
545) A nurse is monitoring a 3-year-old child for signs and symptoms of increased
intracranial pressure (ICP) after a craniotomy. The nurse plans to monitor for which early
sign or symptom of increased ICP?
a) Excessive vomiting
b) Bulging anterior fontanel
c) Increasing head circumference
d) Complaints of a frontal headache
Answer: A
546) The nurse is monitoring a client in the Medical- Surgical Unit who is recovering
overnight following a femoral angioplasty and observes that his leg has become pale
and cool to the touch. After paging the health care provider for an emergency
evaluation, the nurse receives an angry telephone call asking why this department
always calls at 2 AM with ridiculous questions. Which of the following is the most
appropriate response?
A. "how would you feel if it was your father not getting the medical care, he needed."
B. "I am worried that this man may lose his leg unless something is done immediately."
C. "your behavior at this time is extremely unprofessional, and I will be making a formal
report to my supervisor."

ِ‫يِاْلَ أرض‬
‫أ‬ ُ ‫َم‬
ُ ‫ك‬
‫ثِِف‬ َ ‫اس‬
‫ِِفي أ‬َ َّ ‫ع‬
‫ِالن‬ ُ ‫ف‬
َ ‫ماِيَ أن‬ َّ َ‫وأ‬
َ ِ‫ما‬ َ
{ 135 }

D. I am sorry, but it is hospital policy to report any post-operative complications to the


health care provider on call. May I ask what's has been bothering you about this
department's pages”?
Answer: B
547) A newborn admitted to the NICU with tracheoesophageal fistula (TOF). Which of
the following nursing intervention should be included?
A. Elevate the head for feeding.
B. Elevating the head but keep the child NPO.
C. Insert a nasogastric tube for feeding.
D. Encourage the mother to breast feed.
Answer: B
548) A pregnant mother has been in labor for three hours with her membranes
spontaneously ruptured. The nurse observed that the liquor amnl is meconium stained.
Which of the following is the nurse's best interpretation?
A. Fetal hypoxia
B. Low-lying placenta
C. Intrauterine infection
D. It is mixed with maternal urine.
Answer: A
549) Which of the following psychiatric term best describes this symptom "While talking,
the psychiatric patient repeated the same words stated by the nurse"?
A. Verbigeration.
B. Echolalia. ✅✅
C. Echopraxia.
D. Preservation.
Answer: B
550) When the psychiatric patient repeats his words boat, boat, boat. What is the best
term describing the patient action?
A. Echolalia
B. Palilalia ✅

ِ‫يِاْلَ أرض‬
‫أ‬ ُ ‫َم‬
ُ ‫ك‬
‫ثِِف‬ َ ‫اس‬
‫ِِفي أ‬َ َّ ‫ع‬
‫ِالن‬ ُ ‫ف‬
َ ‫ماِيَ أن‬ َّ َ‫وأ‬
َ ِ‫ما‬ َ
{ 136 }

C. Neologism
D. Ward Salad
Answer: B
551) A nurse is interviewing a client with schizophrenia when the client begins to say,
"Kite, night, right, height, fright." The nurse documents this as:
A. Verbigeration.
B. Stilted language.
C. Clang association.
D. Neologisms.
Answer: C
552) Which of the following is the best contraceptive for mother who is breastfeeding
her baby?
A. Combined oral contraceptives.
B. Contraceptive Patches
C. Estrogen only pills contraceptive
D. Progesterone only "mini" pills
Answer: D
553) A 2-year-old child is admitted to the pediatric unit with a diagnosed pneumonia.
Which of the following intervention would be a nursing priority?
A. Encourage coughing
B. Encourage exercise
C. Perform postural drainage✅✅

D. Avoid food high in carbohydrates.


Answer: C
554) A Client who underwent surgical procedure the preceding day normal assessment
with an oral body temperature of 37.5 0800 hours. If the condition remains stable the
client is to be in the afternoon. which of the following is the highest priority intervention?
A. Observation
B. Inform the surgeon

ِ‫يِاْلَ أرض‬
‫أ‬ ُ ‫َم‬
ُ ‫ك‬
‫ثِِف‬ َ ‫اس‬
‫ِِفي أ‬َ َّ ‫ع‬
‫ِالن‬ ُ ‫ف‬
َ ‫ماِيَ أن‬ َّ َ‫وأ‬
َ ِ‫ما‬ َ
{ 137 }

C. Administer a dose of aspirin.


D. Ensure proper use of the incentive spirometer.
Answer: A
555) For a patient with a colostomy, which of the following-intervention
is appropriate for preventing the risk of the impaired skin related to
exposure excretions?
A. Empty pouch when it is completely full
B. Remove the skin barrier inspect the skin monthly
C. Recaps Skin barrier opening to size of stoma with each
change
D. Cut an opening in the skin barrier then the circumference of
the stoma ✅
Answer: D
556) Mastitis is an infection of the breast that occurs most often 2-4 after childbirth.
Which of the following is considered first line treatment of mastitis?
A. Drainage of breast abscess
B. Antibiotic therapy and cessation of breast feeding
C. Antibiotic therapy and continuation of breastfeeding
D. Advise mother to stop breastfeeding until infection is clean.
Answer: C
557) A new community nurse was caring for a long - term disabled patient. The patient
said, " I prefer to have my care given this way “. the nurse explained more alternative
approaches were clinically more effective. However, the patient was Insisting on his
opinion and said, " I am more comfortable if my care is given this way " What should the
nurse do in this case to manage the situation?
A. accept that the patient knows best what works for him.
B. review the recent research with the patient to convince him.
C. give evidence-based care according to current findings.
D. Compromise between patients
preference and research approach of care
Answer: D

ِ‫يِاْلَ أرض‬
‫أ‬ ُ ‫َم‬
ُ ‫ك‬
‫ثِِف‬ َ ‫اس‬
‫ِِفي أ‬َ َّ ‫ع‬
‫ِالن‬ ُ ‫ف‬
َ ‫ماِيَ أن‬ َّ َ‫وأ‬
َ ِ‫ما‬ َ
{ 138 }

558) There is psychiatric patient, every time says to a nurse, “You are my sister, always
calling her my sister,” and the nurse replied, “Your sister is not here". What should the
nurse use from therapeutic communication?
A. Placing event in time or sequence
B. Presenting reality
C. Refer to reality or Realization✅
Answer: C
559) After repairing for child with cleft lip left side. Which of the following position should
the nurse put the baby to prevent Aspiration?
1- Prone
2- On stomach
3- Right lateral
4- Left lateral
Answer: C
560) When assessing neonate after delivery, the nurse notes the infant have caput
succedaneum. What is the nurse should do?
A. Code blue
B. Order to physician
C. Observation only nothing to do.
Answer: C
561) What is the function of serotonin neurotransmitter receptors?
A. Motor
B. Memory
C. Inhibit pain
Answer: B
Serotonin receptor ---- Mood + memory
Dopamine - - - - - - Motor
562) The child diagnosed with anemia and what should the nurse instruct about the
food should be Limited and avoided for child???
A. Apple juice

ِ‫يِاْلَ أرض‬
‫أ‬ ُ ‫َم‬
ُ ‫ك‬
‫ثِِف‬ َ ‫اس‬
‫ِِفي أ‬َ َّ ‫ع‬
‫ِالن‬ ُ ‫ف‬
َ ‫ماِيَ أن‬ َّ َ‫وأ‬
َ ِ‫ما‬ َ
{ 139 }

B. Orange juice
C. Lemon juice
D. Chocolate✅

Answer: D
563) Pregnant mother check the baby’s movements every day for an hour, she always
feels among this hour baby's movement 10 times or more. Once time while she was
checking movement, she wasn't feel any movement of the baby for an hour, what
should the nurse do?
A. Instruct her to measure again the kick movement in another

hour✅

B. Measure again movement on the next day


C. Ask her to come in clinic quickly to do a non-stress test
Answer: A
564)A patient with pre-eclampsia is admitted to the unit with an order for magnesium
sulfate. The nurse will understand that the therapy is effective if?
A. Scotomas are present
B. Ankle clonus is increased
C. No seizures occur
D. Blood pressure drops
Answer: C
565) The nurse was endorsing patient on TPN. The doctor prescribed to a certain rate,
after 2 hours from endorsement the nurse checked the TPN rate it was in low rate than
the prescribed the solution was slowly. What should the nurse do?
A. Ask doctor to write again new rate
B. Return the rate to the prescribed rate
C. Keep the rate then adjust the rate to prescribed from the next bottle✅

D. Increase the rate immediately


Answer: C
566)What is the food that a pregnant woman should not eat because it can lead to
infection for her??
A. Pasteurized milk

ِ‫يِاْلَ أرض‬
‫أ‬ ُ ‫َم‬
ُ ‫ك‬
‫ثِِف‬ َ ‫اس‬
‫ِِفي أ‬َ َّ ‫ع‬
‫ِالن‬ ُ ‫ف‬
َ ‫ماِيَ أن‬ َّ َ‫وأ‬
َ ِ‫ما‬ َ
{ 140 }

B. Unprocessed cheese✅

C. Processed cheese
D. Yogurt
Answer: B
567) The patient with a kidney transplantation, then he came after few days from
discharge complaining from fever, tenderness and edema. What should the nurse
expect??
A. UTI
B. Organ rejection✅

Answer: B
568) What position should the nurse place the head of the bed for heart failure patient
with jugular vein distention?
A. High fowler's
B. Raised 10 degrees

C. Raised 30 degrees Semi fowler ✅

D. Supine position
Answer: C

569) What is the priority patient for triage nurse to assess?


A. Patient with Sever chest pain
B. Patient with Sever bleeding✅

Answer: B
570) Pregnant woman came to ER with rupture of membrane. Why should the nurse
Limit vaginal examination for her?
A. Prevent risk of infection✅

B. Avoid bleeding
C. Prevent further loss of membrane fluid
D. Prevent fetal hypoxia

ِ‫يِاْلَ أرض‬
‫أ‬ ُ ‫َم‬
ُ ‫ك‬
‫ثِِف‬ َ ‫اس‬
‫ِِفي أ‬َ َّ ‫ع‬
‫ِالن‬ ُ ‫ف‬
َ ‫ماِيَ أن‬ َّ َ‫وأ‬
َ ِ‫ما‬ َ
{ 141 }

Answer: A
571) The patient has joint arthroplasty surgery. The patient with urinary catheter and IV
line is attached. He was on oxygen, what needs immediate intervention?
A. Pain, redness, and swelling

B. Shortness of breath and coughing✅

Answer: B
572) The nurse was doing research with two groups of smokers, the first group took
Nicotine Patch and the second group not acheived the target goal, what is the
research type?
A. Case Study
B. Cohort study✅

Answer: B
573) The psychiatric patient, She was speaking and laughing in a loud voice, She was
wearing a brightly-colored dress and an inconsistent make up, and She had a history of
depression before, what is her diagnosis?
A. Major depression
B. Bipolar disorder I✅
C. Bipolar disorder ll
D. Cyclothymic disorder
Answer: B
574) Woman came to ER with buffy eye (swelling)her eye disappeared she was pale
and yellow, hair loss. She was very exhausted. What should the nurse ask her?
A. Family history for same symptoms
B. Check nails and ask if she expose to any chemical substance✅

Answer: B
575) The pregnant woman is asking the nurse about the function of placenta. The nurse
explains to her that placenta provide your baby with food and Oxygen. Which another
additional function for placenta??
A. Cushion and protect baby.
B. Produce HCG
C. Produce contraction

ِ‫يِاْلَ أرض‬
‫أ‬ ُ ‫َم‬
ُ ‫ك‬
‫ثِِف‬ َ ‫اس‬
‫ِِفي أ‬َ َّ ‫ع‬
‫ِالن‬ ُ ‫ف‬
َ ‫ماِيَ أن‬ َّ َ‫وأ‬
َ ِ‫ما‬ َ
{ 142 }

Answer: B
576) The size of uterus at the end of pregnancy
A. 900 g
B. 1000 g
C. 1500 g
Answer: A
577) The uterus size after delivery:
A. 900 g
B. 1000 g
C. 1500 g
Answer: B
578) What is the side effect of baclofen if given for diabetic patient?
A. Blurred vision.
B. Increase of insulin demand
C. Fever
Answer: B
579) The patient with SLE (systemic lupus erythematous). The patient is complaining skin
rash. What are the factors affecting and cause the rash??
A. Skin rash after exposure of sun
B. Skin rash increase in winter season
Answer: A
580) A 45-year-old woman presents with a generalized rash that is not itchy. She reports
that she has had the problem for the past 15 years. Examination reveals a well-outlined,
reddish plaque over the right gluteal fold. The plaque has scales over it and is cracked
in some areas Which of the following interventions is the most appropriate?
A. Apply topical cream to the affected area.
B. Expose area to sunlight for twenty minutes' daily
C. Maintain immunosuppressant therapy regimen.
D. Increase dietary intake of vitamin A

ِ‫يِاْلَ أرض‬
‫أ‬ ُ ‫َم‬
ُ ‫ك‬
‫ثِِف‬ َ ‫اس‬
‫ِِفي أ‬َ َّ ‫ع‬
‫ِالن‬ ُ ‫ف‬
َ ‫ماِيَ أن‬ َّ َ‫وأ‬
َ ِ‫ما‬ َ
{ 143 }

Answer: C
581) A patient with SLE (systemic lupus erythematous) report
decreased urinary output during the past 2-4 days and chest pain.
that is aggravated by breathing and coughing. The patient vital signs
remain within the baseline normal range s1 and s2 are present with
audible friction rub. Which of the following statement would be?
appropriate for the nurse to make?
a. It sounds like SLE is being well controlled.
b. I need to get some nitroglycerine for your chest pain.
c. There may be some inflammation surrounding your heart.
d. Your symptoms may be due to a urinary tract infection.
Answer: C
582) Which hormone is release from placenta?

A. HCG✅

B. Progestron
C. Estrogen
Answer: A
583) A patient with bowlegs due to abnormal bone formations and deformities has
calcium level of 7.5 mg/100ml.
Which of the following foods would the nurse most likely instruct the patient to
add to a diet? 3
a- Organ meats
b- Whole grains
c- Egg yolks
d - Chicken meat
Answer: C
584) According to Maslow hierarchy patient with schizophrenia has lack of motivation
to see herself. What are the needs for patient that lost??

ِ‫يِاْلَ أرض‬
‫أ‬ ُ ‫َم‬
ُ ‫ك‬
‫ثِِف‬ َ ‫اس‬
‫ِِفي أ‬َ َّ ‫ع‬
‫ِالن‬ ُ ‫ف‬
َ ‫ماِيَ أن‬ َّ َ‫وأ‬
َ ِ‫ما‬ َ
{ 144 }

A. Physiological need
B. Self esteem
C. Love and belonging
D. Safety
Answer: B
585) Patient with leukemia which blood component most affected?
A. RBCs

B. WBC✅

C. Plasma
D. Platelet
Answer: B
586) Which of the following term the nurse used for explaining the normal respiratory
rhythm and depth in a patient??
A- Apnea
B- Eupnea ✅
C- Brydpnea
D-Tachypnea
Answer: B
587) 22 years old girl people advise her that she has voice is very bad. she joined in
Academic music institution. which defense mechanism she used?
A) Denial
B) displacement
C) projection
D)repression
Answer: C
588) Patient with chest tube drainage. What is the most appropriate nursing diagnosis?
A. High risk for infection✅

B. Disturbed body image


Answer: A

ِ‫يِاْلَ أرض‬
‫أ‬ ُ ‫َم‬
ُ ‫ك‬
‫ثِِف‬ َ ‫اس‬
‫ِِفي أ‬َ َّ ‫ع‬
‫ِالن‬ ُ ‫ف‬
َ ‫ماِيَ أن‬ َّ َ‫وأ‬
َ ِ‫ما‬ َ
{ 145 }

589) When messure Blood pressure for adult patient with using child cuff. What is the
expected result?
A. False high reading✅

B. False low reading


C. Normal reading
Answer: A
590) When messure Blood pressure for child patient with using adult cuff. What is the
expected result?
A. False high reading

B. False low reading✅

C. Normal reading
Answer: B
591) The nurse was providing Education about oral Contraceptives pill for mothers What
should the nurse tell ?

A. If you take antibiotic drug change oral contraceptives ✅

B. if you forget one or more pills, take for a week two pills daily
Answer: A
592) The nurse was thinking that doctor forgot to write one of drug prescriptions. Then
later she discovered that doctor wrote it and she didn't give medication to the patient.
To reduce malpractice What should the nurse do??
A. Ask the doctor write again the medication
B. Write Incident report ✅

Answer: B
593)Bedridden PT nurse is doing pathing for him and she used lotion after bathing to
prevent pressure ulcer for patient. What is appropriate action??
A. Dry patient after bathing

B. Use Air mattress ✅


C. Keep the sheet loosely.
Answer: B

ِ‫يِاْلَ أرض‬
‫أ‬ ُ ‫َم‬
ُ ‫ك‬
‫ثِِف‬ َ ‫اس‬
‫ِِفي أ‬َ َّ ‫ع‬
‫ِالن‬ ُ ‫ف‬
َ ‫ماِيَ أن‬ َّ َ‫وأ‬
َ ِ‫ما‬ َ
{ 146 }

594) The mother was coming to give her baby vaccination. Then she asked the nurse
about her baby that he has chronic constipation and defecate ribbon like stool. The
stool after bass is bad smell. Which of the following questions should the nurse ask the
mother??
A. What are the fluids is provide to your baby this week?
B. What is the fiber diet do you give to your baby?
C. Have your baby delayed passing meconium after delivery?
Answer: C
595)What is the. Advantage of Nursing documentation system??
a. Cost for hospitals
b. Spent time with patient.
c. Decrease of nurse’s satisfaction
D. Decrease of documentation quality
Answer: B
596) The patient with cancer. He is on I. V infusion the nurse observed there is
Extravasation. What should the nurse do?
A. Stop the Infusion immediately ✅

B. Slow the rate


C. Report the doctor
Answer: A
597)The patient is post-operative. After the nurse taking the vital signs what should the
nurse assess next??
A. pain assessment
B. incision assessment
C. Glasgow assessment
Answer: B
598) Patient after laryngectomy and mastoidectomy. What is the priority nursing
diagnosis?
A. Impaired verbal communication

ِ‫يِاْلَ أرض‬
‫أ‬ ُ ‫َم‬
ُ ‫ك‬
‫ثِِف‬ َ ‫اس‬
‫ِِفي أ‬َ َّ ‫ع‬
‫ِالن‬ ُ ‫ف‬
َ ‫ماِيَ أن‬ َّ َ‫وأ‬
َ ِ‫ما‬ َ
{ 147 }

B. Airway clearance ✅

Answer: B
599) Physician order three times the normal dose (triple dose) - when the nurse calls
him, he insists on giving the ordered medication - what does the nurse do?
a. Call pharmacy
b. Call another doctor.
C. Withhold medication and inform charge nurse.
D. Withhold medication and inform medical office.
Answer: C
600) Patient was complaining sever pain. The nurse gave to him I. V morphine but there
is no effect with him. What should the nurse do?

A. Ask the patient for drug abuse✅


B. Give another dose
C. Ignore the patient
Answer: A
601) A patient is one-day post-operative repair of a large umbilical hernia.
The patient complains of abdominal pain and described feeling the
sutures give way. Upon assessment of the abdomen the nurse
observes an evisceration. The nurse’s IMMEDIATE response should
be to:
A. Medicate the patient for pain
B. Instruct the patient to cough hard
C. Have the patient perform the Valsalva maneuver

D. Cover the abdomen with a sterile soaked dressing✅

Answer: D
602)the surgery and insisting on the nurse to explain to explain the procedure. Which of
the following nursing?
intervention is the most appropriate?

ِ‫يِاْلَ أرض‬
‫أ‬ ُ ‫َم‬
ُ ‫ك‬
‫ثِِف‬ َ ‫اس‬
‫ِِفي أ‬َ َّ ‫ع‬
‫ِالن‬ ُ ‫ف‬
َ ‫ماِيَ أن‬ َّ َ‫وأ‬
َ ِ‫ما‬ َ
{ 148 }

A. Assess anxiety level.


B. Assure he will be better after surgery.
C. Call doctor to Explain surgery.
D. Give patient some sedative medication
Answer: C
603)New nurse during orientation period she feel that her preceptor isolated from her.
What should she do?
A-Ask questions as much as she can ✅✅

B-Ask to increase orientation days.


C- came to the work late.
D. Ask preceptor for important summary lectures in Nursing.
Answer: A
604) woman patient admitted with indwelling Foley catheter with a closed drainage
system was order collection for sterile urine specimen. Which of the following steps is
considered the best way to collect the sterile urine specimen?
A-Obtain the specimen from the drainage bag
B-Obtain the specimen from the aspiration port✅
C-Obtain the specimen from the first voiding in the morning
D-Obtain a clean mid-stream catch cleaning the perineum
Answer: B
605) Which of the following lab. test must be
done on a patient with major burns, prior to administration of antibiotics??
A. complete blood account
B. wound culture
C. type and cross match
D. sensitivity studies✅

Answer: D
606) Couple asked the nurse which of the first investigation they should do for Infertility.
Which of the following should be the proper a nurse answer?
A. Hysterosalpingogram

ِ‫يِاْلَ أرض‬
‫أ‬ ُ ‫َم‬
ُ ‫ك‬
‫ثِِف‬ َ ‫اس‬
‫ِِفي أ‬َ َّ ‫ع‬
‫ِالن‬ ُ ‫ف‬
َ ‫ماِيَ أن‬ َّ َ‫وأ‬
َ ِ‫ما‬ َ
{ 149 }

B. Serum progesterone

C. Semen analysis ✅

D. Endometrial biopsy
Answer: C
607) The patient came to ER with second degree of burn on his dorsal left arm and
dorsal of both leg TBSA. The patient weight is 62 Kg. What is the total intravenous fluids
the patient need?

A. 5580 ✅

B. 6580
C. 6680
D. 7680
Answer: A
608) Child with VSD. What is blood flow characteristic?
A- mixes atrium blood
B-decreased pulmonary blood flow

C-increase pulmonary blood flow ✅


Answer: C
610) The community nurse was instructing for two partners will marry and advise them
about health life and obligations. What is this considering?
A. Counseling
B. Education✅

C. Screening
Answer: B
611) How we can reduce chronic cases in the community?
A. Evaluate the strategies with outcomes
B. Monitor risk in each family members ✅

Answer: B

ِ‫يِاْلَ أرض‬
‫أ‬ ُ ‫َم‬
ُ ‫ك‬
‫ثِِف‬ َ ‫اس‬
‫ِِفي أ‬َ َّ ‫ع‬
‫ِالن‬ ُ ‫ف‬
َ ‫ماِيَ أن‬ َّ َ‫وأ‬
َ ِ‫ما‬ َ
{ 150 }

612) Patient with abdominal cancer. During diagnosis we cannot examine accurately
and can't detect. What should use for him?
A. laparoscopy
B. MRI✅

Answer: B
613) For proper evaluation actions. What should the manager use?
A. field
B. group✅
Answer: B
Dexamethasone function >>>promote lung Maturation.
614) Which of the following is the most affected drug for The psychiatric patient and
cause more problem?
A. Atypical antipsychotic

B. Typical antipsychotic because it has side effects more than atypical ✅


C. Serotonin inhibitor
D. Dopamine inhibitor
Answer: B

615) When the nurse knows the strengths and weaknesses point in her personality. What
is consider?
A. Self-awareness
B. Self-understanding
C. Self-concept✅

Answer: C
616) Getting birth for mother who has gestational diabetes. How will this affect delivery
process?
A. It will be Easy

ِ‫يِاْلَ أرض‬
‫أ‬ ُ ‫َم‬
ُ ‫ك‬
‫ثِِف‬ َ ‫اس‬
‫ِِفي أ‬َ َّ ‫ع‬
‫ِالن‬ ُ ‫ف‬
َ ‫ماِيَ أن‬ َّ َ‫وأ‬
َ ِ‫ما‬ َ
{ 151 }

B. It will be Difficult ✅

C. It will be fast
D. It will be slow
Answer: B
617)client is diagnosed as having secondary Cushing’s syndrome. The nurse knows that
the client has most likely been taking which medication:
A. Estrogen
B. Penicillin
C. Lovastatin
D. Prednisone
Answer: D
618) A 7-year-old child is assessed with a urinary tract infection BP 99/55 TEMP 36.4.
which of the following is?
the most appropriate nursing action?
A. Notify the physician immediately.
B. Notify the nurse manager immediately.
C. Document the findings in the nurse note
D. Put the patient in Trendelenburg position.
Answer: C
619)Patient with decrease Glomerular filtration rate (GFR) and small kidney size 8cm,
creatinine 8.5 mg/dl. What is the appropriate treatment? 1-nephrotomy
2-nephroctomy
3-heamodialysis
Answer: C
620) Which of the following assess effective liver and consider liver function test?
A. Creatinine
B. BUN
C. Urea

ِ‫يِاْلَ أرض‬
‫أ‬ ُ ‫َم‬
ُ ‫ك‬
‫ثِِف‬ َ ‫اس‬
‫ِِفي أ‬َ َّ ‫ع‬
‫ِالن‬ ُ ‫ف‬
َ ‫ماِيَ أن‬ َّ َ‫وأ‬
َ ِ‫ما‬ َ
{ 152 }

D. Urine gravity
Answer: B
621)The nurse is preparing to give injection to patient. The patient refused to get
medication. Then the nurse forced him and gave the injection to him aggressively.
What is the description of nurse action?
A. Assault
B. Battery
C. Abuse
D. Malpractice
Answer: B
622)The patient asked nurse to check his temperature then the nurse ignore the patient
and she did not give him care?
A. Malpractice
B. Negligence
C. Slander
Answer: B
623)Hospital starts in implementation of EHR.
EHR (Electronic health record). What is the first step before implementation the EHC
program?
A. Set plan for implementation program
B. Check workflow
C. Check provided supplies for EHC.
D. Explain for workers EHC concept and program.
Answer: D
624)How to prevent heat loss via evaporation for neonate after delivery?
A. Avoid exposure to air draft.
B. Avoid contact to wall.
C. Dry neonate and cover him, avoid any cold objects.
D. Warm any equipment's before touching neonate.

ِ‫يِاْلَ أرض‬
‫أ‬ ُ ‫َم‬
ُ ‫ك‬
‫ثِِف‬ َ ‫اس‬
‫ِِفي أ‬َ َّ ‫ع‬
‫ِالن‬ ُ ‫ف‬
َ ‫ماِيَ أن‬ َّ َ‫وأ‬
َ ِ‫ما‬ َ
{ 153 }

Answer: C
625)Woman has experienced recurrent vaginal infection and she is asking the nurse
how to prevent the infection. What is the response if she says the nurse expect she need
further instructions and consider alter preventive measures that kill good bacteria?
A. Wear cotton untied underwear
B. Wipes front to back each toilet.
C. Wear condom to prevent infection.
D. Avoid douche and perfume spray.
Answer: C
626)What is the Position after thyroidectomy??
A. Lateral flexed
B. Semi fowler with slight neck flexed.
C. Prone head extended.
D. High Fowler with neck extended.
Answer: B
627)What is the Position during thyroidectomy??
A. Supine with hyper flexion of neck
B. Supine with hyperextended neck
C. Semi fowler
D. Lateral with slightly flexion neck
Answer: B
628)Which of the following category of CTG?? See image.
A. Normal
B. Category 1
C. Category 2
D. Category 3

ِ‫يِاْلَ أرض‬
‫أ‬ ُ ‫َم‬
ُ ‫ك‬
‫ثِِف‬ َ ‫اس‬
‫ِِفي أ‬َ َّ ‫ع‬
‫ِالن‬ ُ ‫ف‬
َ ‫ماِيَ أن‬ َّ َ‫وأ‬
َ ِ‫ما‬ َ
{ 154 }

Answer: A

629) Which of the following category of CTG?? See image👆👆👆

A. Normal
B. Category 1
C. Category 2
D. Category 3
Answer: D

ِ‫يِاْلَ أرض‬
‫أ‬ ُ ‫َم‬
ُ ‫ك‬
‫ثِِف‬ َ ‫اس‬
‫ِِفي أ‬َ َّ ‫ع‬
‫ِالن‬ ُ ‫ف‬
َ ‫ماِيَ أن‬ َّ َ‫وأ‬
َ ِ‫ما‬ َ
{ 155 }

630)Which of the following category of CTG?? See image. 👆👆👆


A. Normal
B. Category 1
C. Category 2
D. Category 3
Answer: B

631)Which of the following category of CTG?? See image. 👆👆👆

A. Normal
B. Category 1
C. Category 2

ِ‫يِاْلَ أرض‬
‫أ‬ ُ ‫َم‬
ُ ‫ك‬
‫ثِِف‬ َ ‫اس‬
‫ِِفي أ‬َ َّ ‫ع‬
‫ِالن‬ ُ ‫ف‬
َ ‫ماِيَ أن‬ َّ َ‫وأ‬
َ ِ‫ما‬ َ
‫} ‪{ 156‬‬

‫‪D. Category 3‬‬


‫‪Answer: D‬‬

‫شرح ‪CTG Categories‬‬

‫يِاْلَ أرضِ‬
‫أ‬ ‫َم ُ‬
‫ك ُ‬
‫ثِِف‬ ‫اس َ‬
‫ِِفي أ‬‫َ‬ ‫ع َّ‬
‫ِالن‬ ‫ف ُ‬
‫ماِيَ أن َ‬ ‫وأَ َّ‬
‫ماِ َ‬ ‫َ‬
‫} ‪{ 157‬‬

‫يِاْلَ أرضِ‬
‫أ‬ ‫َم ُ‬
‫ك ُ‬
‫ثِِف‬ ‫اس َ‬
‫ِِفي أ‬‫َ‬ ‫ع َّ‬
‫ِالن‬ ‫ف ُ‬
‫ماِيَ أن َ‬ ‫وأَ َّ‬
‫ماِ َ‬ ‫َ‬
{ 158 }

632)Category 2 C II C2

Variable decelerations 👆👆

Vitamin A deficiency cause?

Night Blindness
633) Low fat diet. Which of the following is?

A. Skimmed milk with yolk✅


B. Evaporated milk with whole egg
Answer: A

634)year-old man presents to the Emergency Department


complaints of breathing difficulties. He appears restless and reports.
that he has had a cough with thick green sputum for days. The nurse
performs auscultation over the lung fields.
Blood pressure 130/80 mmHg Heart rate 100 /min, irregular
Respiratory rate 24 /min, irregular Temperature 39.0C Which
clinical finding is most likely heard over the right lower?
A. Decreased vocal resonance.
B. Decreased fremitus
C. Tympani
D. Bronchial

ِ‫يِاْلَ أرض‬
‫أ‬ ُ ‫َم‬
ُ ‫ك‬
‫ثِِف‬ َ ‫اس‬
‫ِِفي أ‬َ َّ ‫ع‬
‫ِالن‬ ُ ‫ف‬
َ ‫ماِيَ أن‬ َّ َ‫وأ‬
َ ِ‫ما‬ َ
{ 159 }

Answer: D
635)A 15-month-old child is admitted for hypospadias repair. After surgery, which of the
following instruction
should be given to the parents?
A. Limit activity for 2 weeks
B. Avoid apply ointment or powder.
C. Give a child a diet that is high in protein.
D. Isolate the child from other children with the infection
Answer: A
636)A patient has a history of severe, uncontrolled epistaxis. The patient's blood.
pressure and platelet count are normal. To minimize the occurrence of bleeding
episodes the nurse should teach the patient to.
a- Sleep with the head elevated on at least two to three pillows.
b- Apply firm pressure to the nostrils four times a day.
c- Apply a water- soluble lubricant to the nasal septum twice daily.
d- Minimize the intake of caffeine and increase fluids intake.
Answer: C

ِ‫يِاْلَ أرض‬
‫أ‬ ُ ‫َم‬
ُ ‫ك‬
‫ثِِف‬ َ ‫اس‬
‫ِِفي أ‬َ َّ ‫ع‬
‫ِالن‬ ُ ‫ف‬
َ ‫ماِيَ أن‬ َّ َ‫وأ‬
َ ِ‫ما‬ َ
{ 160 }

637). 27 years old female brought to the Emergency Room accompanied by her
husband. He described that she
had marked Weight loss with episodes of emesis in the past three months. She is
diagnosed as having anorexia.
She reported feeling Febrile but had not measured her temperature. Her White Blood
Count was 11,000/mm3.
Which of the following most Likely describe her diagnostic criteria for her anorexia?
A. Restricting food intake
B. Fear of gaining weight.
C. Problems with body image
D. Binge eating disorder.
Answer: B
638) 11-month baby how many ml orally he can receive?
A. 200
B. 400
C. 800
D. 1200
Answer: B

ِ‫يِاْلَ أرض‬
‫أ‬ ُ ‫َم‬
ُ ‫ك‬
‫ثِِف‬ َ ‫اس‬
‫ِِفي أ‬َ َّ ‫ع‬
‫ِالن‬ ُ ‫ف‬
َ ‫ماِيَ أن‬ َّ َ‫وأ‬
َ ِ‫ما‬ َ
{ 161 }

639) A woman is at 30 weeks' gestational age admitted to antenatal with premature


rupture of membrane. the
nurse administered Dexamethasone to her according doctor prescription. She asks
what the Drug is for. Which of
the following the best answer.
A. To promote fetal lung maturation.
B. Prevention of chorioamnionitis
C. To increase uteroplacental exchange.
D. Treatment of fetal respiratory distress
Answer: A
640)A 71-year-old woman who resides in a long-term nursing home fell while walking
downstairs. The attending nurse arrives to find the patient sitting motionless on the stairs.
She is alert and oriented but wishes to rest. While she rests, the nurse reviews the chart
and notes that her medication regimen includes metformin, loratadine, warfarin and
diclofenac. Which medication is most likely to increase the patient's risk of injury?
A. Metformin
B. Loratadine
C. Warfarin
D. Diclofenac
Answer: B
641)A nurse is giving discharge planning instructions to the parents of a one-year-old
child acute otitis media
which of the following discharge instruction takes priority?
A. Administrate antibiotics as prescribed.
B. Administer influenza vaccination.
C. Breast feeding as long as possible.
D. Continue using of pacifier.
Answer: A
642)Which of the following is the primary level of prevention?
A. Detect and treat existing disease.

ِ‫يِاْلَ أرض‬
‫أ‬ ُ ‫َم‬
ُ ‫ك‬
‫ثِِف‬ َ ‫اس‬
‫ِِفي أ‬َ َّ ‫ع‬
‫ِالن‬ ُ ‫ف‬
َ ‫ماِيَ أن‬ َّ َ‫وأ‬
َ ِ‫ما‬ َ
{ 162 }

B. Prevent illness or injury occurring.


C. Reduce the extent and severity of health problem.
D. Minimize disability and restore to prevent function.
Answer: B
643)Patient with NGT and intermittent suction, suddenly the patient complaining of pain
and the abdomen is distended, at 10:00am NGT output=120 cc, 14:00 NGT output= 60
cc, what is the first action should be taken by the nurse?
A. Increase the suction
B. Administer pain medication.
C. Irrigate NGT to check patency.
D. Notify the physician
Answer: C
644)A 45-year-old woman presents to the Emergency Room and reported that she has
lost her husband due to lung cancer three months aga. She believes that she has colon
cancer, despite of negative results in repetitive and extensive diagnostic tests which
were negative. What is the most likely diagnosis?
A. Pain disorder
B. Hypochondriasis
C Depersonalization
D. Body dysmorphic disorder
Answer: B
645)Patient with ABG PH 7.33 ‫ و‬HCO3 30, PCO2 50?
A. Compensate respiratory
B. Compensate metabolic
C. Uncompensated respiratory
D. Uncompensated metabolic
Answer: C
646) Fastest and most sanitary thermometer is:
A. Glass

ِ‫يِاْلَ أرض‬
‫أ‬ ُ ‫َم‬
ُ ‫ك‬
‫ثِِف‬ َ ‫اس‬
‫ِِفي أ‬َ َّ ‫ع‬
‫ِالن‬ ُ ‫ف‬
َ ‫ماِيَ أن‬ َّ َ‫وأ‬
َ ِ‫ما‬ َ
{ 163 }

B. Electronic
C. Chemical
D. Tympanic
Answer: D
647)Considered as the most accessible and convenient method for
temperature taking is:
a. Oral
b. Rectal
c. Tympanic
d. Axillary
Answer: A
648)The nurse is performing a prenatal examination on a client in the third.
trimester. The nurse begins an abdominal examination that includes Leopold.
maneuvers. What information should the nurse be able to determine after?
performing the assessment’s first maneuver?
1. Fetal descent
2. Placenta previa
3. Fetal lie and presentation
4. Strength of uterine contractions
Answer: C
649)The nurse prepares the client for the removal of a nasogastric tube. During
the tube removal, the nurse instructs the client to take which action?
1. Inhale deeply.
2. Exhale slowly.
3. Hold in a deep breath.
4. Pause between breaths.
Answer: C

ِ‫يِاْلَ أرض‬
‫أ‬ ُ ‫َم‬
ُ ‫ك‬
‫ثِِف‬ َ ‫اس‬
‫ِِفي أ‬َ َّ ‫ع‬
‫ِالن‬ ُ ‫ف‬
َ ‫ماِيَ أن‬ َّ َ‫وأ‬
َ ِ‫ما‬ َ
{ 164 }

Valsalva Maneuver
‫ كمان مشهور اكتر وانا بركب او بشيل‬CVP

650)25 y. Patient came to ER with family history of ADHD try to suicide by ingesting 225
mg (Methylphenidate) concerta since 3-hour, heart rate 190beat /min, what is the
nursing priority:
1. Gastric lavage
2. Put pt. on cardiac monitor
3. Give activated charcoal
4. Check blood pressure
Answer: C

651)A neonatal nurse performs Apgar assessment at 1 minute


of birth to evaluate the physical condition of the newborn
and immediate need for resuscitation. At 1 minute, Apgar
score is 7. At 5 minutes Apgar score is to the progression of
scores suggests
A. A healthy newborn ✅
B. The need for supplement oxygen
C. A genetic defect
D. The infant is becoming stable
Answer: A

ِ‫يِاْلَ أرض‬
‫أ‬ ُ ‫َم‬
ُ ‫ك‬
‫ثِِف‬ َ ‫اس‬
‫ِِفي أ‬َ َّ ‫ع‬
‫ِالن‬ ُ ‫ف‬
َ ‫ماِيَ أن‬ َّ َ‫وأ‬
َ ِ‫ما‬ َ
{ 165 }

652)Neonate fontanelle shape??


A. Soft and flat
B. Soft and sunken
Answer: A
653) Why do we give the patient plasma?
A-Immunity
B-stability ✅

C-Fighting Virus
Answer: B
654) Which hormone causes the release of the follicle from
the ovary?
A. FSH✅

B. Progesterone
C. Estrogen
Answer: A
655)A newly assigned nurse to the Home Health Care Department was
conducting pre-entry phase of home visit.
Which of the following activities should the nurse do?
A. Call physician for a referral order
B. Collect information and schedule a visit.
C. Establish mutually acceptable goals for care.
D. Conduct a health history and physical assessment
Answer: B
656) A nurse is planning to assess the corneal reflex on
unconscious client. Which of the following is the safest?
stimulus to touch the client’s cornea.
a. Cotton buds

ِ‫يِاْلَ أرض‬
‫أ‬ ُ ‫َم‬
ُ ‫ك‬
‫ثِِف‬ َ ‫اس‬
‫ِِفي أ‬َ َّ ‫ع‬
‫ِالن‬ ُ ‫ف‬
َ ‫ماِيَ أن‬ َّ َ‫وأ‬
َ ِ‫ما‬ َ
{ 166 }

b. Sterile glove
c. Sterile tongue depressor
d. Wisp of cotton L Answers & Rationale at
Answer: D
657)Nursing manager her unscheduled staff absence
what done
1_self-staffing
2_on call staffing
Answer: B
658)35years old female patient after taking breast biopsy she diagnosed with 3rd stage
breast cancer.
She asked what is the purpose of chemotherapy?
A-it make no need for surgery.
B- destroy cancer cells and prevent it from spreading.
C- prevent recurrence of cancer.
Answer: B
659)nurse conducting research by using qualitative data
Which of the following methods she used to gather this data?
A_ weekly interview or one to one interview
B_ surveys
C_ data analysis
Answer: A
660)What is the function of Maslow hierarchy for nursing??
A. Set priority
B. Helping establish nursing diagnosis NANDA.
Answer: A
661) The patient has skull fracture. The patient is complaining racoon eye, pain,and
posterior neck fracture what is the type of skull fracture?
A. Basilar✅

ِ‫يِاْلَ أرض‬
‫أ‬ ُ ‫َم‬
ُ ‫ك‬
‫ثِِف‬ َ ‫اس‬
‫ِِفي أ‬َ َّ ‫ع‬
‫ِالن‬ ُ ‫ف‬
َ ‫ماِيَ أن‬ َّ َ‫وأ‬
َ ِ‫ما‬ َ
{ 167 }

B. Depressed
C. Compound
D. Linear
Answer: A
662)An older-adult patient has developed acute confusion. The patient has been on
tranquilizers for the past week. The patient’s vital signs are normal. What should the
nurse do?
a. Consider age-related changes in body systems that affect pharmacokinetic activity.
b. Increase the dose of tranquilizer if the cause of the confusion is an infection.
c. Note when the confusion occurs and medicate before that time.
d. Restrict phone calls to prevent further confusion.
Answer: A
663). A30week gestational preterm admitted to NICU 2hours ago the neonate starts to
have grunting, nasal flaring.
which of the following the nurse recognize regarding signs and symptoms?
A. Neonate has RDS.
B. It is normally in the first 24 hours of birth.
C. This is not significant unless become cyanosis.
D. Neonate has hypoglycemia.
Answer: A

After umbilical hernia / inguinal hernia repair


Avoid coughing. Avoid straining. Avoid heavy weight.
664)While a nurse is assessing an infant born 11 hours ago full term by caesarean
section, she auscultated moist lung sounds.
Which of the following is the most likely interpretation?
A. Abnormal finding
B. Normal finding
C. Pneumothorax
D. Surfactant aspiration
Answer: B

ِ‫يِاْلَ أرض‬
‫أ‬ ُ ‫َم‬
ُ ‫ك‬
‫ثِِف‬ َ ‫اس‬
‫ِِفي أ‬َ َّ ‫ع‬
‫ِالن‬ ُ ‫ف‬
َ ‫ماِيَ أن‬ َّ َ‫وأ‬
َ ِ‫ما‬ َ
{ 168 }

665)All the following are normally present in urine result EXCEPT:


a) Urea.
b) Creatinine.

c) Albumin. ✅

d) Sodium.
Answer: C
666)A patient has had open heart surgery and is in a coma. The patient’s spouse brings
suit against the physician, alleging that the patient was not informed of the possible
complications from use of general anesthesia. The suit will be filed on the following
basis.
A. malfeasance.
B. misfeasance.
C. nonfeasance
D. negligence
Answer: B

Malfeasance - - - - intentional to do harm .‫قاصد أضر المريض‬


Misfeasance - - - - - - unintentional action ‫عملت خطأ بدون قصد‬
667) Patient with paranoid schizophrenia most sign and symptoms
A⁃ Grandiose/granduer delusions ✅

B ⁃ - psychomotor
Answer: A

-pyloric stenosis (metabolic alkalosis)


668)The mother brought her child to hospital then the child admitted to NICU. After
leaving the mother for him. He started crying.
what stage of anxiety separation?

A. Protest, despair, denial ✅

B. Protest, despair and denail

ِ‫يِاْلَ أرض‬
‫أ‬ ُ ‫َم‬
ُ ‫ك‬
‫ثِِف‬ َ ‫اس‬
‫ِِفي أ‬َ َّ ‫ع‬
‫ِالن‬ ُ ‫ف‬
َ ‫ماِيَ أن‬ َّ َ‫وأ‬
َ ِ‫ما‬ َ
{ 169 }

C. Denial, protest, despair


D. Protest, denial, despair
Answer: A
669)Nephrotic syndrome what color urine?
Brown✅

Red
Answer: A
670)Nephritic syndrome urine color?

Bright red✅

Dark Brown
Answer: A
671)pregnancy patches contradiction?

Over wright 80 ✅
Have diarrhea.
With antibiotic
Answer: A

Hypertension 177/80 after 180/60! Risk for? Stroke


Catheter when remove? Ask patient deep breath.
Chest tube or CVP removal Valsalva maneuver
672)Patient have coped diagnosis high priority.
A. Ineffective Breath pattern
B. Gas exchange
Answer: A
673) The nurse was giving drug to the patient it was tablet, the patient said that is
not my medication. Why my drug is changed in color? What should the nurse
response??
A. Check the medication list

ِ‫يِاْلَ أرض‬
‫أ‬ ُ ‫َم‬
ُ ‫ك‬
‫ثِِف‬ َ ‫اس‬
‫ِِفي أ‬َ َّ ‫ع‬
‫ِالن‬ ُ ‫ف‬
َ ‫ماِيَ أن‬ َّ َ‫وأ‬
َ ِ‫ما‬ َ
{ 170 }

B. Check the doctor order ✅


C. Say this is your medication
D. Say the doctor wrote new order new drug
Answer: B
674)500mg diluted in ml 250ml D5NS with infusion pump then she modified the dose to
20mg/hr. How many ml/hr. should the nurse give??
A)5
B) 10
C) 15
D) 20
Answer: B
675)Nurse manager is doing change in her unite. What is
third stage of change??
A) select strategies
B) solution
C) Change strategies.
Answer: Bbb
676)The nurse is evaluating the patient with end stage chronic. obstructive pulmonary
disease (COPD). The patient has not achieved any of the goals in the plan of care. The
spouse reports. Concerns about the patient’s mood and increased dependency. What
action should the nurse take FIRST?
a. Continue the care plan for 1more month.
b. Refer the patient to psychiatric services.
c. Collaborate with the patient and spouse to revise the care plan.
d. Revise the care plan based on the spouse’s input.
Answer: C
677)A nurse is caring for a patient receiving total parenteral nutrition.
(TPN). The patient reports the sudden onset of feeling short of
breath and anxious. The nurse hears crackles in bilateral lower.
lobes of the lungs and the patient’s O2 saturation is 90%on room

ِ‫يِاْلَ أرض‬
‫أ‬ ُ ‫َم‬
ُ ‫ك‬
‫ثِِف‬ َ ‫اس‬
‫ِِفي أ‬َ َّ ‫ع‬
‫ِالن‬ ُ ‫ف‬
َ ‫ماِيَ أن‬ َّ َ‫وأ‬
َ ِ‫ما‬ َ
{ 171 }

air. The nurse must IMMEDIATELY:


a. Turn off the TPN.
b. Notify the physician.
c. Asses the patient’s capillary blood glucose level
d. Attempt to suction the patient’s airway.
Answer: B

Tocolytics drug
(also called anti-contraction medications or labor suppressants) are medications used
to suppress premature labor.

Oxytocin drug
used to cause contraction of the uterus to start labor, increase the speed of labor, and
to stop bleeding following delivery.
678) newly RN nurse is about to remove a nasogastric tube to a client with Guillen Barre
Syndrome. To determine the proper Time of removing the tube, the nurse should do it
when.
A-when the client feeling hungry
B-when the client wants to remove it.
C-when the client totally conscious
D- when the client exits flatulence or gases✅

Answer: D
679)A 25yearold woman presents to the Emergency Room with problem. Her main
complaint is that she woke up that morning see anything. All medical examination was
normal. However, reported that two days ago her husband had asked for a divorce.
What is the most likely disorder?
A. Pain
B. Conversion
C. Somatization
D. Body Dimorphic
Answer: C

ِ‫يِاْلَ أرض‬
‫أ‬ ُ ‫َم‬
ُ ‫ك‬
‫ثِِف‬ َ ‫اس‬
‫ِِفي أ‬َ َّ ‫ع‬
‫ِالن‬ ُ ‫ف‬
َ ‫ماِيَ أن‬ َّ َ‫وأ‬
َ ِ‫ما‬ َ
{ 172 }

680) What is the updated technique for CPR According to AHA?


A. 30:2120at5cm✅
B. 15:2120at5cm
C. 15:2120at5cm
D. 30:2100at4cm
Answer: A
681) A patient is being prepared for discharge following hip replacement surgery. The
nurse is providing him with discharge education. Which of the following information
should be taught to him as an effective pain management principle?
A. Avoid giving pain medication prior to participating in physical therapy
B. Give a double dose of pain medication if pain is intolerable
C. Give pain medication before pain becomes severe✅

D. Delay giving pain medication as long as possible


Answer: C
682) Skeletal traction urinary catheter were discontinued for a patient who was
immobilized in traction for sex weeks. The patient developed a problem with urinary
incontinence. Which of the following is the most appropriate intervention?
A. Scheduled toileting
B. Bladder retraining✅
C. Promoted voiding
D. Behavioral training
Answer: B

What is the process that sperm starting penetrates the ovum?? - Fusion process

Capacitation process
is the penultimate step in the maturation of mammalian spermatozoa and is required
to render them competent to fertilize an oocyte. This step is a biochemical event the
sperm move normally and look mature prior to Capacitation.

ِ‫يِاْلَ أرض‬
‫أ‬ ُ ‫َم‬
ُ ‫ك‬
‫ثِِف‬ َ ‫اس‬
‫ِِفي أ‬َ َّ ‫ع‬
‫ِالن‬ ُ ‫ف‬
َ ‫ماِيَ أن‬ َّ َ‫وأ‬
َ ِ‫ما‬ َ
{ 173 }

683) mother of a patient who is on antipsychotic drug asked a doctor about some
more detail about the drug. The doctor told the mother that her son is taking an
atypical antipsychotic drug. Which of the following is an examples of this type of drug?
A. Thioridazine
B. Clozapine✅
C. Chlorpromazine
D. Haloperidol
Answer: B
684). The nurse is assisting a doctor with the removal of a central venous catheter. To
prevent complications, the patient should be instructed to
A. Turn his head to the left side and hyperextend the neck while looking up.
B. Take slow, deep breaths as the catheter is advanced.
C. Perform the Valsalva maneuver as the catheter is pulled.
D. Turn his head to the right while grasping the siderails.
Answer: C

Noninvasive test of Hirsch rung disease anorectal mammography.


685)Which of the following of maternal age and other factors that affect negatively on
labor and cause severe complications?
A. Female 50and above
B. Female 16 or less
C. Hypertension
D. Anemia
Answer: A
686)The hospital director attended to ward for evaluation of staff regarding vital signs
follow up during blood transfusion. What is the most appropriate action for monitoring
vital signs during blood transfusion depends on?
A. According to physician order
B. According to nursing supervisor instructions
C. According to hospital policy
D. According to blood bank staff instruction
Answer: C

ِ‫يِاْلَ أرض‬
‫أ‬ ُ ‫َم‬
ُ ‫ك‬
‫ثِِف‬ َ ‫اس‬
‫ِِفي أ‬َ َّ ‫ع‬
‫ِالن‬ ُ ‫ف‬
َ ‫ماِيَ أن‬ َّ َ‫وأ‬
َ ِ‫ما‬ َ
{ 174 }

687)female patient is diagnosed with deep-vein thrombosis. Which nursing diagnosis


should receive the highest priority at this time?
A. Impaired gas exchange related to increased blood flow.
B. Fluid volume excess related to peripheral vascular disease.
C. Risk for injury related to edema.
D. Altered peripheral tissue perfusion related to venous congestion.
Answer: D
688)A nurse is collecting a urine of a 4-year-old child with nephrotic syndrome. Which of
following observation about the color of the child's urine the nurse expected to?
will chart.
A. Bright red
B. Amber
C. Dark, frothy
D. Orang
Answer: C
689)A nurse is collecting a urine of a 4-year-old child with nephritic syndrome. Which of
following observation about the color of the child's urine the nurse expected to
will chart.
A. Bright red
B. Amber
C. Dark, frothy
D. Orange
Answer: A
690) Patient diagnosed with UTI. He has frequent urination. The urine analysis is
rescheduled to the next day. What is the best action for the patient?
A. Increase fluid intake✅
B. Decrease fluid intake
C. Normal fluid intake
D. Avoid fluid intake
Answer: A

ِ‫يِاْلَ أرض‬
‫أ‬ ُ ‫َم‬
ُ ‫ك‬
‫ثِِف‬ َ ‫اس‬
‫ِِفي أ‬َ َّ ‫ع‬
‫ِالن‬ ُ ‫ف‬
َ ‫ماِيَ أن‬ َّ َ‫وأ‬
َ ِ‫ما‬ َ
{ 175 }

691)A female client who received general anesthesia returns from surgery.
Postoperatively, which nursing diagnosis takes highest priority for this client?
A. Acute pain R/T surgery
B. Deficient fluid volume R/T blood and fluid loss from surgery
C. Impaired physical mobility R/T surgery
D. Risk for aspiration R/T anesthesia
Answer: D
692)What is the Contraindication of baby vaccine?
A. Antiemetic
B. Antibiotics
C. Steroid ✅

Answer: C
693)A 70-year-old man presents to the clinic with difficulty sleeping at night. He has not
had a good night's rest for several months and feels exhausted. He needs to place
three pillows behind his back in order to sleep. Examination of the lungs reveals crackles
and wheel Auscultation of the heart confirms an S gallop. Which of the following is the
most likely underlying health problem?
A. Asthma
B. Pulmonary stenosis
C. Right-sided heart failure
D. Left-sided heart failure
Answer: D
694)nurse makes a clinical judgment that an African American male patient in a
stressful job is more vulnerable to developing hypertension than White male patients in
the same or similar situation. The nurse has formulated what type of nursing diagnosis?
A. Actual
B. Risk
C. Possible
D. Wellness
Answer: B

ِ‫يِاْلَ أرض‬
‫أ‬ ُ ‫َم‬
ُ ‫ك‬
‫ثِِف‬ َ ‫اس‬
‫ِِفي أ‬َ َّ ‫ع‬
‫ِالن‬ ُ ‫ف‬
َ ‫ماِيَ أن‬ َّ َ‫وأ‬
َ ِ‫ما‬ َ
{ 176 }

695) What is the difference between measles and German measles?

Koplik spot and high fever ✅

696) What is signs and symptoms of Esophagial atrasia?


A. Crying

B. Excessive frothy salivation from nose and mouth ✅✅

C. Absent respiratory sound


Answer: B

Spina Bifida Occulta: A small hairy patch on back, or a soft skin colored mass
called a lipoma,
That is the only external signs for diagnosis of spina bifida occult.

697)According to Maslow's hierarchy of needs, which nursing diagnosis has the lowest
priority for a client admitted to the intensive care unit with a diagnosis of congestive
heart failure?
a) Impaired urinary elimination
b) Ineffective airway clearance
c) Ineffective coping
d) Risk for body image disturbance
Answer: D
698)Which of the following nursing diagnoses has the highest priority when caring for an
older adult client with Alzheimer's disease?
a) Impaired physical mobility
b) Impaired memory
c) Self-care deficit
d) Risk for injury
Answer: D

ِ‫يِاْلَ أرض‬
‫أ‬ ُ ‫َم‬
ُ ‫ك‬
‫ثِِف‬ َ ‫اس‬
‫ِِفي أ‬َ َّ ‫ع‬
‫ِالن‬ ُ ‫ف‬
َ ‫ماِيَ أن‬ َّ َ‫وأ‬
َ ِ‫ما‬ َ
{ 177 }

699)A nurse is reviewing the plan of care for a client with a breathing problem. Which of
the following would the nurse most likely expect to identify as a relevant nursing
diagnostic statement for this client?
a) Altered airway
b) Impaired respiration
c) Impaired breathing rate
d) Ineffective airway clearance
Answer: D
700)planning the care for a client who has pneumonia, the nurse collects data and
develops nursing diagnoses. Which of the following is an example of a properly
developed nursing diagnosis?
a) Ineffective health maintenance as evidenced by unhealthy habits
b) Ineffective airway clearance as evidenced by inability to clear secretions
c) Ineffective therapeutic regimen management due to smoking
d) Ineffective breathing pattern related to pneumonia
Answer: B
701)Using Maslow's hierarchy of needs, rank the following nursing diagnoses in order of
importance, beginning with the highest-priority diagnosis.
1)Anxiety
2)Risk for infection
3)Disturbed body image
4)Sleep deprivation
Answer: Arrangement 4213
702)patient with a stroke is paralyzed on the left side of the body and has developed a
pressure ulcer on the left hip. The best nursing diagnoses for this patient is:
a) Impaired physical mobility related to left-sided paralysis
b) Risk for impaired tissue integrity related to left-sided weakness
c) Impaired skin integrity related to altered circulation and pressure
d) Ineffective tissue perfusion related to inability to move independently

ِ‫يِاْلَ أرض‬
‫أ‬ ُ ‫َم‬
ُ ‫ك‬
‫ثِِف‬ َ ‫اس‬
‫ِِفي أ‬َ َّ ‫ع‬
‫ِالن‬ ُ ‫ف‬
َ ‫ماِيَ أن‬ َّ َ‫وأ‬
َ ِ‫ما‬ َ
{ 178 }

Answer: C
703)A nurse is performing an initial assessment for a client. Which of the following would
be considered subjective information received during the assessment?
A. The client rates pain at a level of 6 on the numeric rating scale
B. The client has a pinpoint rash on the face and trunk.
C. The client's blood pressure increases when the provider enters the room.
D. The client weighs 186 pounds
Answer: A
704) You are preparing the nursing care plan for a middle-aged patient admitted to
the intensive care unit for an acute myocardial infarction (heart attack). His symptoms
include tachycardia, palpitations, anxiety, jugular vein distention, and fatigue. Which of
the following nursing diagnoses is most appropriate?
1)Decreased Cardiac Output
2)Impaired Tissue Perfusion
3)Impaired Cardiac Contractility
4)Impaired Activity Tolerance
Answer: A
705)The nurse performs an assessment of a 23-year-old man who believes that.
people are spying on him. During the interview, he keeps his eyes to the floor.
and answers questions awkwardly. He has never had an intimate relationship.
and avoids contact with his family members. He has never been employed and
tells the nurse that he is not looking for a job. The nurse considers Erickson’s.
theory of psychosocial development.
Which stage is this patient most likely experiencing?
a- Autonomy versus shame and doubt
b- Initiative versus guilt
c- Trust versus mistrust
d- Identity versus confusion
Answer: C

ِ‫يِاْلَ أرض‬
‫أ‬ ُ ‫َم‬
ُ ‫ك‬
‫ثِِف‬ َ ‫اس‬
‫ِِفي أ‬َ َّ ‫ع‬
‫ِالن‬ ُ ‫ف‬
َ ‫ماِيَ أن‬ َّ َ‫وأ‬
َ ِ‫ما‬ َ
{ 179 }

706) A nurse is assessing the uterus of a G5P4 patient immediately after delivery. The
nurse notes the fundus is

not contracted. Which of the following is the most appropriate immediate action
should be taken?

A. Massage the fundus

B. Assess the bladder

C. Elevated the mother's legs

D. Encourage the mother to void

Answer: A
707) *What is Early sign of portal hypertension?
A. Bradycardia
B. Hypotension✅
C. Flat jugular vein
Answer: B
708) A 39-year-old female is prepared for endoscopic craniotomy for.
removal the accumulated fluid in the dura matter and a biopsy. She
is prepared preoperatively. What essential knowledge about the?
procedure that should be explained to her.
A. Small skull bone is removed to biopsy brain tissue.
B. Needle is guided to remove abnormal brain tissue.
C. Specialized tools are used to remove bones section.
D. Lighted tube camera is used to look inside the skull.
Answer: D
709) *Isolation for RSV>>> Contact isolation
710) Pregnant mother come to ER complain of painless vaginal discharge ?
A. placenta previa,
* Pregnant mother 36 weeks of gestational vitally stable what nursing should instruct the
pt ?
A. wait untill 40 weeks

ِ‫يِاْلَ أرض‬
‫أ‬ ُ ‫َم‬
ُ ‫ك‬
‫ثِِف‬ َ ‫اس‬
‫ِِفي أ‬َ َّ ‫ع‬
‫ِالن‬ ُ ‫ف‬
َ ‫ماِيَ أن‬ َّ َ‫وأ‬
َ ِ‫ما‬ َ
{ 180 }

What is the Role of head nurse?


A. Ambulate the pt.
B. Monitor ecg for the patient
C. Recruiting the nurses
D. Allocate the nurse to the patient ✅

Answer: D
711)Which of the following mothers is High risk for laceration during labor?
A. 1 para mother for 12-hour delivery
B. 3 para mother for 9 hours' delivery
C. 2 para mother for 9 hours' delivery
D. 2 para mother for 6 hours' delivery
Answer: B

712)Incubation period of infection is:


A) Person is most infectious and nonspecific sign and symptom.
B (Organism growing and multiplying.
C) Recovery from infection.
D) Presence of specific sign and symptom.
Answer: A
713) 25 weeks-pregnant, primary gravid woman is on her first antenatal visit, completing
physical examination and

history, the midwife found out her husband has sickle cell anemia minor. What should
be the most appropriate plan of care?

A. Genetic counselling ✅

B. Identify severity of disease

C. Discuss the chances of transferring

D. Amniocentesis to identify genetic abnormality

Answer: A

ِ‫يِاْلَ أرض‬
‫أ‬ ُ ‫َم‬
ُ ‫ك‬
‫ثِِف‬ َ ‫اس‬
‫ِِفي أ‬َ َّ ‫ع‬
‫ِالن‬ ُ ‫ف‬
َ ‫ماِيَ أن‬ َّ َ‫وأ‬
َ ِ‫ما‬ َ
{ 181 }

714) Nurses work together in team project... suddenly there conflict between two nurses
....what should nurse supery do?

A. Make nurses work independently

B. Make nurses continue work together

C. Make nurse confront team members

D. Inform nurse supervisor to resolve issues

Answer: B
715)Patient exposure to second degree burn what electrolytes imbalance?
A. Sodium depletion due to shift water✅✅

B. Increase potassium due to cell destruction


Answer: A
716) Patient recieves (Psychotrophic medication ). The Patient develop
agranulocytosis.

This side effects to which medication cause ?

A. Typical antipsychotic

B. Atypical antipsychotic ✅

C. Sertonin reuptake inhibitor

D. Noradrenaline reuptake inhibitor

Answer: B
717) Woman came to ER with right abdomenal pain ...vaginal bleeding.. AFter
Ultrasound. THEY Detect she is pregnant in 10، weeks .... there is adenxal tenderness
...this most likely ?

A. Appendecitis

B. Ectopic pregnancy ✅

C. Threatened abortion

Answer: B
718) How to differentiate between hypovolemic shock and postpartum hemorrhage?
A. Hemoglobin

ِ‫يِاْلَ أرض‬
‫أ‬ ُ ‫َم‬
ُ ‫ك‬
‫ثِِف‬ َ ‫اس‬
‫ِِفي أ‬َ َّ ‫ع‬
‫ِالن‬ ُ ‫ف‬
َ ‫ماِيَ أن‬ َّ َ‫وأ‬
َ ِ‫ما‬ َ
{ 182 }

B. Hematocrit
C. Cold clammy skin
D. Increase Pulse✅

Answer: D
719) During providing care to the patient. There was emergency situation in the ward.
She already given medication to patient but still no documentation yet. What should
the nurse do to prevent any error result from delayed documentation?
A. Document at the end of the shift
B. Write note that drug done and the dose✅

C. Don't go to emergency situation until you finish documentation


Answer: B
720) During operation procedure for patient with inguinal hernia. The doctor explored
another site for hernia not included in the informed consent. What is the most
appropriate action?
A. Do the extra operation then get consent later after patient aware
B. Wait for procedure until patient signed another consent
C. Call for client's medical power attorney to provide additional informed consent For
additional procedure✅
D. Additional informed consent and document in patient care
Answer: C
721) What is hormons for biological difference between male and female?
A. Estrogen and HCG
B. Testosterone and progestron✅✅

Answer: B
722)Child with Pyloric stenosis. What is the expected signs and symptoms
postoperative?
A. Abdominal pain
B. Watery stool

C. Vomiting ✅

ِ‫يِاْلَ أرض‬
‫أ‬ ُ ‫َم‬
ُ ‫ك‬
‫ثِِف‬ َ ‫اس‬
‫ِِفي أ‬َ َّ ‫ع‬
‫ِالن‬ ُ ‫ف‬
َ ‫ماِيَ أن‬ َّ َ‫وأ‬
َ ِ‫ما‬ َ
{ 183 }

D. Urinary
Answer: C
723)-A 40 years old complains of severe pain, scheduled for surgery of abdomen,
medical history he smokes one packed of cigarettes per day. Which of the following
complications he might develop?
A- atelectasis ✅
B- DVT
C- constipation
Answer: A
724)-A nurses is providing teaching session for pre-operative patient before
appendectomy about how to use spirometer, which of the following nursing process?
A- assessment
B- evaluation

C- implementation ✅
D- diagnoses
Answer: C
725)A parent asks the nurse how dissociative disorder can be treated?
A- psychotherapy ✅
B- psychology
C- electro convulsive therapy (ECT)
Answer: A
726)Patient post, the goal for him to be mobile at 9 but he mobilized at 12, what should
the nurse do?
A- delay discharge process

B- discharge and write goal met✅

C- collaboration with therapist to review goal and plan.


Answer: B
727) What is the Ministry of health in saudi arabia precutions regarding H5N1?

ِ‫يِاْلَ أرض‬
‫أ‬ ُ ‫َم‬
ُ ‫ك‬
‫ثِِف‬ َ ‫اس‬
‫ِِفي أ‬َ َّ ‫ع‬
‫ِالن‬ ُ ‫ف‬
َ ‫ماِيَ أن‬ َّ َ‫وأ‬
َ ِ‫ما‬ َ
{ 184 }

A. Hand washing before enter patient room ✅

B. Be cautious with patient things

C. Wear gloves and gown

D. Don't follow precautions

Answer: A
728) A 17-year-old mother presented to the primary health center ten after delivery. She
is suffering from fatigue,
anemia, fever and vaginal discharge (see lab results)
Blood pressure 80/50 mmHg
Heart rate 112 /min
Respiratory rate 35 /min
Temperature 39.6 C
Test Result Normal Values
RBC 4 4.7-6.1 × 1012 /L (male) 4.2-5.4 × 1012 /L (female)
Hb 90 130-170 g/L 120-160 g/L (female)
HCT 0.29 0.42-0.52 (male) 0.37-0.48 (female)
WBC 12.8 4.5-10.5 × 109/L
Which of the following is the best diagnosis of health problem in this case?
A. Severe urinary track infection
B. Vesical-vaginal fistula
C. Puerperal sepsis ✅✅

D. Post-partum hemorrhage
Answer: C
729) Nurse was documenting, and she documented wrongly (Error), what she should
do?
A- Review policy of how to correct wrong documentation.
B- A- use liquid and Waite until dry then write the correct one above it.
C- use liquid and write the correct one beside.

ِ‫يِاْلَ أرض‬
‫أ‬ ُ ‫َم‬
ُ ‫ك‬
‫ثِِف‬ َ ‫اس‬
‫ِِفي أ‬َ َّ ‫ع‬
‫ِالن‬ ُ ‫ف‬
َ ‫ماِيَ أن‬ َّ َ‫وأ‬
َ ِ‫ما‬ َ
{ 185 }

D- do line then write the correct beside ✅

Answer: D
730) A nurse wants to delegate a post op patient from major surgery to assistant nurse,
what task she will delegate?

A- ambulate the patient ✅✅

B- obtain vital signs.


C- check patency of NGT
Answer: A
731(A nurse is providing session regarding lung cancer, which statement indicates
teaching was not effective?
A- I will eat fruit and vegetables
B- if I have COPD does not mean I will have lung cancer.
C- no need to worry because I do not smoke, my husband does. ✔️

Answer: C
732) A nurse is obtaining vital signs for patient, which of the following she will document
Bp; 138/78
HR: 108
RR: 18
T: 36.5

A- tachycardia✅

B- arrhythmia
C- tachypnea
D- hypertension
Answer: A
733)which of the following statements regarding SLE nurse should provide to group of
people?

A- hypertension must be anticipated ✅

B- hypertension is not common with SLE.

ِ‫يِاْلَ أرض‬
‫أ‬ ُ ‫َم‬
ُ ‫ك‬
‫ثِِف‬ َ ‫اس‬
‫ِِفي أ‬َ َّ ‫ع‬
‫ِالن‬ ُ ‫ف‬
َ ‫ماِيَ أن‬ َّ َ‫وأ‬
َ ِ‫ما‬ َ
{ 186 }

C- the symptom will exacerbate more in winter and spring.


Answer: A
734) position of patient with endoscopy?

Supine
735) first thing for nurse after needle stick injury will do.
A- squeeze her finger.

B- write incident report✅


Answer: B
736)When communicating with a client who speaks a different language, which best
practice should the nurse implement?
A. Speak loudly and slowly.
B. Arrange for an interpreter to translate.
C. Speak to the client and family together.
D. Stand close to the client and speak loudly.
Answer: B
737)Doctor order new antibiotics first thing to do before administering medication.
1- check the order in the system
2- do CBC
3- Obtain blood cultures ✅

4- administration first dose when standard medication time.


Answer: C
738)Infant with breast feeding associate jaundice came to clinic. The Infant has
yellowish sclera discoloration. What should the nurse do?

A. Encourage frequent breast feeding✅

B. Admitted baby and do phototherapy.


C. Withhold breast feeding.
Answer: A

ِ‫يِاْلَ أرض‬
‫أ‬ ُ ‫َم‬
ُ ‫ك‬
‫ثِِف‬ َ ‫اس‬
‫ِِفي أ‬َ َّ ‫ع‬
‫ِالن‬ ُ ‫ف‬
َ ‫ماِيَ أن‬ َّ َ‫وأ‬
َ ِ‫ما‬ َ
{ 187 }

739)Patient attached with mechanical ventilator. The doctor assesses the patient to
weaning off ventilator. What is weaning mood must be adjusted?
A) assist control mood
B) CPAP MODE✅

Answer: B
740)Multiple myeloma is a neoplastic proliferation of Which of the following??
a- lymphocytes
b- Granulocytes

c- Plasma cells✅
d- Monocytes
Answer: C
741)The Diagnosis is the reduction in the amount of circulating.
hemoglobin, red blood cells or both:
a- polycythemia

b- Anemia✅
c- Hemophilia
d- Leucopenia
Answer: B
742)A nurse is caring for a client with head injury and monitoring him for decerebrate
posturing.
Which of the following is the characteristic of the posture?
A. Flexion of the extremities after stimulus
B. Extension of the extremities after a stimulus✅

C. Upper extremity flexion with lower extremity flexion


D. Upper extremity flexion with lower extremity extension
Answer: B
743) A 62-year-old male patient, admitted in the surgical Ward is scheduled for the
surgical removal of polyps from

ِ‫يِاْلَ أرض‬
‫أ‬ ُ ‫َم‬
ُ ‫ك‬
‫ثِِف‬ َ ‫اس‬
‫ِِفي أ‬َ َّ ‫ع‬
‫ِالن‬ ُ ‫ف‬
َ ‫ماِيَ أن‬ َّ َ‫وأ‬
َ ِ‫ما‬ َ
{ 188 }

his descending colon under general anesthesia. he is experiencing fatigue, abdominal


pain and blood streaked.
stools for a couple of months. he is worried whether the bleeding in his stools is going to
stop after surgery.
What is most appropriate response by the nurse for the patient concern?
A. Surgery often relieves the symptoms.

B. Let us have a detail discussion with your physician✅

C. Your condition may or may not resolve, it depends.


D. In fact surgery is the only treatment for the problem.
Answer: B
744) Cardiac patient smoking. He has blue lower extremity, pale skin and very
exhausted. the nurse observe that lower limb without hair. No hair growth in lower limb.
What is the appropriate cause of hair loss in his leg ?
A. Hermonal distubance
B. Impaired tissue perfusion✅
Answer: B
745)side effects of bCG vaccination
1-cold and small scar✅

2-diarrhea
C- rash for three days
Answer: A
746) mother has infant 6 months came to the clinic and afraid that her baby May will
have meningitis as his brother already have.
What should the nurse tell the mother?
1-hib vaccine can decrease the meningitis ✅✅
2-there are now vaccinations for all meningitis type
3- tell the mother the disease is will not come
Answer: A

ِ‫يِاْلَ أرض‬
‫أ‬ ُ ‫َم‬
ُ ‫ك‬
‫ثِِف‬ َ ‫اس‬
‫ِِفي أ‬َ َّ ‫ع‬
‫ِالن‬ ُ ‫ف‬
َ ‫ماِيَ أن‬ َّ َ‫وأ‬
َ ِ‫ما‬ َ
{ 189 }

747) The patient with burn in the face and hand what will be site to assess the pulse‫؟‬
A.radial
B.apical

C.femoral✅

Answer: C
748) The patient see black dots flying what does it mean?
A. Glaucoma
B. Retina deatachment ✅

Answer: B
749)Which influenza virus strain currently presents the greatest risk to human health?
A. H1N1
B. H2N2
C. H3N2
D. H5N1✅

Answer: D
750)Signs and symptoms of early fluid volume deficit, except.
A. Decreased urine output
B. Decreased pulse rate ✅

C. Concentrated urine
D. Decreased skin turgor
Answer: B
751)A 45-year-old man is admitted to the neurosurgery ward for the surgical elevation
of depressed skull fracture. He has episodic severe headache with seizure and is unable
to concentrate. Which of the following initial nursing problem needs more attention?
A. Disturbed coping and anger spells
B. Risk of injury to seizure
C. Disturbed communication and irritability
D. Pain management and comfort measure

ِ‫يِاْلَ أرض‬
‫أ‬ ُ ‫َم‬
ُ ‫ك‬
‫ثِِف‬ َ ‫اس‬
‫ِِفي أ‬َ َّ ‫ع‬
‫ِالن‬ ُ ‫ف‬
َ ‫ماِيَ أن‬ َّ َ‫وأ‬
َ ِ‫ما‬ َ
{ 190 }

Answer: B
752)A patient is seen in the emergency department with complaints of angina.
Nitroglycerin (Nitro-stat) is ordered by the physician. This medication is.
to be administration via which of the following routs?
A. Intradermal
B. Buccal
C. Parental
Answer: B
753) How can we know that this child has intussusception??
A. Baby crying &vomiting
B. Baby pull knee to chest ✅

Answer: B
754) The patient had adrenal insufficiency & take corticosteroid she will have?

A. Pigment skin & dry ✅


B. Pale skin & dry
C. Pink skin & healthy
Answer: A

755)Before sending a client for a CT with contrast dye, what the nurse's most important
action?
A. Teach about the need for post-procedure hydration.
B. Verify that the informed consent is complete.
C. Place the side rails of the bed up before transport.
D. Check the client's health record for allergies
Answer: D
756)A midwife is conducting a health a health education session to the prim gravid
mothers. The session is about.
antepartum care to ensure the health of the mother their babies. Which of the following
statement made by?

ِ‫يِاْلَ أرض‬
‫أ‬ ُ ‫َم‬
ُ ‫ك‬
‫ثِِف‬ َ ‫اس‬
‫ِِفي أ‬َ َّ ‫ع‬
‫ِالن‬ ُ ‫ف‬
َ ‫ماِيَ أن‬ َّ َ‫وأ‬
َ ِ‫ما‬ َ
{ 191 }

mother suggest their appropriate understandings of anti-partum periods.


A. Beginning of labor till the baby birth
B. Diagnosis of pregnancy till the baby birth
C. Last three months of pregnancy
D. Forty days after the child is born.
Answer: C
757) A patient has a hip fracture complaining of severe pain. He nurses finds no
analgesic was ordered. She calls the
orthopedic doctor by telephone and received a medication order.
What is the most appropriate nursing action to ensure correct medication order taken?
A. Write down the order and get pharmacy o supply.
B. Write down the order and get medication immediately with another registered nurse.
C. Get another registered to witness, nurse write down the order and read back to the
doctor
D. Write down the medication and after serving get the doctor to witness.
Answer: C
758) Following lumbar surgery a patient has a 4 millimeter (mm)
surgical incision. The incision is clean, and the edges are well
appropriate. This type of tissue healing is classified as which of the
following?
a. Primary intention
b. Secondary intention
c. Tertiary intention
d. Superficial epidermal
Answer: A
759) A surgeon instructs a nurse to serve as a witness to an elderly
patient’s informed consent for surgery. During the explanations to
the patient, it becomes clear that the patient is confused and does not.

ِ‫يِاْلَ أرض‬
‫أ‬ ُ ‫َم‬
ُ ‫ك‬
‫ثِِف‬ َ ‫اس‬
‫ِِفي أ‬َ َّ ‫ع‬
‫ِالن‬ ُ ‫ف‬
َ ‫ماِيَ أن‬ َّ َ‫وأ‬
َ ِ‫ما‬ َ
{ 192 }

understand the procedure, but reluctantly sign the consent form. The
nurse should:
A. Sign the form as a witness, making a nation that the patient did not.
appear to understand.
B. Not sign the form as a witness and notify the nurse supervisor.
C. Not sign the form and answer the patient’s questions after the surgeon.
leaves he room.
D. Sign the form and tell surgeon that the patient does not understand the
procedure
Answer: B
760) Which week lung (surfactant) will be mature?
A. 20 week
B. 24 week
C. 16 week
D. 28 week✅

Answer: D
761. What is the main organism that cause meningitis for a child?
1. Meningococcal
2. Staphylococcus
3. H. Influenza
4. Streptococcal Pneumonia
Answer: D

ِ‫يِاْلَ أرض‬
‫أ‬ ُ ‫َم‬
ُ ‫ك‬
‫ثِِف‬ َ ‫اس‬
‫ِِفي أ‬َ َّ ‫ع‬
‫ِالن‬ ُ ‫ف‬
َ ‫ماِيَ أن‬ َّ َ‫وأ‬
َ ِ‫ما‬ َ
{ 193 }

762)A13-weeks-pregnant, multigravida women is anxious, and apprehensive she has


five children and is not.
willing to continue with this pregnancy she is requesting the midwife to about the fetus,
she is underweight, malnutrition and is over worked, BMI 17kg/m2, what intervention is
desired immediately?
A. Admission and intravenous line management for induction.
B. Family planning and birth control measures
C. Dietary management and supplements.

D. Support, reassurance and counselling. ✅


Answer: D
763) When does mature surfactant begin to be Produced?
A. 21 week

B. 24 week✅
C. 28 week
D. 16 week
Answer: B
764) 1 month-old infant is admitted to the surgical unit with hypertrophic pyloric stenosis
and scheduled for the
surgery. Which of the following is the findings of abdominal examination?
A. palpable olive-like mass in the left side

ِ‫يِاْلَ أرض‬
‫أ‬ ُ ‫َم‬
ُ ‫ك‬
‫ثِِف‬ َ ‫اس‬
‫ِِفي أ‬َ َّ ‫ع‬
‫ِالن‬ ُ ‫ف‬
َ ‫ماِيَ أن‬ َّ َ‫وأ‬
َ ِ‫ما‬ َ
{ 194 }

B. palpable olive-like mass in the right side ✅


C. Palpable olive-like mass moved from left to right.
D. Palpable olive-like mass moved from right to left.
Answer: B
765) PYLORIC stenosis peristalsis movement?
A. From right to left.
B. From left to right✅

Answer: B
766) Surgery for pyloric stenosis movement?
A. Pylorotomy
B. Pylorectomy
C. Pylorostomy
D. Pyloromyotomy✅

Answer: D
767)A 5-week-old newborn was admitted to pediatric Ward with pyloric stenosis, the
newborn has weight loss,
and projectile vomiting during feeding. They scheduled surgical repair of pyloric stenosis
Which of the following.
postoperative intervention for this

A. IV fluid infant is retaining adequate amount by mouth ✅


B. Administration of proper analgesia until infant discomfort resolve
C. Start feeding immediately after postoperative.
D. Vomiting is uncommon in the first24-48 hrs.
Answer: A
768) Which of the following can be considered as a major development in nursing
home visits in terms of
information technology?
A. Medication dispensing
B. Telehealth

ِ‫يِاْلَ أرض‬
‫أ‬ ُ ‫َم‬
ُ ‫ك‬
‫ثِِف‬ َ ‫اس‬
‫ِِفي أ‬َ َّ ‫ع‬
‫ِالن‬ ُ ‫ف‬
َ ‫ماِيَ أن‬ َّ َ‫وأ‬
َ ِ‫ما‬ َ
{ 195 }

C. Patient monitoring
D. Prevention of epidemiological di
Answer: B
769) Best management for fracture..??

A. Immbolization ✅

B. Moblization

C. Control pain
Answer: A
770) Nurse manger want to make organizational structure which allow communication
to flow in all lines of structure and involving workers in decision making?

A. Centralized

B. Decentralized ✔️

C. Infomal
Answer: B
771) At is the recommended weight gain during pregnancy of a woman with MI of <
18.5?
A. 12.5 -18 Kg ✅

B. 11.5 -16 Kg
C. 7 -11.5 Kg
D. 5- 9 Kg
Answer: A
772)Normal weight gain during pregnancy.?
A. 12.5 -18 Kg
B. 11.5 -16 Kg ✅✅

C. 7 -11.5 Kg
D. 5- 9 Kg
Answer: B

ِ‫يِاْلَ أرض‬
‫أ‬ ُ ‫َم‬
ُ ‫ك‬
‫ثِِف‬ َ ‫اس‬
‫ِِفي أ‬َ َّ ‫ع‬
‫ِالن‬ ُ ‫ف‬
َ ‫ماِيَ أن‬ َّ َ‫وأ‬
َ ِ‫ما‬ َ
{ 196 }

773) 21 years old patient exhibit nervousness .. isolated from all family and friends ...
difficult falling sleep ... develop this after having crash train accident and have been
trapped for aday in train but wihout injury?

A. Post traumatic stress disorder

Answer: A
774)The nurse educator presents an in-service training session on case management to
nurses on
the clinical unit. During the presentation, the nurse educator clarifies that what is a
characteristic.
of case management?
A. Requires that 1 nurse take care of 1 client.
B. Promotes appropriate use of hospital personnel.
C. Requires a case manager who plans the care for all clients.
D. Uses a team approach, but 1 nurse supervises all other employees.
Answer: B
775)a child with tracheoesophageal fistula is scheduled for an operation. what should
the nurse do preoperatively?
A) Insert NGT
B) Suction periodically ✅✅

C) Prepare tracheostomy set if necessary


Answer: B
776) The doctor touches the bed by the sterile gloves the nurse should:
A. Ask him to change the gloves and to give him new one ✅✅

B. Let him to complete the Central line procedure.


Answer: A
777) A child with asthma has an order for albuterol, before administration of the
medication the nurse MUST.
a- Pre-oxygenate the patient
b- Assess the patient's heart rate ✔️✔️

ِ‫يِاْلَ أرض‬
‫أ‬ ُ ‫َم‬
ُ ‫ك‬
‫ثِِف‬ َ ‫اس‬
‫ِِفي أ‬َ َّ ‫ع‬
‫ِالن‬ ُ ‫ف‬
َ ‫ماِيَ أن‬ َّ َ‫وأ‬
َ ِ‫ما‬ َ
{ 197 }

c- Obtain venous Access


d- Feed the patient a snack
Answer: B
778)Steal syndrome in hemodialysis symptoms?
A. Pain and hotness of skin
B. Pain and coldness of skin ✔️✔️
Answer: B
779)The patient scheduled for operation. The doctor informed the patient about
expected complication and the outcomes May be not acceptable. What is the non-
preferable ethical principles?
A. Malpractice
B. Negligence
C. Ethical dilemma ✅

D. Molars concussi
Answer: C
780)Initially and important physical examination for Which patient?
A. Pneumonia

B. Pulsating in abdomen ✅
Answer: B
781)Classification system forth category???
A. Close monitoring complete assistance most activities ✅
B. Recovering serious illness and need to some assistance.
Answer: A
782)Which of the following is consider sputum expectorant?
A. Emphysema
B. Asthma
C. Bronchitis ✅

Answer: C
783)) preeclampsia suspected patient is presented to the ER. what level of triage?

ِ‫يِاْلَ أرض‬
‫أ‬ ُ ‫َم‬
ُ ‫ك‬
‫ثِِف‬ َ ‫اس‬
‫ِِفي أ‬َ َّ ‫ع‬
‫ِالن‬ ُ ‫ف‬
َ ‫ماِيَ أن‬ َّ َ‫وأ‬
َ ِ‫ما‬ َ
{ 198 }

A. Emergency✅

B. Urgent
Answer: A
784)postpartum priority assessment episiotomy for
A. Colour ✅

B. Edema
C. Discharge
D. Appearance
Answer: A

According to REEDA Redness Edema ecchymosis discharge Approximation

785)Complication DVT deep vein thrombosis:


A. pulmonary embolism ✅
B. Thrombophlebitis
Answer: A
786) Patient with burn >>>> Reverse isolation

787)Position that increases heart Murmur.


A. High Fowler
B. Sideline

C. Trendelenburg✅
Answer: C
788) Definition of effacement >> thining of cervix

789) The Height of enema is ?


A. 50 cm ✅
B. 150 cm
C. Blow red edge
Answer: A
790)A 1-year-old girl admitted to podiatric medical unit significant weight loss,
diminished mid-arm.

ِ‫يِاْلَ أرض‬
‫أ‬ ُ ‫َم‬
ُ ‫ك‬
‫ثِِف‬ َ ‫اس‬
‫ِِفي أ‬َ َّ ‫ع‬
‫ِالن‬ ُ ‫ف‬
َ ‫ماِيَ أن‬ َّ َ‫وأ‬
َ ِ‫ما‬ َ
{ 199 }

circumference diarrhea, and red hair. Which of the following type of malnutrition do
the nurse suspect?
A. Marasmus
B. Spitting up
C. Kwashiorkor ✅
D. Rickets
Answer: C

Stick like muscle is Marasmus.


791)A 12- year- old boy was brought to the Emergency respiratory arrest due to
drowning. Cardiac resuscitation
what is the major complication that might happen if treated after drowning quickly?
A. Sepsis
B. Alkalosis
C. Acidosis
D. Hypothermia
Answer: C
792)A 40-year-old man. smoker, presents to the clinic. On examination, the toes are
cold to the touch. Extremities are pale to blue. The pedal pulse examination of the
fingers shows small ulceration the skin. Blood glucose testing is normal and then history
of diabetes. On admission Blood pressure 140/90 mmHg Heart rate 86 /min Respiratory
rate 22 /min Oxygen saturation 98% room air Which of the following would be most
effective?
A. Antibiotic administration
B. Reduced fat intake
C. Smoking cessation
D. Regular exercise
Answer: C
793)A postpartum patient was in labor for 30 hours and had ruptured membranes for 24
hours. For which of the following would the nurse be alert?
A. Endometritis
B. Endometriosis

ِ‫يِاْلَ أرض‬
‫أ‬ ُ ‫َم‬
ُ ‫ك‬
‫ثِِف‬ َ ‫اس‬
‫ِِفي أ‬َ َّ ‫ع‬
‫ِالن‬ ُ ‫ف‬
َ ‫ماِيَ أن‬ َّ َ‫وأ‬
َ ِ‫ما‬ َ
{ 200 }

C. Salpingitis
D. Pelvic thrombophlebitis
Answer: A
794)schizophrenia patient started shouting loudly in the ward to anyone who speaks to
him. what should the nurse do?
A) administer tranquilizer IM
B) call the security
C) isolate the patient
D) speak quietly to the patient and be cautious
Answer: D
795)patient with Alzheimer's disease has been hospitalized after sustaining a fall and
fractured hip. The patient attempts to get out of bed and has fallen twice since
admission. What is the most likely nursing action?
A. Continue the care plan as written.
B. Revise the goals and interventions✅

C. Add restraints to the interventions.


D. Add a new nursing diagnosis of noncompliance.
Answer: B
796)Patient with (HF) he is on Digoxin, when the nurse checks vital signs, she noticed
pulse was 110. what is the nurse intervention?
A) hold digoxin medication
B) inform doctor
C) recheck pulse

D) give medication ✅

Answer: D
797)When examining the fetal monitor strip after the rupture of the
membranes in a laboring client. the nurse notes variable decelerations in the fetal heart
rate. The nurse should:
A. Stop the oxytocin infusion.

ِ‫يِاْلَ أرض‬
‫أ‬ ُ ‫َم‬
ُ ‫ك‬
‫ثِِف‬ َ ‫اس‬
‫ِِفي أ‬َ َّ ‫ع‬
‫ِالن‬ ُ ‫ف‬
َ ‫ماِيَ أن‬ َّ َ‫وأ‬
َ ِ‫ما‬ َ
{ 201 }

B. Change the client’s position✅

C. Prepare for immediate delivery.


D. Take the client’s blood pressure.
Answer: B
798)When providing care for and collecting evidence from a victim of
sexual assault, which initial step should the nurse focus on?

A- Obtaining consent ✅

B- Collecting specimens
C- Assessing the client's emotional state.
Answer: A
799).: Offer ....... medical equipment to encourage the child to expression of his feelings
about new hospitalization experience and illness.
A. Imitation
B. Toys✅

C. Material
D. Rules
Answer: B
800)The wrong management for marasmus may lead.
A. CNS complication
B. Kwashiorkor
C. Rickets
D. Diarrhea✅

Answer: D
801)the child fears of being alone and some machine sound during hospitalization at
age.
A.5 months
B.8 months
C.9 months

ِ‫يِاْلَ أرض‬
‫أ‬ ُ ‫َم‬
ُ ‫ك‬
‫ثِِف‬ َ ‫اس‬
‫ِِفي أ‬َ َّ ‫ع‬
‫ِالن‬ ُ ‫ف‬
َ ‫ماِيَ أن‬ َّ َ‫وأ‬
َ ِ‫ما‬ َ
{ 202 }

D.7years✅

Answer: D
802)The immunity system of the child reaches normal full mature level, at age.
A. 10 years
B. 9 years
C. 12 months✅
D. 15 years
Answer: C
803)A student taking the Saudi Commission exam in Dublin for the first time was
optimistic and happy accompanied by his friend, a nurse specialist, and after the
exam, his mood was sour, anxious and frustrated. Which of the following is the first
intervention that his friend makes?
1 let him cry✅✅

2 Hold him and lie down on his back.


3 share a hot drink.
4 drinks a cold beer
Answer: A
804) the doctor writes a new order (PRN RESTRAIN) for an aggressive patient, how can
doctor know the nurse understand the order:
A. Restrain patient once if need.
B. Call the doctor if patient be aggressive to restrain.
C. Do not follow the order and call supervisor.
D. Isolate the patient and not restrain.
Answer: A
805)Disadvantages of lntra Uterine Contrastive (IUD)
Expensive
Increase Bleeding✅✅

Answer: B
806) A 60-year-old patient was admitted with hepatic coma in the intensive care unit.
The physician has ordered.

ِ‫يِاْلَ أرض‬
‫أ‬ ُ ‫َم‬
ُ ‫ك‬
‫ثِِف‬ َ ‫اس‬
‫ِِفي أ‬َ َّ ‫ع‬
‫ِالن‬ ُ ‫ف‬
َ ‫ماِيَ أن‬ َّ َ‫وأ‬
َ ِ‫ما‬ َ
{ 203 }

protein restriction diet for the patient. Which of the following substances is most likely
causes harmful effects when?
the patient increases protein intake.
A. urea
B. creatinine
C. ammonia
D. amino acid
Answer: C
807)A patient complains of severe pain which he stated to be 9/10, the physician
ordered morphine 50 mg IV every 4 hours, the last dose was given 2 hours ago, what is
the best action his caring nurse would take:
A. Give another dose of morphine.
B. Inform the doctor to change the order.
C. Distract the patient by TV, radio or games for 2 hours.
D. Ignore the patient completely.
Answer: B
808)The infant 1 month with colic. What should nurse do??
Inform mother to increase milk product.
Put infant in prone position✅✅
Answer: B
809)Spina pifida risk for Which of the following complication??
Infection ✅✅
Fever
Bleeding
Which of the following Noninvasive test used to assess fetus??
A. Contraction stress

B. Non stress test✅✅


Answer: B

Aspirin should be stopped at least 7 to 10 days before surgery.


Heparin should be held 6 hours before surgery.

ِ‫يِاْلَ أرض‬
‫أ‬ ُ ‫َم‬
ُ ‫ك‬
‫ثِِف‬ َ ‫اس‬
‫ِِفي أ‬َ َّ ‫ع‬
‫ِالن‬ ُ ‫ف‬
َ ‫ماِيَ أن‬ َّ َ‫وأ‬
َ ِ‫ما‬ َ
{ 204 }

Warfarin should be stopped at


least 2 to 3 days before operation.

810)The most risk of death??

hemorrhage ✅
pulmonary embolism
-cervical cancer
Cardiac disease
Answer: A

VF - - - - - - Life Threatening
Stress incontinence - - - - - Kegel exercise

ِ‫يِاْلَ أرض‬
‫أ‬ ُ ‫َم‬
ُ ‫ك‬
‫ثِِف‬ َ ‫اس‬
‫ِِفي أ‬َ َّ ‫ع‬
‫ِالن‬ ُ ‫ف‬
َ ‫ماِيَ أن‬ َّ َ‫وأ‬
َ ِ‫ما‬ َ
{ 205 }

811)Which of the following we consider Congenital diaphragmatic hernia is related to


any conditions or under cases??
A-Sepsis
B-Pulmonary
C-Heart
D-Alkalosis
Answer: B
812)a nurse is putting together a presentation Meningitis. on Which of the following has
not been linked to Meningitis in?
A-S. Pneumonia
B-H. Influenza
C-n. Meningitis
D-C. difficile
Answer: D
813)A patient who is in isolation needs a temperature taken several times a day. Where
is the appropriate place for the thermometer to be kept?
A. At the nurses’ station.
B. On the isolation cart outside the patient’s room.
C. In the dirty utility room.
D. In the patient’s room
Answer: D
814)The nurse is caring for an adult patient who is admitted with.
chest pain that started four hours ago. Which test will be?
most specific in identifying acute heart damage.
A. CKP
B. Troponin level
C. CK-MB
D. Cholesterol level

ِ‫يِاْلَ أرض‬
‫أ‬ ُ ‫َم‬
ُ ‫ك‬
‫ثِِف‬ َ ‫اس‬
‫ِِفي أ‬َ َّ ‫ع‬
‫ِالن‬ ُ ‫ف‬
َ ‫ماِيَ أن‬ َّ َ‫وأ‬
َ ِ‫ما‬ َ
{ 206 }

Answer: B
815)The nurse is caring for a patient who is admitted with
chest pain and diagnosed with MI. Which test is assisting in diagnosis?
A. CK-MR
B. CK-MH
C. CK-MB
D. CK-HM
Answer: C

816)The purpose of the Adam's forward bend test is detecting


structural or functional scoliosis. This test is most often used during
school screening for scoliosis.

‫تيجي باالختبار‬ ‫👇👇أشهر تعريفات‬

ِ‫يِاْلَ أرض‬
‫أ‬ ُ ‫َم‬
ُ ‫ك‬
‫ثِِف‬ َ ‫اس‬
‫ِِفي أ‬َ َّ ‫ع‬
‫ِالن‬ ُ ‫ف‬
َ ‫ماِيَ أن‬ َّ َ‫وأ‬
َ ِ‫ما‬ َ
{ 207 }

817) Evaluation
-a systematic process by which the worth or value of something is judged and a basis
for those judgments is defined. -a continuous feedback process whereby information is
used to guide and direct decisions. -requires measurement.

818)Surveillance
Systematic continuous observation of populations, and collection and analysis of data
from many varied sources. FOR: Rapid detection and timely, appropriate response to
important health events. Social surveillance for public health in community. Health
surveillance for hospitals and health facilities.

819) Incubation period


Person is most infectious and nonspecific signs and symptoms.

820) Hirschsprung disease


Absence of ganglion cells of intestine. OR is a congenital condition characterized by
partial or complete colonic obstruction associated with the absence of intramural
ganglion cells. Because of the ganglionitis,

821) Placenta previa


Occurs when the placenta abnormally implants in the lower segment of the uterus near
o over the cervical os instead of attaching to the fundus.

822)Abruptio placenta
Premature Separation of a normally implanted placenta before the fetus is born.

823)OVR report.
Incident reports write on clear legible handwriting.

ِ‫يِاْلَ أرض‬
‫أ‬ ُ ‫َم‬
ُ ‫ك‬
‫ثِِف‬ َ ‫اس‬
‫ِِفي أ‬َ َّ ‫ع‬
‫ِالن‬ ُ ‫ف‬
َ ‫ماِيَ أن‬ َّ َ‫وأ‬
َ ِ‫ما‬ َ
{ 208 }

824)Nosocomial infection
Hospital acquired infection.

825)Blood pressure
Force of blood against arterial wall

826)Blood deficit
The difference between apical pulse and and peripheral pulse

827)Fusion process
The process that sperm starting penetrates the ovum.
Process of egg and sperm fusing to form a new individual.

828)Capacitation process
Switching on of sperm to become fertilizing; it’s a biochemical event as sperm
essentially demonstrates expected morphology and motility.
This step is a biochemical event the sperm move normally and look mature prior to
Capacitation.

829) Pulse pressure


The difference between systolic blood pressure and diastolic blood pressure

Cranial Nerves
‫اسم العصب‬ ‫الوظيفة‬ ‫األدوات المستخدمة‬
I -Olfactory ‫االول‬ smell sensory Coffee or soap
II- Optic ‫الثاني‬ Vision distances sensory Vision distances chart
chart and cotton. Torch
III-Oculomotor ‫الثالث‬ Eyelid elevation motor

ِ‫يِاْلَ أرض‬
‫أ‬ ُ ‫َم‬
ُ ‫ك‬
‫ثِِف‬ َ ‫اس‬
‫ِِفي أ‬َ َّ ‫ع‬
‫ِالن‬ ُ ‫ف‬
َ ‫ماِيَ أن‬ َّ َ‫وأ‬
َ ِ‫ما‬ َ
{ 209 }

IV-Trochlear ‫الرابع‬ Turns eyes motor


downward and
laterally
V- trigeminal ‫الخامس‬ Chewing facial both Pinprick and brushing
and mouth teeth & Hammer for
sensation jaw reflex
VI-Abducens ‫السادس‬ Turns eye Motor
laterally
VII-Facial ‫السابع‬ Facial expression both sweet & salt &
and lip closure hammer
VIII- ‫الثامن‬ Hearing sensory The weber and Rinne
vestibulocochlear equilibrium test) tuning fork
IX- ‫التاسع‬ Gaging and both tongue depressor &
glossopharyngeal swallowing penlight
X-vague ‫العاشر‬ Gaging and both tongue depressor &
swallowing penlight
XI-Accessory ‫الحادي عشر‬ Shoulder motor tongue depressor
movement and
head rotation
XII-Hypoglossal ‫الثاني عشر‬ Tongue motor tongue depressor
movement and
speech

ِ‫يِاْلَ أرض‬
‫أ‬ ُ ‫َم‬
ُ ‫ك‬
‫ثِِف‬ َ ‫اس‬
‫ِِفي أ‬َ َّ ‫ع‬
‫ِالن‬ ُ ‫ف‬
َ ‫ماِيَ أن‬ َّ َ‫وأ‬
َ ِ‫ما‬ َ
{ 210 }

830) A 29-year-old man is in the Surgical Ward on his first post-open thyroidectomy. He
appears drowsy but he is.
able to respiration by nodding head. He is developing mild dy restlessness. What is the
initial recommended goal?
of care?
A. Monitor vital signs of thyroid storm
B. Assess for bilateral vocal fold mobility.
C. Monitor for swelling on the neck.
D. Monitor for vocal cord paralysis
Answer: B

ِ‫يِاْلَ أرض‬
‫أ‬ ُ ‫َم‬
ُ ‫ك‬
‫ثِِف‬ َ ‫اس‬
‫ِِفي أ‬َ َّ ‫ع‬
‫ِالن‬ ُ ‫ف‬
َ ‫ماِيَ أن‬ َّ َ‫وأ‬
َ ِ‫ما‬ َ
{ 211 }

831)The Barlow Maneuver is a physical examination performed on infants to screen


for developmental dysplasia of the hip. Barlow's test identifies posterior sublimations or
dislocation. Ortolani test also for DDH

ِ‫يِاْلَ أرض‬
‫أ‬ ُ ‫َم‬
ُ ‫ك‬
‫ثِِف‬ َ ‫اس‬
‫ِِفي أ‬َ َّ ‫ع‬
‫ِالن‬ ُ ‫ف‬
َ ‫ماِيَ أن‬ َّ َ‫وأ‬
َ ِ‫ما‬ َ
{ 212 }

Answer: C
832)Nursing student training under the supervision of Staff Nurse
3-month-old infant came to a normal checkup and the student assessed the pulse it is
165 b/m
What should you do?
A. Inform the doctor
B. Ask parents about a history of heart disease.
C. The staff nurse should check all the vital signs and repeat assessment✅

D. Ask the student nurse to check child's file.


Answer: C
833)A child with a diagnosis of tetralogy of Fallot is scheduled.
to be discharged from the hospital the nurse planning
discharge education should instruct the caregivers that.
during a hyper cyanotic spell, the position MOST likely to
benefit the child is:
Supine
Side-lying
Prone
Knee-chest✅

Answer: D

ِ‫يِاْلَ أرض‬
‫أ‬ ُ ‫َم‬
ُ ‫ك‬
‫ثِِف‬ َ ‫اس‬
‫ِِفي أ‬َ َّ ‫ع‬
‫ِالن‬ ُ ‫ف‬
َ ‫ماِيَ أن‬ َّ َ‫وأ‬
َ ِ‫ما‬ َ
{ 213 }

834) Nurse manager tells one of the staff, “I don’t have time to discuss the matter with
you now. See me in my office later” when the latter asks if they can talk about an issue.
Which of the following conflict resolution strategies did she use?
A. Smoothing
B. Compromise
C. Avoidance✅
D. Restriction
Answer: C
835)A COPD patient siting and leaning forward, what should be improve to patient?
A. position
B. oxygen exchange ✔️

C.respiration pattern
Answer: B
836)Collection of air in pleural space?
A. pneumothorax✔️

B. hemothorax
C. Atelectasis
C. Pleural effusion
Answer: A
837)Patient has barrel chest, dyspnea and pink skin. what should the nurse expect
another symptom for patient?
A. Obese
B. Thin✅

Answer: B
838)An 8-month-old patient has gastroenteritis, and He is treated, now what do you
instruct the mother about?
A. weaning

B. Nutritional requirements✅

ِ‫يِاْلَ أرض‬
‫أ‬ ُ ‫َم‬
ُ ‫ك‬
‫ثِِف‬ َ ‫اس‬
‫ِِفي أ‬َ َّ ‫ع‬
‫ِالن‬ ُ ‫ف‬
َ ‫ماِيَ أن‬ َّ َ‫وأ‬
َ ِ‫ما‬ َ
{ 214 }

C. Toilet training
D. Accidents
Answer: B
839)Mothers of children aged 1-4 years, what should you teach them about for their
child safety?
A. Pick up medicine and sharps✅

B. Don't use violence because it turns out to be a homicide.


C. Accidents
Answer: A
840)Infant came to ER and during the examination, the doctor said he had spinal bifida
occult. What does the doctor know about this condition?

A. Infant has hair patch at back✅


B. Infant has open wound on back.
Answer: A

841)How much does a full-term baby weigh?

2.5-4 Kg
842) NICU child, when his mother came out, he sat shouting, and crying at Which stage
of separation?
A. Despair
B. Denial
C. Protest✅
D. Anxiety
Answer: C
843) Which option helps the mother in the taking in stage?

A. let her talk about her experience✅

B. Let her take care of the baby independent


C. Give her educational papers and instruction

ِ‫يِاْلَ أرض‬
‫أ‬ ُ ‫َم‬
ُ ‫ك‬
‫ثِِف‬ َ ‫اس‬
‫ِِفي أ‬َ َّ ‫ع‬
‫ِالن‬ ُ ‫ف‬
َ ‫ماِيَ أن‬ َّ َ‫وأ‬
َ ِ‫ما‬ َ
{ 215 }

Answer: A

844)Vaccine temperature 2-8° c


845)-Patient with blood type AB-ve for blood transfusion what are the compatible blood
group for this patient:
A-
B-
O-
AB-✅

Answer: D
846)-nurse manager told his staff if one of you demonstrate punctuality, will have extra
day off, what are the method of motivation used by the nurse manager?
A. Coercive
B. Rewarding✅

Answer: B
847) The healthcare provider is caring for a patient with a diagnosis of first- degree
atrioventricular (AV) block. Which of these waveform patterns identified on the
cardiac monitor is consistent with this arrhythmia?
A. QRS complexes are dropped randomly
B. No association between the P waves and QRS complexes
C.Significant shortening of the PR interval
D. Slowed conduction through the AV node✅
Answer: D
848) When the nurse starting change dressing procedure. What is the first step for her?
A. Wear mask
B. Wear gown
C. Sterile gloves✅

Answer: C
849)pregnant women G1 P0 vaginal delivery observed in the second postpartum day
that the perineal pad.

ِ‫يِاْلَ أرض‬
‫أ‬ ُ ‫َم‬
ُ ‫ك‬
‫ثِِف‬ َ ‫اس‬
‫ِِفي أ‬َ َّ ‫ع‬
‫ِالن‬ ُ ‫ف‬
َ ‫ماِيَ أن‬ َّ َ‫وأ‬
َ ِ‫ما‬ َ
{ 216 }

saturated with bright red lochia rubra what is the priority nursing intervention?
A. Massage fundus
B. Obtain vital signs.
C. Inform physician
D. Inquire about time of pervious saturated perineal pads.
Answer: D
850)A patient with sever varicose veins of the left leg presents to the clinic. The patient
states that three days
ago the right leg became very swollen and the skin on the right half area was very
darkly colored. The capillary
refill in the fingers is three seconds and on the right toes is four seconds.
A. Teach the patient self-blood monitoring.
B. Assess the patient to sided heart failure.
C. Encourage the patient to joint an exercise class.

D. Teach the patient how to apply thigh high anti embolic stocking✅

Answer: D
851)What is the most common hazard in hospitals?
A. Chemical
B. Physical✅
C. Biological
D. Psychological
Answer: B
852)Patient with bipolar disorder / schizophrenia. What should the nurse ask the patient
to assess orientation for him?
A. Family history
B. The last events in country

C. Date, Time, Person name✅


D. Months from back to first
Answer: C

ِ‫يِاْلَ أرض‬
‫أ‬ ُ ‫َم‬
ُ ‫ك‬
‫ثِِف‬ َ ‫اس‬
‫ِِفي أ‬َ َّ ‫ع‬
‫ِالن‬ ُ ‫ف‬
َ ‫ماِيَ أن‬ َّ َ‫وأ‬
َ ِ‫ما‬ َ
{ 217 }

853)The head nurse needs to increase her staff nursing the next year. What is this
considering??
A. Selection
B. Recruitment
C. Downsizing
D. Rightsizing ✅✅

Answer: D
854)The head nurse needs to increase her staff nursing the next year. What is this
considering??
A. Selection
B. Recruitment
C. Downsizing
D. Organizing ✅✅

Answer: D
855) A nurse cares for a group of clients in a long-term care facility.
Which situation represents a situation in which the nurse supports the
client's autonomy?
A.A client falls and fractures a hip. The nurse contacts the health care.
provider for a prescription for pain medication prior to transfer for
treatment.
B.A client reports to the nurse regarding observing staff smoking on
facility grounds when it was banned for residents and family members.
C.A competent client who has received a terminal diagnosis request.
the nurse to not reveal the diagnosis to the family due to fear of them.
seeking long-term mechanical ventilation.
D. A client wishes to have a do not resuscitate (DNR) order to
prevent heroic measures by the health care team in the event of
cardiac or respiratory arrest. ✅

ِ‫يِاْلَ أرض‬
‫أ‬ ُ ‫َم‬
ُ ‫ك‬
‫ثِِف‬ َ ‫اس‬
‫ِِفي أ‬َ َّ ‫ع‬
‫ِالن‬ ُ ‫ف‬
َ ‫ماِيَ أن‬ َّ َ‫وأ‬
َ ِ‫ما‬ َ
{ 218 }

Answer: D

856) A nurse cares for a group of clients in a long-term care facility.


Which situation represents a situation in which the nurse supports the client’s autonomy?
A. Patient refused to get I. V line
B. Doctor order for patient no CPR but patient insists to do CPR ✅

Answer: B
857)Patient came to ER with dyspnea, Heart rate 136, Respiration rate 29. After doing x-
ray the patient diagnosed lung abscess. Which Unit should the nurse have admitted the
patient?
A. ICU

B. Medical, Surgical ward✅


C. Cardiac ward
Answer: B
858) The patient with dementia and complaining of memory deficit. When the nurse
enters patient room. She found the patient on the floor beside the bed. When she
asked him What happened he said I forgot to press the call bell. She applied her
nursing diagnosis and set intervention for that. See the table includes
Nursing diagnosis:
_Risk for fall related to cognitive impairment
_Steps for intervention (Nurse care plan) :
_Remind patient to use call bell continuously
_Immediate Response to the patient after calling
_Don't wait while patient is asking
What is the most appropriate action?
A. Continue for applying care plan that is enough interventions
B. Change care plan and add more intervention to punish patient and provide
restrain for him to avoid reoccurrence.
C. Change intervention to add bed sensor in the side rails for quick response and
provide bedside cane If he forgot call bell✅

ِ‫يِاْلَ أرض‬
‫أ‬ ُ ‫َم‬
ُ ‫ك‬
‫ثِِف‬ َ ‫اس‬
‫ِِفي أ‬َ َّ ‫ع‬
‫ِالن‬ ُ ‫ف‬
َ ‫ماِيَ أن‬ َّ َ‫وأ‬
َ ِ‫ما‬ َ
{ 219 }

Answer: C
859)A 19-year-old boy has been hospitalized with fracture in upper and lower
extremities after accident then provided with casts for upper and lower limbs. Which of
these Nursing diagnoses should the nurse consider in the Nursing plan of care
According to his age??
A. Impaired social interaction
B. Alteration in body image✅

C. Risk for infection


D. Anxiety
Answer: B

Definition of hemiparesis /weakness of one entire side of the body


Osteoma / to remove waste from body.
860)DM patient at night shift with cold skin, tachycardia, diaphoresis what you will do
fist?
A. check blood glucose.

B. give him cup of orange juice✅


Answer: B
861) Nurse said to patient ... your identity will not be linked to responses... What's the
principal?

A. Privacy

B. Confidentiality

C. Anonymity ✅

D. Debrifeing
Answer: C

complication of COPD: (core pulmonale)


862)Patient with Crackle, wheezing, fever and dull sound after percussion. What is the
Diagnosis?
A. Pneumonia✅

ِ‫يِاْلَ أرض‬
‫أ‬ ُ ‫َم‬
ُ ‫ك‬
‫ثِِف‬ َ ‫اس‬
‫ِِفي أ‬َ َّ ‫ع‬
‫ِالن‬ ُ ‫ف‬
َ ‫ماِيَ أن‬ َّ َ‫وأ‬
َ ِ‫ما‬ َ
{ 220 }

B. Tuberculosis
C. Cor pulmonale
Answer: A
863)Patient has ulcers in his foot. What is most important nursing diagnosis??
A. High risk for injury✅

B. Altered body image.


Answer: A
864)COPD patient attached on mechanical ventilation. We need to change his
position to prevent bed sores. What is the most appropriate position?
A. Supine with elevation head of bed

B. Lateral with elevated HOB 40 degree✅

C. Setting position
Answer: B
865)What should the nurse do post test for the patient after IVP intravenous pyelogram?
A. Doing exercise
B. Encourage oral intake fluids✅✅
C. Eat something
Answer: B
866)Woman came to ER with euphoric mood, hyperactive and talkative. But after
physical examination and assessment she does not has any psychotic symptoms. The
symptoms of hyperactivity and talkative not effect on her social life or job. What is the
appropriate diagnosis?
A. Dysesthesia

B. Mania✅
C. Mood disorder
Answer: B
868)Woman with severe pain in left iliac fossa and tenderness. They request for her CT.
What should the nurse expect on CT??
A. Sigmoid diverticulosis✅

B. Sanitary diverticulosis

ِ‫يِاْلَ أرض‬
‫أ‬ ُ ‫َم‬
ُ ‫ك‬
‫ثِِف‬ َ ‫اس‬
‫ِِفي أ‬َ َّ ‫ع‬
‫ِالن‬ ُ ‫ف‬
َ ‫ماِيَ أن‬ َّ َ‫وأ‬
َ ِ‫ما‬ َ
{ 221 }

C. Diverticolum
Answer: A
869)Nurse supervising on assistant nurses and delegate assignment to him and say
“diving up patient's assignment."
what type of leadership
A. Autocratic
B. laissez-faire
C. Situational
D.transformational2
Answer: B

870)A 46-year-old patient is in the male Urology Ward after the surgical removal of the
stone from his left kidney.
through percutaneous nephrolithotomy under general anesthesia He

ِ‫يِاْلَ أرض‬
‫أ‬ ُ ‫َم‬
ُ ‫ك‬
‫ثِِف‬ َ ‫اس‬
‫ِِفي أ‬َ َّ ‫ع‬
‫ِالن‬ ُ ‫ف‬
َ ‫ماِيَ أن‬ َّ َ‫وأ‬
َ ِ‫ما‬ َ
{ 222 }

has nausea and dull aching pain in left lumbar region His nephrostomy.
bag is attached through a tube in his left kidney for a few days (see
image). What of the following problems is is needed to focus on?

A. Risk of impaired skin integrity due to infection✅

B. Disturbed life cycle related to nephrostomy bag.


C. Knowledge deficiency for self-care management
D. Impaired social interaction due to altered lifestyle
Answer: A
871)46-year-old patient is in the male Urology Ward after the surgical removal of the
stone from his left kidney through percutaneous nephrolithotomy under general
anesthesia. He has nurse and dull acting pain in left lumbar region. His nephrostomy
bag is attached through a tube in his left kidney for a few days (see image)
What findings should alert the nurse to report to the physician immediately?
A. Abdominal discomfort
B. Patient Very exhausted
C. Presence of blood and stone gravels in urine
D. Urine output less than 30 ml/hr.✅
Answer: D
872) Pregnant mother always delivery on 36 weeks. She was asking the nurse What
to do to prevent that. May be because of frequent sexual intercourse. How many
times should the nurse provide education for her?
A. Once per week✅
B. Once per month
Answer: A

873)Patient came to ER after motor accident. The patient diagnosed hemiparesis


/hemiplegia. Which part of spinal cord injury is affected?
A. Cervical✅

B. Lumbar
C. Thoracic
D. Sacral
Answer: A

ِ‫يِاْلَ أرض‬
‫أ‬ ُ ‫َم‬
ُ ‫ك‬
‫ثِِف‬ َ ‫اس‬
‫ِِفي أ‬َ َّ ‫ع‬
‫ِالن‬ ُ ‫ف‬
َ ‫ماِيَ أن‬ َّ َ‫وأ‬
َ ِ‫ما‬ َ
{ 223 }

874)Patient has sever itching all over the body. What medication should the nurse give?
A. Antibiotic

B. Diphenhydramine✅

C. Zinc Oxide
Rational Antihistamine
Answer: B

875)Patient came to ER complain from blood in stool. What is the most common
diagnosis??
A. GERD
B. Gastroenteritis
C. BID bowel inflammatory disease ✅

Answer: C
876. Quinine sulphate drug side effects:

A. Ringing in ears ✅

B. Blindness

C. Hypotension

D. Insomnia

Answer: A
877. Measurements rates of epidemiology.. rate of birth in the year of 2016 is 19.5 per
1000 .. what is it considered?

A. Crude rate ✅

B. Observation rate

C. Specific rate

D. Standardized rate

Answer: A

ِ‫يِاْلَ أرض‬
‫أ‬ ُ ‫َم‬
ُ ‫ك‬
‫ثِِف‬ َ ‫اس‬
‫ِِفي أ‬َ َّ ‫ع‬
‫ِالن‬ ُ ‫ف‬
َ ‫ماِيَ أن‬ َّ َ‫وأ‬
َ ِ‫ما‬ َ
{ 224 }

878. Schizophrenic patients says “ i feel like my arms and limbs are deatcahed from my
body and i don’t feel like i am a alive and it made me avoid socializing” the nurse
interpreted this as :

1. Depersonalization ✅

2. Loss of association

3. Dissociative identity disorder

4. Dissociative amnesia

Answer: A
879. The nurse is monitoring a post surgery patient, she noticed that his urine output has
declined from 50 ml to 30 ml, she interprets this as witch cause?

1. Anesthesia effects ✅

Answer: A
880. Which patient is a contraindicated for enema??

A. Gluocoma

B. hypertensive

C. renal failure

D. liver disease

Answer: C
881. A 48yearold patient in the male Surgical Ward had his gall
bladder removed through laparoscopic cholecystectomy 24 hours ago. While
evaluating his general condition, the patient appears lethargic and complains of
severe nauseated feeling along with disco mfort in the
abdomen. What nursing problem needs to be prioritized?

A. Disturbed metabolism due to higher energy demand

B. Weak and lethargic due to low food and fluid intake

C. Nausea and vomiting due to slower gut movement ✅

D. Impaired comfort related to postsurgical effects

Answer: C

ِ‫يِاْلَ أرض‬
‫أ‬ ُ ‫َم‬
ُ ‫ك‬
‫ثِِف‬ َ ‫اس‬
‫ِِفي أ‬َ َّ ‫ع‬
‫ِالن‬ ُ ‫ف‬
َ ‫ماِيَ أن‬ َّ َ‫وأ‬
َ ِ‫ما‬ َ
{ 225 }

882)Patient with Manic bipolar disorder. He is hyperactive, too much moving, talk alot
and he has loss of appetite, very thin . The patient also don't sleep. What is the most
appropriate nursing diagnosis?
A. High risk for suicide
B. High risk for injury✅

C. Nutritional disturbance
D. Alter sleep pattern
Answer: B
883)Psychiatric patient was crying. When the nurse asked him, He said I want to stay
alone. I don’t want anyone with me leave me cry. What is the best action for the nurse?
A. Maintain privacy and leave him alone

B. You appear sad. I am here to help you✅

C. Stay with him and be silent


D. Say Okay and let him cry
Answer: B
884) The nurse was transporting patient with H1N1. She was asking another nurse for use
of N95 with this patient. What is the action?
A. Use the mask only inside patient room✅

B. Use the mask if patient has cold symptoms


Answer: A
885)The nurse was discussing with group of women and providing educational session
about breast self-examination. Which of the following if said need furthur education?
A. Breast examination after age of 20 years' old✅

B. Doing the examination after the shower


C. Doing the examination either standing or sleeping
D. Doing the examination 5/6 days after period.
Answer: A

ِ‫يِاْلَ أرض‬
‫أ‬ ُ ‫َم‬
ُ ‫ك‬
‫ثِِف‬ َ ‫اس‬
‫ِِفي أ‬َ َّ ‫ع‬
‫ِالن‬ ُ ‫ف‬
َ ‫ماِيَ أن‬ َّ َ‫وأ‬
َ ِ‫ما‬ َ
{ 226 }

886)Patient with 88% O2 saturation What should use?

A. Facial mask✅

A. Nasal cannula
Answer: A
887)The patient in ICU weaned from ventilator and start endotracheal tube. What
should the nurse use?

A. Ventru mask✅

B. trachestomy collar
Answer: A
888)Which of the following vitamin supplements can decrease the incidence of Neural
tube defects such as anencephaly and spina bifida new-born or congenital
anomalies?
A. Vitamin A
B. Riboflavin

C. Folic Acid✅

D. Vitamin K
Answer: C
889) While caring for a neonate with a meningococcal, the nurse should avoid
positioning the child on the:
A. Abdomen
B. Left side
C. Right side
D. Back✅

Answer: D
890) A nurse is assessing a 6-month-old infant that has retar reduced responsiveness and
interaction with the environment to smile or make eye contact. The nurse notices that
the attempt to hold or comfort the crying infant. What diagnosis should the nurse
anticipate?
A. Celiac disease
B. Failure to thrive✅

ِ‫يِاْلَ أرض‬
‫أ‬ ُ ‫َم‬
ُ ‫ك‬
‫ثِِف‬ َ ‫اس‬
‫ِِفي أ‬َ َّ ‫ع‬
‫ِالن‬ ُ ‫ف‬
َ ‫ماِيَ أن‬ َّ َ‫وأ‬
َ ِ‫ما‬ َ
{ 227 }

C. Cystic fibrosis
D. Growth hormone deficiency
Answer: B
891)When assessing the funds level At 36 week it is at the >>>> Xiphoid process
Midway between symphysis pubis and Umblical level >>>> 16 week
At the level of Umblicus >>>>>> 20 week
At symphysis pubis level>>>>> 12 week
892)A 28-year-old pregnant woman at 9 weeks presents to the o with vaginal bleeding.
During assessment, the nurse found height is 12cm. Which of the following is the most
likely diagnosis?
A. Placenta previa
B. Abruptio placenta
C. Ectopic pregnancy
D. Hydatidiform mole✅

Answer: D
893)Orthostatic hypotension:

A - 80/60.✅
B- 90/70.
C- 110/75.
Answer: A
894)while taking care of the a patient with a spinal cord injury, the patient suddenly
complains of pounding headache upon assessment the patient was found to have
diaphoresis , drop in heart and elevated blood pressure , autonomic dysreflexia is
suspected and the head on the bed is elevate . Which of the following is the most
appropriate to important immediately?
A-Notify the physician
B-Assess bladder for distension ✅

C-Continue to monitor for next hour


Answer: B

ِ‫يِاْلَ أرض‬
‫أ‬ ُ ‫َم‬
ُ ‫ك‬
‫ثِِف‬ َ ‫اس‬
‫ِِفي أ‬َ َّ ‫ع‬
‫ِالن‬ ُ ‫ف‬
َ ‫ماِيَ أن‬ َّ َ‫وأ‬
َ ِ‫ما‬ َ
{ 228 }

895)During vaginal examination the nurse palpated the posterior fontanel to be at the
right side and upper quadrant of the maternal pelvis ?
A. ROP
B. LOP.

C. ROA✅

D. LOA
Answer: C
896)The nurse is assessing a child (an infant) with pyloric stenosis. which of the following
is likely to note?
A. Diarrhea
B. Projectile vomiting✅

C. Swallowing difficulties
D. Currant jelly like stool
Answer: B
897) A 5- week old newborn admitted paediatric ward with pyloric stenosis the
newborn has weight loss, and
projectile vomiting after feeing. Which of the following abdominal organs are directly
affected when stenosis diagnosed?
A. Stomach and duodenum✅

B. Stomach and esophagus


C. Liver and spleen
D. Liver and bile duct
Answer: A
898) A 5year-old child was seen to the Emergency Department abdominal pain,
palpable sauge-shaped mass, and Intussusception is suspected Which of the following
is the best diagnostic evaluation to?
A. X-ray
B. endoscopy
C. Rectal biopsy
D. Ultrasonograph✅
Answer: D

ِ‫يِاْلَ أرض‬
‫أ‬ ُ ‫َم‬
ُ ‫ك‬
‫ثِِف‬ َ ‫اس‬
‫ِِفي أ‬َ َّ ‫ع‬
‫ِالن‬ ُ ‫ف‬
َ ‫ماِيَ أن‬ َّ َ‫وأ‬
َ ِ‫ما‬ َ
{ 229 }

899)57- A 30-year-old pregnant lady in her 33 weeks' pregnancy. When the nurse
assesses the health condition of the lady, she provides nursing care and health
teaching. Which of the following types of home visiting is the best to be conducted?

A. Follow up✅

B. Selective
C. Systematic routine
Answer: A
900) Complication of hyperthyroidism?
A. Bleeding

B. Thyrotoxicosis✅

Answer: B
901)A Rehabilitation nurse reviews post stroke patient immunization history
which immunization is a priority for a 72-year-old patient
A. hepatitis A vaccine
B. hepatitis B vaccine
C. rotavirus vaccine
D. pneumococcal vaccine
Answer: D
902) Pregnant woman in third trimaster is complaining fatigue, tired and chest
heavieness and dificulty breath. The nurse told her that is normal specially in the last
months of pregnancy. What is the Medical cause and interpretation filor that?

A. The fetus and uterus on exophoid and press on lung and limit its expand ✅
Answer: A
903)Polycystic ovary syndrome (PCOS) Complications?
A. Amenorrhea
B. Dysmenorrhea
C. Infertility✅
Answer: C
904)Polycystic ovary syndrome (PCOS) Common Signs and symptoms?
A. Dysmenorrhea

ِ‫يِاْلَ أرض‬
‫أ‬ ُ ‫َم‬
ُ ‫ك‬
‫ثِِف‬ َ ‫اس‬
‫ِِفي أ‬َ َّ ‫ع‬
‫ِالن‬ ُ ‫ف‬
َ ‫ماِيَ أن‬ َّ َ‫وأ‬
َ ِ‫ما‬ َ
{ 230 }

B. Infertility✅

C. Polymenorrhea
Answer: B
905)Nursing manager her scheduled staff absence. She had to float another staff to
cover defect from another unit What is the manner that manager use?
A. self-staffing
B. on call staffing
C. Borrowing staffing

D. Floating staffing✅
Answer: D
906)When nursing manager invite unscheduled staff from same unit to cover nursing
shortage. What this manner?
A. Self-staffing

B. On call staffing✅
Answer: B
907)Patient with fever, cough with blood. The nurse call to infection center to provide
precaution to the patient. Which precaution should the nurse Apply?
A. Airborne✅

B. Droplet
C. Contact
D. Precaution until the confirm of diagnosis
Answer: A
908) A group of mothers, in their third trimesters, are attending the education session
regarding the alert signs for which they are immediately contact the hospital. The
midwife is to assess the understanding of the session by asking question. What response
for the mothers helps assure the midwife about right understanding?
A. Gush of urinary outflow without any urge
B. Low back pain at night and difficulty in sleep
C. Activity intolerance and breathlessness on exertion

ِ‫يِاْلَ أرض‬
‫أ‬ ُ ‫َم‬
ُ ‫ك‬
‫ثِِف‬ َ ‫اس‬
‫ِِفي أ‬َ َّ ‫ع‬
‫ِالن‬ ُ ‫ف‬
َ ‫ماِيَ أن‬ َّ َ‫وأ‬
َ ِ‫ما‬ َ
{ 231 }

D. Absence of contraction after 42 week of pregnancy✅

Answer: D
909)What is the Assessment of data after patient discharge??
A. Audit
B. Retrospective✅

C. Concrete
Answer: B
910) Patient post heart valve replacement. What is the expected drug for the patient
after operation?
A. Nitrates
B. Beta blockers

C. Anticoagulant✅

Answer: C
911) A patient with a history of atrial fibrillation has an order for 0.25 milligrams of drug
(X), the available supply/dose is
0.125 milligrams.
Which of the following doses in correct?
A. Dispense two tablets to the patient ✅
B. Administer one tablet twice daily
C. Dispense one half tablet to the patient
D. Return tablets to the pharmacy and re-order
Answer: A
912) Patient Bl. P 230/135. What would patient high risk for??
A. Stroke✅

B. Hypertensive crisis
Answer: A
913) Why should control of high blood pressure with hypertensive patient. What is the
most common complication?

ِ‫يِاْلَ أرض‬
‫أ‬ ُ ‫َم‬
ُ ‫ك‬
‫ثِِف‬ َ ‫اس‬
‫ِِفي أ‬َ َّ ‫ع‬
‫ِالن‬ ُ ‫ف‬
َ ‫ماِيَ أن‬ َّ َ‫وأ‬
َ ِ‫ما‬ َ
{ 232 }

A. Hypertensive Encephalopcy✅

B. Cardiac decompensation
Answer: A
914) Patient with unstable angina. The chest pain is ruled out. What is the priority nursing
diagnosis?
A. Anxiety from threatened disease

B. Pain related to Angina pectoris✅

Answer: B
915) Self limited crisis Duration for solving ??
A. 2 weeks
B. 4 weeks✅

C. 6 weeks
D. 8 weeks
Answer: B
916) Mother has 8 months child wants to give her baby an egg to eat what is the kind of
egg she should give?
A. Give whole egg
B. Don’t give for child until 1 year
C. Give white egg without yolk

D. Give yolk egg without white✅

Answer: D
917) Patient on oxygen with nasal cannula. How should the nurse ensure effectiveness?
A. Increase HR to 90 b/m
B. Increase RR to 24
C. Increase O2 saturation 95%✅
D. Increase Bl. P 120/80 mmhg
Answer: C

918) A nurse manager is open minded, listens to the team, understands others, makes
changes to improve unit

ِ‫يِاْلَ أرض‬
‫أ‬ ُ ‫َم‬
ُ ‫ك‬
‫ثِِف‬ َ ‫اس‬
‫ِِفي أ‬َ َّ ‫ع‬
‫ِالن‬ ُ ‫ف‬
َ ‫ماِيَ أن‬ َّ َ‫وأ‬
َ ِ‫ما‬ َ
{ 233 }

operations and procedures. What type of leadership style is this?


A. Situational
B. Democratic ✅

C. Compassionate
D. Transformational
Answer: B
919) The Postpartum mother was asking the nurse about timing for restating sexual
intercourse activity. What should the nurse response?
A. 3 weeks after delivery
B. As long as taking contraceptives

C. After stop of lochia discharges✅

D. Any time she wants


Answer: C
920)Chicken box vaccine - - - - - - Varicella vaccine
Patient with COPD came to ER. The patient was very exhausted. He was complaining
shortness of breathing and cough. What is the most appropriate PaO2 percentage
should be given for him?
A. 55 or below mmhg✅
B. 80 or high
C. 70 mmhg
Answer: A
921) What is the best system that help nursing in data interpertation for patient care?
A. Patient documentation system✅

B. Nursing documentation system


Answer: A
922) Sulfa allergy patient should avoid - - - - - Sulfadiazine, sulfamethoxazole,
sulfapyridine & sulfisoxazole are Antibiotics.
923) Primary prevention - - - - prevent disease occurrence rather than cure

ِ‫يِاْلَ أرض‬
‫أ‬ ُ ‫َم‬
ُ ‫ك‬
‫ثِِف‬ َ ‫اس‬
‫ِِفي أ‬َ َّ ‫ع‬
‫ِالن‬ ُ ‫ف‬
َ ‫ماِيَ أن‬ َّ َ‫وأ‬
َ ِ‫ما‬ َ
{ 234 }

924) The nurse use internet to see patient file outside hospital?
A. Telehealth
B. Telecommunication✅

C. Nursing informatics
Answer: B
925)Child with pyloric stenosis and re-hydration. What should the nurse do?
A. Induces Vomiting for child
B. Give oral feeding

C. Start rehydration by nasogastric tube ✅

Answer: C
926)Patient in triage area. What is the first subjective data should triage nurse Obtain?
A. Chief compliance ✅

B. Level of pain
C. Family history
Answer: A
927) Person work in factory. Every day expose to hazard. What type of hazard?
A. Chemical✅

B. Environmental
C. Biological
D. Psychological
Answer: A
928) Psychiatric patient doesn’t like her nose and mouth shape. She has false belief of
disturbed body image and counselled many doctors' for doing operation. Doctor's not
accepted her due to no need everything is normal. What is type of disorder?
A. Pain
B. Conversion
C. Body dysmorphic disease✅
D. Hypochondriasis
Answer: C
929) A patient with congestive heart failure and severe peripheral edema has a

ِ‫يِاْلَ أرض‬
‫أ‬ ُ ‫َم‬
ُ ‫ك‬
‫ثِِف‬ َ ‫اس‬
‫ِِفي أ‬َ َّ ‫ع‬
‫ِالن‬ ُ ‫ف‬
َ ‫ماِيَ أن‬ َّ َ‫وأ‬
َ ِ‫ما‬ َ
{ 235 }

nursing diagnosis of fluid volume excess


What are the two MOST important interventions for the nurse to initiate?
a- Diuretic therapy and intake and output
b- Nutritional education and low-sodium diet
c- Daily weights and intake and output ✅✅

d- Low-sodium diet and elevate legs when in bed


Answer: C
930)Patient has phobia to get infection. He always do hand washing too much until his
hand injured from washing. What is the Diagnosis?
Obsessive compulsive disorder ✅

Answer: A
931)Psychiatric patient repeats unknown words for nurse but the patient can
understand it. What is the best term describing situation?
A. Word salad
B. Neologism ✅

C. Circumstantiality
D. Thought problem
Answer: B
932)Adult Patient (big Scenario with ECG finding at last situation) patient with HR : 55
b/m. What is the most appropriate Diagnosis?
A. Sinus Bradycardia ✅

Answer: A
933) Patient is experiencing Myocardial ischemia. The ECG is shown ST depression. What
is more changes in ECG the nurse expects?
A. T waves inversion✅

B. Tall T wave
Answer: A
934) Hemoglobin low + Platelets low - - - - - - Risk for bleeding

ِ‫يِاْلَ أرض‬
‫أ‬ ُ ‫َم‬
ُ ‫ك‬
‫ثِِف‬ َ ‫اس‬
‫ِِفي أ‬َ َّ ‫ع‬
‫ِالن‬ ُ ‫ف‬
َ ‫ماِيَ أن‬ َّ َ‫وأ‬
َ ِ‫ما‬ َ
{ 236 }

935) Patient in the Medical ward is experiencing chronic disease. The patient condition
become exacerbation. What this means?
A. The patient condition is improved
B. The Patient condition is worsen ✅

C. The patient condition is stable


Answer: B
936) Infant 2 months with patent ductus arteriosus. What is the expected signs and
symptoms for infant?
A. Acrocyanosis
B. Central cyanosis
C. Tachycardia and tachypnea ✅

D. Tachypnea
Answer: C
937) Late deceleration patterns are noted when assessing the monitor tracing of a
woman whose labor is being induced with an infusion of Pitocin. FHT go down from 140
b/m to 130 b/m. The woman is in a side-lying position. and her vital signs are stable and
fall within a normal range. Contractions are intense. last 90 seconds. and occur every 1
1/2 to 2 minutes. The nurse's immediate action would be to:
A. Change the woman’s position
B. Stop the Pitocin
c. Elevate the woman’s legs
D. Administer oxygen via a tight mask at 8 to 10 liters/minute
Answer: B
938)Psychiatric patient came to ER. The patient wrote on his arm bye tattoo. What is the
most appropriate diagnosis?
A. Borderline personality disorder ✅

B. Dissociative disease
C. Major manic episodes
Answer: A

ِ‫يِاْلَ أرض‬
‫أ‬ ُ ‫َم‬
ُ ‫ك‬
‫ثِِف‬ َ ‫اس‬
‫ِِفي أ‬َ َّ ‫ع‬
‫ِالن‬ ُ ‫ف‬
َ ‫ماِيَ أن‬ َّ َ‫وأ‬
َ ِ‫ما‬ َ
{ 237 }

939)Famous football player came to hospital. Many nurses want to get up his medical
information. The nurse refused ti give any data due to privacy. What is the best
effective explanation?

A. Viewing patients files for only valid authorized people ✅

Answer: A
940)The normal weight gain during pregnancy to mother BMI =19 ...?
A. 6.5 Kg - 10.5 Kg

B. 11.5 Kg 16 Kg ✅

C. 12 Kg - 18 Kg
Answer: B
941)surgical unit nurse assessed a 35 years old post appendectomy observe abdominal
distention with absent
bowel sounds. Which of the following interventions is most appropriate?
A. Encourage ambulation ✅

B. Provide liquid diet as tolerated


C. Administer hemolytic enema
D. Check surgical site for signs of infection
Answer: A
942)How can the nurse give health education to prevent or reduce sinusitis by health
education?
A. Increase carbohydrate intake
B. Increase fluids intake
C. Avoid travel with air plane ✅

Answer: C
943)Pregnant woman with hydatidiform mole. She came to ER for x-ray. What is the
signs that confirm diagnosis??
A. Grapes like clusters shape
Answer: A

ِ‫يِاْلَ أرض‬
‫أ‬ ُ ‫َم‬
ُ ‫ك‬
‫ثِِف‬ َ ‫اس‬
‫ِِفي أ‬َ َّ ‫ع‬
‫ِالن‬ ُ ‫ف‬
َ ‫ماِيَ أن‬ َّ َ‫وأ‬
َ ِ‫ما‬ َ
{ 238 }

944) The nurse begins a morning shift with the following see the doctor in the waiting
room (see table). 1. A first time mother who delivered two days ago and is having
Difficulties breastfeeding 2. A60 year-old man who has a history of deep vein
Thrombosis and is taking daily heparin 3. A76 year-old women who was hospitalized
three days Previously with pneumonia 4. A 56-year-old man who has used all his diuretic
pills Coughing up sputum Which patient should be seen first?
A. One
B. Two
C. Three
D. Four ✅
Answer: D
* Bonding
Relationship between parents and child from childbirth to few months after delivery
945) The nurse was taking history from the patient and she asked one question then
repeated that question again to patient. The patient said (Are you hearing me I
answered before). What should the nurse do to clarify?
A. Ask new question
B. Use eye contact during communication ✅

C. Ignore the patient


D. Repeat again the previous question
Answer: B
946) Post pleural effusion position during physical examination?

A. Setting ✅
B. Supine
Answer: A
947) Mother has child with Spastic cerebral palsy. The mother was asking the nurse
when her child be adult he will have mental retardation or impairment. What should the
nurse reply?
A. Who has cerebral palsy get more than 70% in IQ.
B. Who has epilepsy and CP Limited for them to have mental problem

ِ‫يِاْلَ أرض‬
‫أ‬ ُ ‫َم‬
ُ ‫ك‬
‫ثِِف‬ َ ‫اس‬
‫ِِفي أ‬َ َّ ‫ع‬
‫ِالن‬ ُ ‫ف‬
َ ‫ماِيَ أن‬ َّ َ‫وأ‬
َ ِ‫ما‬ َ
{ 239 }

C. Who has CP the main cause genetic factors.


D. Who has CP most of them has cognitive impairment with some mental
retardation✅

Answer: D
948)According to Ramsey sedation scale pt in icu on sedation on assessment patient
deep sleep and not response
to light what is score
A. 2
B. 3
C. 4
D. 6 ✅
Answer: D
949) The nurses in medical unit decided to make research study about fall. What is the
first step for doing?
A. Literature review ✅
B. Select research design
C. Research question
Answer: A
950) Multipara postpartum woman was experiencing hemorrhage. Which of the
following complication most common for her?
A. Infection
B. DVT
Answer: A
951)Nurse is caring for a client with a nasogastric tube that is attached to low suction
The nurse monitors the client, knowing that the client is at risk for which acid-base
disorder?
A. Metabolic acidosis
B. Metabolic alkalosis
C. Respiratory acidosis
D. Respiratory alkalosis
Answer: B

ِ‫يِاْلَ أرض‬
‫أ‬ ُ ‫َم‬
ُ ‫ك‬
‫ثِِف‬ َ ‫اس‬
‫ِِفي أ‬َ َّ ‫ع‬
‫ِالن‬ ُ ‫ف‬
َ ‫ماِيَ أن‬ َّ َ‫وأ‬
َ ِ‫ما‬ َ
{ 240 }

952) 40-year-old women patient with Parkinsonism Medical Ward. The patient stated
that she has the past two
weeks. The nurse was planning to Which type of diet is most suitable for parkin?
A. Solid
B. Liquid
C. Semi solid ✅✅

D. Clear liquid
Answer: C

953) What is the tool for skin assessment temperature?


A. Stethoscope
B. Thermometer
C. Back of fingers
Answer: C
954) A nurse manger is preparing and writing a plan for dealing disasters (code red).
Which procedure is the top
priority for the nurse manager the plan?
A. Infection control
B. Staff orientation ✅✅

C. Patient education
D. Patient relocation
Answer: B
955) Father always use violence with his child. What should the nurse expect the cause?
A. Financial status
B. His father abuse him when he was young ✅

Answer: B

ِ‫يِاْلَ أرض‬
‫أ‬ ُ ‫َم‬
ُ ‫ك‬
‫ثِِف‬ َ ‫اس‬
‫ِِفي أ‬َ َّ ‫ع‬
‫ِالن‬ ُ ‫ف‬
َ ‫ماِيَ أن‬ َّ َ‫وأ‬
َ ِ‫ما‬ َ
{ 241 }

956) Schizophrenic patient say ( I will eat fish, I will drink milk, I will go to the sea). What is
the most term for that?
A. Ward salad
B. Loss of association ✅

C. Neologism
Answer: B
957) Schizophrenic patient say (Sea, Airport, bus). What is the most term for that?
A. Ward salad ✅✅
B. Loss of association
C. Neologism
Answer: A
*Chicken box rash start from Trunk
*Measles rash start from Face
958) Multipara postpartum woman was experiencing hemorrhage. Which of the
following complication most common for her?
A. Infection
B. DVT
Answer: A
959) Which of the following from pregnents mothers is priority?
A. Vaginal bleeding
B. Eclampsia ✅

C. Diabetes
Answer: B
960)Which of the following drug contraindication during pregnancy?
A. Antibiotic
B. Analgesic
C. Steroids✅

Answer: C

ِ‫يِاْلَ أرض‬
‫أ‬ ُ ‫َم‬
ُ ‫ك‬
‫ثِِف‬ َ ‫اس‬
‫ِِفي أ‬َ َّ ‫ع‬
‫ِالن‬ ُ ‫ف‬
َ ‫ماِيَ أن‬ َّ َ‫وأ‬
َ ِ‫ما‬ َ
{ 242 }

961)Which of the following drug should the doctor give Instruction for patient when
using?
A. Antiemetic's
B. Steroids ✅

C. Antibiotic
D. Paracetamol
Answer: B
962) The charge nurse noticed that there is staff shortage during shift. For Whom should
the charge nurse report?

A. Supervisor ✅

B. Chief Nurse
C. Director office
Answer: A
963) Child post total adenoctomy / tonsillectomy position
A. Prone or side lying ✅

Answer: A
964) There is one Senior evening shift has been promoted to supervisor. What is the most
effective cause that helped to take decision and accept new position
A. Numbers of staffing during shift
B. Financial promotion
C. His Leadership skills ✅✅

Answer: C
965(The patient came to receive his Lab results from the nurse. The patient has
diagnosed with cancer. The patient said to nurse I am not believe that is my diagnosis,
that is not mine. What is the stage considering?
A. Denial✅

B. Anger
C. Depression
D. Bargaining

ِ‫يِاْلَ أرض‬
‫أ‬ ُ ‫َم‬
ُ ‫ك‬
‫ثِِف‬ َ ‫اس‬
‫ِِفي أ‬َ َّ ‫ع‬
‫ِالن‬ ُ ‫ف‬
َ ‫ماِيَ أن‬ َّ َ‫وأ‬
َ ِ‫ما‬ َ
{ 243 }

Answer: A
966) Scrub nurse was counting sheets during operation before doctor closing. The
doctor asked her to give surgical threads. What should the nurse do?
A. Don 'reply and Complete counting
B. Give him threads then Complete counting
C. Say for him wait I am still counting not finished yet ✅✅

Answer: C
967) Patient with active TB ...what about his family?

A. Carriers

B. Susceptible host ✅

Answer: B

968) How to teach a child to give insulin injection?


A. Teach him to give a stuffed animal
B. it is too early to teach him to give✅

Answer: B
‫ ولو موجود اني أعلم‬teach to parent ‫هتكون أصح‬

*Pancytopenia - - - - RBCs and platelet deficiency

969) The doctor order 600 ml of drug during 10 hours. How many minutes for 15 ml of the
drug?
A. 15✅

B. 20
C. 25
D. 30
Answer: A

ِ‫يِاْلَ أرض‬
‫أ‬ ُ ‫َم‬
ُ ‫ك‬
‫ثِِف‬ َ ‫اس‬
‫ِِفي أ‬َ َّ ‫ع‬
‫ِالن‬ ُ ‫ف‬
َ ‫ماِيَ أن‬ َّ َ‫وأ‬
َ ِ‫ما‬ َ
{ 244 }

*Schizophrenia Signs Hallucination and delusion

970) The child always fears from nurses while entering the room. What is the most
appropriate nursing diagnosis??
A. - Refusal effect
B- Anxiety related to exposure of strange environment
C- Anxiety / Fear related to medical staff✅

Answer: C
971) Woman came to ER with severe Vomiting. Vomiting not stop and the patient was
anxious and crying. She said I will die. What is the most appropriate nursing diagnosis?
A. Fear to unknown disease ✅

B. Distribution body image


Answer: A
972)Which of the following nurses best for providing support during labor?
A Nurse skilled to support ✅

B. Older nurse to support


C. Mother with Epidural no need for support
Answer: A
973) how to maintain patient free of infection during labor?
A. Use clean technique
B. Change pad
C. Limit vaginal examination ✅

Answer: C
974) The nurse care for a 60-year-old woman who history hypertension, hypothyroidism
and elevated cholesterol levels. She takes tablets daily for each of the health problem.
The doctor orders a routine dual- x-ray absorptiometry test that shows decrease bone
density. Which medication most likely contributed the test result?
A. Statins
B. Anti-hypertensive

ِ‫يِاْلَ أرض‬
‫أ‬ ُ ‫َم‬
ُ ‫ك‬
‫ثِِف‬ َ ‫اس‬
‫ِِفي أ‬َ َّ ‫ع‬
‫ِالن‬ ُ ‫ف‬
َ ‫ماِيَ أن‬ َّ َ‫وأ‬
َ ِ‫ما‬ َ
{ 245 }

C. Synthetic thyroid hormones ✅

D. Cholesterol absorption inhibitors


Answer: C
975) Infant with tracheoesophageal fistula TOF. What is the signs that need suction for
him?
A. Crying

B. Chocking and cyanosis ✅✅

C. Increase heart rate


Answer: B
976) How to prevent infant colic :
A- give infant warm shower then Trendelenburg position

B- Put a warm water bottle on your baby's belly✅

Answer: B
977) Why should the nurse while doing CPR ensure from complete chest recoil?
A - Allow the lung to expand ✅

B- increase cardiac output


Answer: A
978) What is the best nutrition teaching instruction for patient with
Parkinson disease?
A- Choking
B- Drooling ✅✅

C- Aspirations
D- Dysphasia
Answer: B
979) Position for lumbar puncture?!

A. Lateral recumbent position ✅

B. Prone position

ِ‫يِاْلَ أرض‬
‫أ‬ ُ ‫َم‬
ُ ‫ك‬
‫ثِِف‬ َ ‫اس‬
‫ِِفي أ‬َ َّ ‫ع‬
‫ِالن‬ ُ ‫ف‬
َ ‫ماِيَ أن‬ َّ َ‫وأ‬
َ ِ‫ما‬ َ
{ 246 }

C. Supine position
Answer: A
980) The relation between organism and environment is :

A. Ecology ✅✅

B. Pathology
C. Bacteriolog
D. Parasitolog
Answer: A
981) What is the treatment for Rheumatic Fever?

A. Antireumatic ✅

B. Anticoagulent
C. Corticol
Answer: A
982) During tonic clonic seizure patient moving and has difficulty of breathing and
cyanosis what is the appropriate nursing intervention:
A- support arm and extremities
B- put soft material under patient
C- put tongue depressed between teeth

D- protect patient head from injury ✅✅


Answer: D
983) Elderly patient referred to home health care department ...nurse observe home
environment for any harms .?

A. Benefiece

B. Non malifecnce ✅

C. Privacy

Answer: B

ِ‫يِاْلَ أرض‬
‫أ‬ ُ ‫َم‬
ُ ‫ك‬
‫ثِِف‬ َ ‫اس‬
‫ِِفي أ‬َ َّ ‫ع‬
‫ِالن‬ ُ ‫ف‬
َ ‫ماِيَ أن‬ َّ َ‫وأ‬
َ ِ‫ما‬ َ
{ 247 }

984) A pregnant mother at early pregnancy was admitted in Emergency Room with
leakage of amniotic fluid, vaginal bleeding and lower abdominal cramping pain. What
is the possible diagnosis should the nurse suspected?
A. Missed
B. Inevitable ✅

C. Incomplete
D. Threatened
Answer: B
985) The nurse is assigned to care for a patient with Paget’s dis complaining of bone
pain. Which of the following
conditions are this patient high risk for?
A. Hypomagnesaemia
B. Hypernatremia

C. Hypercalcemia ✅
D. Hyperkalemia
Answer:
986) Patient came to Rr after traffic accident and have sever bleeding ...The doctor
order she must enter operating room immediately.... what is appropriate actions to do
to obtain consent?

A. Call his husband to obtain telephone consent from him

B. Obtain order from nurse supervisor to enter operation room and do surgery

C. Enter pateint to operation immediately according to dr order without obtaining


consent ✅

D. Obtain consent from her friend who come with her to hospital

Answer: C
987) A 4 - year - old boy is admitted with diabetes insidious. The nurse is doing
assessment on the boy. Which of the following findings should the nurse anticipate?
A. bradycardia

B. excessive thirst ✅✅

ِ‫يِاْلَ أرض‬
‫أ‬ ُ ‫َم‬
ُ ‫ك‬
‫ثِِف‬ َ ‫اس‬
‫ِِفي أ‬َ َّ ‫ع‬
‫ِالن‬ ُ ‫ف‬
َ ‫ماِيَ أن‬ َّ َ‫وأ‬
َ ِ‫ما‬ َ
{ 248 }

C low urine outpatient


D. high systolic blood pressure
Answer: B
988) The patient with leukemia. There is development of large numbers of immature /
premature lymphocytes. What is the type of leukemia?
A. Acute ✔️✔️

B. Chronic
C. Leukopnia
Answer: A
989) Patient underwent for endoscopic GIT. What is the expected symptoms that
patient complain?
A. Fever
B. Sore throat ✅

C. Abdominal pain
Answer: B
990) Patient with SLE. What is the patient statement indicating that need furthered
education?
A. Exposure to sun will harm my skin
B- Hypertension must be anticipated

C- Hypertension is not common with SLE. ✔️✔️


D- The symptom will exacerbate more in winter and spring.
Answer: C
Intentional tort example - - - - - Battery

991) Nurse during in emergency situation ...dr order to administer medication verbally..

What nurse do ?

A. Write order repeat it to dr and administer

B. Repeat order again to dr and administer it ✅

ِ‫يِاْلَ أرض‬
‫أ‬ ُ ‫َم‬
ُ ‫ك‬
‫ثِِف‬ َ ‫اس‬
‫ِِفي أ‬َ َّ ‫ع‬
‫ِالن‬ ُ ‫ف‬
َ ‫ماِيَ أن‬ َّ َ‫وأ‬
َ ِ‫ما‬ َ
{ 249 }

C. Have another nurse to hear order and administer

Answer: B

992) A 40-year-old client underwent an exploratory laparotomy an aesthesia. An


assessment of the abdomen 36-hours showed abdominal distension and an absence of
bowel movement in all quadrants. Which complication is most likely?

A. Paralytic ileus ✅✅

B. Hemorrhage
C. Ruptured colon
D. Intussusception
Answer: A
993) The nurse is preparing a preoperative client for cholecystectomy and transfer him
to the operating room. The nurse should take which of the following actions at this time?
A. Skin preparation ✅
B. Patient identification
Answer: A
994) While transferring patient to operation room. Why should the nurse be applying
hospital policy during transferring?
A. Confidentiality

B. Continuity of care ✅
C. Ethical principles
Answer: B
995) A 32-year-old woman being hospitalized since six days for obesity treatment she
reported to the nurse “I feel burning in my left calf for the last two days “after nursing
assessment the nurse found that a left thigh edema, local tenderness, thigh
circumference is more by 8 cm than right thigh and it is warmer to touch, no trauma or
wounds. What is the most possible cause for that?
A. Obesity
B. Local cellulitis
C. Hypertension
D. Deep vein thrombosis ✅

ِ‫يِاْلَ أرض‬
‫أ‬ ُ ‫َم‬
ُ ‫ك‬
‫ثِِف‬ َ ‫اس‬
‫ِِفي أ‬َ َّ ‫ع‬
‫ِالن‬ ُ ‫ف‬
َ ‫ماِيَ أن‬ َّ َ‫وأ‬
َ ِ‫ما‬ َ
{ 250 }

Answer: D
996) A nurse is caring for a newborn in Well Born Nursery she warps the baby with
blanket and ensures the
nursey temperature is suitable for the babies. What type of heat loss is the nurse
preventing?
A. Radiation
B. Conduction
C. Convention ✅

D. Evaporation
Answer: C
997) Which of the following thermometer accuracy is affected by eating drinking and
mouth breathing?
A. Glass ✅
B. Chemical
C. Electronic
D. Tympanic
Answer: A
998)Patient is receiving chemotherapy. When should the nurse give antiemetic to avoid
side effect?
A. Before session ✅

B. During session
C. Half hour After session
Answer: A
999)Mother has inevitable abortion. What should the nurse monitor?

A. Hemorrhage ✅

B. Uterine contractions
Answer: A
1000)Child came to ER with projectile vomiting and dehydration. The child diagnosed
with pyloric stenosis. What should the nurse expect developing for?

ِ‫يِاْلَ أرض‬
‫أ‬ ُ ‫َم‬
ُ ‫ك‬
‫ثِِف‬ َ ‫اس‬
‫ِِفي أ‬َ َّ ‫ع‬
‫ِالن‬ ُ ‫ف‬
َ ‫ماِيَ أن‬ َّ َ‫وأ‬
َ ِ‫ما‬ َ
{ 251 }

A. Metabolic alkalosis

B. Metabolic acidosis ✅

C. Respiratory acidosis
Answer: B
1001)What is the signs should the nurse observed for preterm newborn during
assessment?
A-Ear cartilage well
B-Sole(foot)well deep crease
C-have more subcutaneous fat
D-Prominent labia minor and smaller labia major ✅

Answer: D
1002)Alzheimer's patient high risk for Fall and admitted with fractures hip during
hospitalization the patient fell twice. What should the nurse do?
A. Continue same plan
B. Impair skin integrity
C. Risk for pressure ulcer
D. Noncompliance add new nursing diagnosis ✔️✔️

Answer: D
1003)Patient with asthma. What is the most appropriate position?
A. Semi fowler
B. High Fowler ✅
Answer: B
1004)Smallpox isolation nurse wear?
A. Gloves and mask
B. Gloves and gown
C. Gloves and eye protector
D. Gloves, gown and N95 mask ✅

Answer: D

ِ‫يِاْلَ أرض‬
‫أ‬ ُ ‫َم‬
ُ ‫ك‬
‫ثِِف‬ َ ‫اس‬
‫ِِفي أ‬َ َّ ‫ع‬
‫ِالن‬ ُ ‫ف‬
َ ‫ماِيَ أن‬ َّ َ‫وأ‬
َ ِ‫ما‬ َ
{ 252 }

1005)The neonate after delivery with normal heart rate and normal respiration rate. The
neonate with Pink skin, active reflexes and Lusty cry. What is the most appropriate
Apgar score?
A. 9
B. 10 ✅

C. 8
D. 7
Answer: B
1006) Primipara mother ask nurse about importance of breastfeeding to her baby

Nurse response should be?

A. Lowers risk of asthma and allergies ✅

B. Lower risk of haemolytic diseae

C. Decreass rate of infant linear growth

D. Increase child abstract cognition

Answer: A
1007) A nurse is taking care of a patient who underwent abdominal surgery three days
ago. The patient has not been able to breathe deeply and uses to get out of bed since
the surgery due to pain. Also, the patient complains of shortness of breath, and the lung
sounds are diminished on auscultation.
Blood pressure 120/70 mmHg
Heart rate 75 /min
Respiratory rate 22 /min
Temperature 36.4 ℃
Oxygen saturation 89%
Which of the following conditions should the nurse suspect?
A. Sepsis

B. Atelectasis ✅

C. Congestive heart failure


D. Emphysema

ِ‫يِاْلَ أرض‬
‫أ‬ ُ ‫َم‬
ُ ‫ك‬
‫ثِِف‬ َ ‫اس‬
‫ِِفي أ‬َ َّ ‫ع‬
‫ِالن‬ ُ ‫ف‬
َ ‫ماِيَ أن‬ َّ َ‫وأ‬
َ ِ‫ما‬ َ
{ 253 }

Answer: B
1008)CSF contains NOT all of the following except:
A. Insulin. ✅

B. RBC.
C. WBC.
D. Protein.
Answer: A
1009)An elderly client is experiencing an alteration in his
equilibrium and coordinated muscle movements. The
nurse realizes that these functions are controlled by which
area of the nervous system?
a-Brain stem
b-Cerebrum
c-Diencephalon
d-Cerebellum ✔️
Answer: D
1010)A patient is having difficulty with cognitive abilities after a stroke. What part of the
brain was MOST likely affected?
a. Midbrain
b. Cerebrum ✔️

c. Medulla oblongata
d. Cerebellum
Answer: B
1011)10 years old child die is expected. What should the nurse do?
A. Withdraw medication die anyway
B. Give medication even if there is pain ✅

C. Withdraw medication to prevent pain

ِ‫يِاْلَ أرض‬
‫أ‬ ُ ‫َم‬
ُ ‫ك‬
‫ثِِف‬ َ ‫اس‬
‫ِِفي أ‬َ َّ ‫ع‬
‫ِالن‬ ُ ‫ف‬
َ ‫ماِيَ أن‬ َّ َ‫وأ‬
َ ِ‫ما‬ َ
{ 254 }

Answer: B
1012) 42 multpara arrive to ER on active labour with rupture of membranes and she
want to have mormal delivery without medication ...her husband come with her and
appear anxtious..

What is appropriate response from nurse?

A. Offer her non pharmacological method ...as she can do

B. Support her decision and offer non pharmacological method when needed ✅

C. Speak with her husband to convince her about other method...

D. Use pharmacological method

Answer: C
1013)Pregnant woman with cervical dilatation 6 cm and effacement 100%. What is the
stage of labor?
A. Second stage
B. Active phase ✅
C. Transition phase
D. Latent phase
Answer: B
1014)Pregnant woman with cervical dilatation 7 cm and fetus head on zero station.
What is the stage of labor?
A. Second stage
B. Active phase ✅

C. Transition phase
D. Latent phase
Answer: B
1015)What is collaboration method in conflict?
A. Organization system affected

B. Cooperation between two sides ✅

C. Cooperation on one side

ِ‫يِاْلَ أرض‬
‫أ‬ ُ ‫َم‬
ُ ‫ك‬
‫ثِِف‬ َ ‫اس‬
‫ِِفي أ‬َ َّ ‫ع‬
‫ِالن‬ ُ ‫ف‬
َ ‫ماِيَ أن‬ َّ َ‫وأ‬
َ ِ‫ما‬ َ
{ 255 }

Answer: B
1016)Patient came to ER with acute chest pain, restlessness. The patient has cardiac
surgery before. He has some injuries on his body
Bl. P 110/70 mmhg, HR 14 b/m, RR 22. What is the most appropriate nursing intervention?

A. Obtain 12 lead ECG ✅

B. Assess for bed sores


C. Administer Pain medication
D. Start I. V fluids
Answer: A
1017)Patient with acute renal failure (ARF)
Lab result
NA 120
Potassium 6
Calcium normal result
What is the most appropriate diet should nurse provide in food?
A. Low NA ✅

B. High potassium
C. High phosphate
D. Low carbohydrate
Answer: A
1018) Patient after inguinal hernia

What is best indicator for dischrage ?

A. Patient eat very well

B. Patient free of pain

C. Patient move without assistance ✅

D. Patient urine output500 ml

Answer: c

ِ‫يِاْلَ أرض‬
‫أ‬ ُ ‫َم‬
ُ ‫ك‬
‫ثِِف‬ َ ‫اس‬
‫ِِفي أ‬َ َّ ‫ع‬
‫ِالن‬ ُ ‫ف‬
َ ‫ماِيَ أن‬ َّ َ‫وأ‬
َ ِ‫ما‬ َ
{ 256 }

1019) The nurse is assessing 50-year-old woman whose thyroid enlarged. A blood
sample was collected and an analysis confirm diagnosis.
TSH 0.12 normal 0.4-6.5
Free 210 normal 50-140.
Thyroxin T4 normal 4.5-11.2
A. Thyroidectomy ✅

B. Incision and drainage


C. polythyroidectomy
D. adrenalectomy
Answer: A
1020) patient diagnosed with delirium sees the intravenous (IV) tubing and believes it to
be a snake. How should the healthcare provider document this behavior?
A. Hallucination
B. Illusion ✅

C. Confusion
D. Delusion
Answer: B
1021)What is the position during liver biopsy?
A-Right side lying
B-Prone
C-Lateral

D-supine and right hand below head✔️


Answer: D
1022)Community health nurse is providing health education for diabetic patient during
home visit regarding self-injection of insulin. Which of the following statement need
further teaching?

A. Recap /Cover the needle and throw it in normal home garbage ✅

Answer: A

ِ‫يِاْلَ أرض‬
‫أ‬ ُ ‫َم‬
ُ ‫ك‬
‫ثِِف‬ َ ‫اس‬
‫ِِفي أ‬َ َّ ‫ع‬
‫ِالن‬ ُ ‫ف‬
َ ‫ماِيَ أن‬ َّ َ‫وأ‬
َ ِ‫ما‬ َ
{ 257 }

1023)Psychiatric patient is complaining from extrapyramidal side effect with continuous


spasms and muscle contractions), motor restlessness, rigidity, slowness of movement,
tremor, and irregular, jerky movements, upward eye movement, smacking and sucking
of lips and protruding tongue. What is the most common drug cause that?
A. Clozapine
B. Haloperidol✔️

Answer: B
1024)Patient has been diagnosed with urinary tract infection. What is the
most likely cause of this infection?
A. Staphylococcus aureus
B. Neisseria gonorrhea

C. Escherichia coli enterococci ✅

D. Streptococcal beta-hemolytic A and B


Answer: C
1025)Amenorrhea for 5 weeks and breast tenderness. What should she do now?
A- HCG home test ✅

B- Ultrasound Sonar to see embryo


Answer: A
1026)At 6:00 PM while admitting a woman for a scheduled repeat C section1, a patient
tells a nurse that she drank a cup of coffee at 4:00OM because she wanted to avoid
getting a headache. Which of the following actions should the take first?
A. Inform anesthesia care provider. ✅

B. Ensure preoperative lab results are available.


C. Start prescribed IV with lactated ringers.
D. Contact patient's obstetrician
Answer: A
1027)Which of the following discharge planning instructions takes PRIORITY in patient
with congestive heart failure?
a. Maintaining a low cholesterol, low sodium and low potassium diet

ِ‫يِاْلَ أرض‬
‫أ‬ ُ ‫َم‬
ُ ‫ك‬
‫ثِِف‬ َ ‫اس‬
‫ِِفي أ‬َ َّ ‫ع‬
‫ِالن‬ ُ ‫ف‬
َ ‫ماِيَ أن‬ َّ َ‫وأ‬
َ ِ‫ما‬ َ
{ 258 }

b. Recognizing signs and symptoms that require immediate medical attention ✅

c. The importance of remaining physically active


d. Smoking Cessation
Answer: B
1028)37-year-old woman with inflammatory bowel disease is scheduled to undergo a
procedure in which a stoma will be formed in the right lower quadrant, five centimeters
below the waistline. The nurse advises the patient on how to avoid potential post-
operative intestinal obstruction. Which of the following types of food best
recommended post-operatively?
A. Broccoli and fish
B. Meats and cauliflower
C. Yogurt and parsley ✅

D. Corn and seeds


Answer: C
1029)The community nurse visited Postpartum mother and she complained her. She
feels abdominal pain during breastfeeding and she never feel that on previous delivery
before. What is the most appropriate action?
A. It is not important. No action
B. It is important. Stop breastfeeding

C. It is important. Continuous breastfeeding ✅

D. It is not important. Continuous breastfeeding


Answer: C
1030)Sigma theta tau theory in research. What is first priority guideline for Sigma theory?
A. Patient safety ✅

B. Health Team
C. Nursing staff
D. Patient satisfaction
Answer: A
1031)The head nurse wants to improve quality system in her department among nursing
staff. What is this considering?

ِ‫يِاْلَ أرض‬
‫أ‬ ُ ‫َم‬
ُ ‫ك‬
‫ثِِف‬ َ ‫اس‬
‫ِِفي أ‬َ َّ ‫ع‬
‫ِالن‬ ُ ‫ف‬
َ ‫ماِيَ أن‬ َّ َ‫وأ‬
َ ِ‫ما‬ َ
{ 259 }

A. Case management

B. Quality roles team ✅

Answer: B
1032)Why should recommend food with acid base (increase acidity)?
A. Prevent kidney stones
B. Prevent urinary retention

C. Increase urine output ✅

Answer: C
1033)Patient came to ER and diagnosed with meningitis. How should the nurse handle
with patient?
A. Keep patient in same isolation room ✅

B. Use mask N95 all times


C. Limit the visitors
Answer: A
1034)How decrease heat loss for neonate by convection?
A. Dry baby
B. Air condition AC ✅

Answer: B
1035)Patient admitted to medical unit with paraplegia. The patient can transfer himself
by wheel chair but need some assistance. The nurse inserted foley catheter. And made
urine analysis
Ph of urine 6 normal range 2-8
Urine color yellowish
What is the priority nursing diagnosis for him?

A. High risk for fall ✅

B. Risk for UTI


Answer: A
1036)What is the most common complications of varicose vein?

ِ‫يِاْلَ أرض‬
‫أ‬ ُ ‫َم‬
ُ ‫ك‬
‫ثِِف‬ َ ‫اس‬
‫ِِفي أ‬َ َّ ‫ع‬
‫ِالن‬ ُ ‫ف‬
َ ‫ماِيَ أن‬ َّ َ‫وأ‬
َ ِ‫ما‬ َ
{ 260 }

A. DVT

B. Varicose ulcer ✅

Answer: B
1037)What is the drug to minimize intracranial pressure?
A. Warfarin
B. Morphine
C. Potassium
D. Dulcolax
Answer: D
1038)Pregnant Woman with diabetes melitus. She came for periodic checkup and
follow up. The mother tested for lab investigations. Lab results high protein in urine and
confirmed preeclampsia
Bl. P 164 /100
Glucose level 324 g/dl
What is the toxic effect for preeclampsia?

A. Hypertension ✅
B. Increase heart rate
C. Increase Glucose rate
Answer: A
1039)Patient with smallpox. He became fine and no symptoms but still isolated for 12-14
days. What is the type of the period?
A. Incubation period
B. Infective period
C. Latency period
D. Communicable period
Answer: A
1040) nurse is assigned to the antenatal clinic to give health education to a group of
pregnant women on the danger signs in pregnancy Which of the following would the
nurse instruct women to immediately come to hospital?

ِ‫يِاْلَ أرض‬
‫أ‬ ُ ‫َم‬
ُ ‫ك‬
‫ثِِف‬ َ ‫اس‬
‫ِِفي أ‬َ َّ ‫ع‬
‫ِالن‬ ُ ‫ف‬
َ ‫ماِيَ أن‬ َّ َ‫وأ‬
َ ِ‫ما‬ َ
{ 261 }

A. If they experience some headaches

B. If they start bleeding from the vagina ✅

C. If their estimated data of delivery has passed

D. If they experience more than 10 fetal movements per day


Answer: B
1041)Amnion. inner membrane surrounding the fetus. The amniotic fluid, fetus, and
umbilical cord are all found within the amnion.
Chorion. Outermost embryonic membrane and forms part of the placenta.
To prevent and releive fetal distress related to maternal hypotension. Which of the
position should apply?
A. Left side ✅

B. Right side
C. Semi fowler
D. Knee chest position
Answer: A
1042)37 years old patient with spinal paralysis. He can't do self-care. What is the long
term goal?
A. Strength of body muscles ✅

B. Provide socialization and prevent depression


C. Provide self-care
Answer: A
1043)Woman with tabia fracture. What is the priority action for her?

A. Elevate and support leg ✅

B. Pain management
Answer: A
1044)45-year-old woman is receiving chemotherapy for breast cancer. Two weeks after
the initial treatment she telephoned the nurse at the cancer center and reports she has
hair loss, nausea, tiredness, a body temperature is 38.1 C, and air hunger. Which finding
most likely indicates she needs to report the clinic?

ِ‫يِاْلَ أرض‬
‫أ‬ ُ ‫َم‬
ُ ‫ك‬
‫ثِِف‬ َ ‫اس‬
‫ِِفي أ‬َ َّ ‫ع‬
‫ِالن‬ ُ ‫ف‬
َ ‫ماِيَ أن‬ َّ َ‫وأ‬
َ ِ‫ما‬ َ
{ 262 }

A. Pyrexia
B. Nausea
C. Hair loss

D. Air hunger ✅

Answer: D
1045)A midwife visits a mother four weeks after delivery. The mother is breastfeeding her
baby but she requests the midwife to suggest alternative formula milk as she has to
return back to her job and her baby will stay in a day care center. Which of the
following teaching plans is suitable for the mother?
A. Supplementary medications with bottle feeding
B. Combined schedule of the breastfeed and top feed
C. Hygiene practices with bottle feeding
D. How to express and save milk ✅
Answer: D
1046)What is insomnia drug?
A. Clozapine
B. Zolpidem ✅

Answer: B
1047)The community health nurse is giving instructions for Postpartum mother about oral
contraceptive pills. What is the most appropriate instructions?
A. Start taking pill after 7 days from menstruation for 21 days
B. Take one bill each day at same time ✅
Answer: B
1048)Mother of child came to ER. Her child has 7 times diarrhea. How can we detect
category of dehydration?
A. Do Physical examination ✅

B. Assess how many times of bowel movement


Answer: A
1049)Which statement best describes case management?

ِ‫يِاْلَ أرض‬
‫أ‬ ُ ‫َم‬
ُ ‫ك‬
‫ثِِف‬ َ ‫اس‬
‫ِِفي أ‬َ َّ ‫ع‬
‫ِالن‬ ُ ‫ف‬
َ ‫ماِيَ أن‬ َّ َ‫وأ‬
َ ِ‫ما‬ َ
{ 263 }

Clinical outcomes should occur within a prescribed time frame case manager
definition?
a-invasion of living tissues by pathogenic microorganism.
b- collaborative process of assessment, planning, facilitation, care coordination,
evaluation, and advocacy for options and services to meet an individual's and family's
comprehensive health needs through communication and available resources to
promote quality, cost-effective outcome. ✅

c- the person who leads or commands a group, organization, or country


d-non.
Answer: B
1050) 5-year-old boy has dietary modification due to his diabetes, In his parents'
encourage him to value good nutritional habits, they decided to deny him playing
favorite toys when he eats food that is not on his diet plan. What this mode of value
transmission is called?

A. Modelling

B. Moralising

C. Responsible choice

D. Rewarding and punishing ✅

Answer: D
1051)What is the priority care for patient during bathing?
A. Wear sterile gloves
B. Lower the bed and left side rails ✅

C. Put gauze on any wound


Answer: B
1052)What is signs for positive pregnancy confirmation?
A. Fetal heart sound by Doppler ✔️

B. Fetal movement by health care provider


C. Ultrasound
Answer: A

ِ‫يِاْلَ أرض‬
‫أ‬ ُ ‫َم‬
ُ ‫ك‬
‫ثِِف‬ َ ‫اس‬
‫ِِفي أ‬َ َّ ‫ع‬
‫ِالن‬ ُ ‫ف‬
َ ‫ماِيَ أن‬ َّ َ‫وأ‬
َ ِ‫ما‬ َ
{ 264 }

1053) A 10-month-old infant was admitted with recurrent otitis made with in months. On
assessment, the nurse noticed that the parents lock knowledge about the condition.
Health teaching is planned. Which risk factor should the nurse include in health
teaching?

A. High fever

B. Painful ear

C. Foreign body

D. Respiratory infection ✅

Answer: D
1054)What is the thing that prevent overlapping in duty of work?

A. Organizational structure ✅

B. Job description
C. Need for assessment
Answer: A
1055) Nurse manager conduct survey about patient fall
What first thing to do ?
A⁃ Review patients file
B⁃ Review literature ✅

C⁃ Interview with patient


Answer: B
1056) Women wit Ectopic pregnancy most confirm sign ?
A ⁃ throbbing pain in right upper quadrant

B⁃ Stabbing pain in lower abdomen quadrant ✅


Answer: B
1057) Which structure secret HCG hormones to sign pregnancy ?
A⁃ decidua
B ⁃ Zygote
C ⁃ Morula

ِ‫يِاْلَ أرض‬
‫أ‬ ُ ‫َم‬
ُ ‫ك‬
‫ثِِف‬ َ ‫اس‬
‫ِِفي أ‬َ َّ ‫ع‬
‫ِالن‬ ُ ‫ف‬
َ ‫ماِيَ أن‬ َّ َ‫وأ‬
َ ِ‫ما‬ َ
{ 265 }

D⁃ Blastocyst ✅

Answer: D
1058) Patient came to ER after motor accident. The patient has sever bleeding with
urine output 30 ml /hr and normal vital signs Normal Bl. P, normal HR, Normal respiration).
The doctor ask the nurse to assess good tissue perfusion. Which of the following signs
detect that? ?

A⁃ heart rate

B⁃ Respiration rate

C ⁃ Urine output ✅

D ⁃ Blood pressure

Answer: c
1059) Neurological shock sign ?

A⁃ increase BP

B⁃ Decrease heart rate ✅

Answer: B
1060) Full bladder is important to evacuated it during pregnancy before sonar and
during labour and after. What is the Major complications of full bladder?

A⁃ Pain

B ⁃ False reading

C⁃ Bleeding ✅

D. Infection

Answer: c
1061) Child with recurrent UTI , What is the most risk factors ?

A ⁃ abdominal distention

B⁃ Abdominal pain

C ⁃ Chronic constipation ✅

Answer: c

ِ‫يِاْلَ أرض‬
‫أ‬ ُ ‫َم‬
ُ ‫ك‬
‫ثِِف‬ َ ‫اس‬
‫ِِفي أ‬َ َّ ‫ع‬
‫ِالن‬ ُ ‫ف‬
َ ‫ماِيَ أن‬ َّ َ‫وأ‬
َ ِ‫ما‬ َ
{ 266 }

1062 ) Patient came to ER has chronic COPD with dyspnea, wheezing and large
amount of expectoration . Which type of COPD ?

A⁃ Bronchitis ✅

B⁃ Bronchitis asthma

C ⁃ Emphysema

Answer: A
1063 ) Child came to ER and complaining from abdominal pain. The child is diagnosed
willms. What is the first reaction for parent?

A⁃ Anger ✅

B-Bargaining

Answer: A
1064 ) Maslow's hierarchy of needs is an idea in psychology........

The priority nursing care of maslow hierarchy is :

A. Nursing diagnosis

B. Nursing care

C. Nursing planning ✅

D. Nursing care with scientification

Answer: C
1065 ) Community health nurse was visiting people in certain area for observation and
assessment. She found their high risk for lead poison. She refer all to get lab invistigation
and doing tests. What is the type for that?

A. Screening ✅✅

B. Survillance

C. Assessment

Answer: A

1066) Psychiatric patient use MAOI inhibitors. He should avoid tyramin in food. What is
the risk for tyramin intake?

ِ‫يِاْلَ أرض‬
‫أ‬ ُ ‫َم‬
ُ ‫ك‬
‫ثِِف‬ َ ‫اس‬
‫ِِفي أ‬َ َّ ‫ع‬
‫ِالن‬ ُ ‫ف‬
َ ‫ماِيَ أن‬ َّ َ‫وأ‬
َ ِ‫ما‬ َ
{ 267 }

A. GIT upset

B. Hypertension ✅

C. Kidney problems

Answer: B
1067 ) Scrub nurse experienced wound in her hand. She came to ER and care is done.
The next day she has duty. What should she do? ?

A⁃ Report to head nurse ✅

B ⁃ Change role with circulating nurse

C⁃ Wear double gloves in right hand

D⁃ Wash five minutes before gloving

Answer: A
1068 ) Patient with large liver tumor. The doctor decide to him cryosurgery. What does
this type of surgery means, Cryosurgery was preformed on a patient with cancer
tumors, this surgery ?

A. Freezes the tumor or abnormal or diseased tissue using cold liquid nitrogen or argon
gas ✅

B. Heat therapy

C. Hepatomegaly

Answer: A
1069 ) The neonate is delivered from diabetic mother. He is high risk for hypoglycemia
What should the nurse do to avoid it?

A⁃ Give insulin

B⁃ Oral feeding ✅

C ⁃ Nothing to do

Answer: B
1070 ) A patient is scheduled for an abdominal aneurysm repair .

This is what type of surgical intervention ?

ِ‫يِاْلَ أرض‬
‫أ‬ ُ ‫َم‬
ُ ‫ك‬
‫ثِِف‬ َ ‫اس‬
‫ِِفي أ‬َ َّ ‫ع‬
‫ِالن‬ ُ ‫ف‬
َ ‫ماِيَ أن‬ َّ َ‫وأ‬
َ ِ‫ما‬ َ
{ 268 }

A. Diagnostic

B. Transplant

C. Curative ✅

D. Palliative

Answer: c
1071 ) 26.A Patient has a dissection aortic aneurysm. The patient’s surgery

would be categorized as:

a. Elective

b. Urgent

c. Emergency ✅

d. Diagnosed
Answer: c
1072 ) Patient with I. V infusion . The patient is complaining redness swelling pain at
infusion site. What should the nurse do?

A. Slow rate

B. Stop and inform doctor ✅

C. Cold compress

Answer: B
1073 ) Community health nurse is providing public health survillance. What is the
method of problem solution should use to solve several reasons?

A. Tree

B. Brain storming ✅

C. Delphi
Answer: B
1074) A young Unit Manager of the Pediatric Ward. Most of her staff nurses are senior to
her, very articulate, confident and sometimes aggressive. She feels uncomfortable
believing that she is the scapegoat of everything that goes wrong in her department.
Which of the following is the best action that she must take?

ِ‫يِاْلَ أرض‬
‫أ‬ ُ ‫َم‬
ُ ‫ك‬
‫ثِِف‬ َ ‫اس‬
‫ِِفي أ‬َ َّ ‫ع‬
‫ِالن‬ ُ ‫ف‬
َ ‫ماِيَ أن‬ َّ َ‫وأ‬
َ ِ‫ما‬ َ
{ 269 }

A. Identify the source of the conflict and understand the points of friction ✅

B. Disregard what she feels and continue to work independently

C. Seek help from the Director of Nursing

D. Quit her job and look for another employment.

Answer: A
1075) As a young manager, she knows that conflict occurs in any organization. Which
of the following statements regarding conflict is NOT true?

A. Can be destructive if the level is too high

B. Is not beneficial; hence it should be prevented at all times ✅

C. May result in poor performance

D. May create leaders

Answer: B
1076) The head nurse knows that one of her staff is ‫اد‬burnout. Which of the following is
the best thing for her to do?

A. Advise her staff to go on vacation.

B. Ignore her observations; it will be resolved even without intervention

C. Remind her to show loyalty to the institution.

D. Let the staff ventilate her feelings and ask how she can be of help. ✅

Answer: D
1077) Nurse manager plans of assigning competent people to fill the roles designed in
the hierarchy. Which process refers to this?

A. Staffing ✅

B. Scheduling

C. Recruitment

D. Induction

Answer: A

ِ‫يِاْلَ أرض‬
‫أ‬ ُ ‫َم‬
ُ ‫ك‬
‫ثِِف‬ َ ‫اس‬
‫ِِفي أ‬َ َّ ‫ع‬
‫ِالن‬ ُ ‫ف‬
َ ‫ماِيَ أن‬ َّ َ‫وأ‬
َ ِ‫ما‬ َ
{ 270 }

Barrel chest find in :

A. Lung abcess

B. Emphesema ✅

C. Cor pulmonale

Answer: B
1078) o An 18-year-old client with acute lymphocytic leukemia is admitted to the bone
marrow transplantation unit. His family is having trouble

dealing with the emotional and financial pres-sures of his disease.

The Pharmacist, nurse, doctor, Physiotherapy meet to discuss the care plan for the
patient. What is the type of care interaction?

A- Case management

B- Interdisciplinary ✅

C- Cooperation

D- Collegiality

Answer: B
1079) Patient with somatization. What is the best action?

A. Cognitive development ✅

Answer: A
1080) 4 years old Child with cast. What is the nursing care for cast?

A. Put in hot water

B. Put infront of fan or cooller to dry fast

C. Put in cold water to dry fast

D. Put powder on the edges of the cast to prevent itching

Answer: B
1081) pregnant lady she come to gynocolical word she have vaginal bleeding and
cervex not dialted the problem ocuured before she followed bed rest and when she
did effort it returned back to her . They diagnosed her with threatened abortion. What is
the treatment should the nutse expect for this lady :

ِ‫يِاْلَ أرض‬
‫أ‬ ُ ‫َم‬
ُ ‫ك‬
‫ثِِف‬ َ ‫اس‬
‫ِِفي أ‬َ َّ ‫ع‬
‫ِالن‬ ُ ‫ف‬
َ ‫ماِيَ أن‬ َّ َ‫وأ‬
َ ِ‫ما‬ َ
{ 271 }

A-surgical treatment

B-induction of labor(syntocinon)

C-dialitation and curtage

D. Repeate Bed rest ✅

Answer: D
1082) A 23-year-old vaginal delivery primigravida mother was discomfort due to breast
engorgement on the second post The mother complained of pain on the breast site
and the baby not able to suck the milk. Which of the following will relief the mother's
discomfort?

A. Breast binder

B. Well-fitting brassiere

C. Encourage breast feeding ✅

D. Lactation suppressing medication


Answer: C
1083) Female patient has been advised that laboratory tests confirm genital warts and
recurrent many times . The nurse should teach the patient that a Papanicolaou test
(Pap smear) is recommended:

A. Once Every 6 months ✅

B. Once Every year

C. Once Every 3 years

Answer: A
1084) A 38-year- old patient is about to have lumber disk surgery. during preoperative
care, the nurse instructs the patient including the family.

members how to do "log rolling" to change patient position. One of the family

members ask why they must do such action in turning the patient.

postoperative.

Which of the following is the nurse best response?

A. Facility good circulation

ِ‫يِاْلَ أرض‬
‫أ‬ ُ ‫َم‬
ُ ‫ك‬
‫ثِِف‬ َ ‫اس‬
‫ِِفي أ‬َ َّ ‫ع‬
‫ِالن‬ ُ ‫ف‬
َ ‫ماِيَ أن‬ َّ َ‫وأ‬
َ ِ‫ما‬ َ
{ 272 }

B. Avoid spinal movement ✅

C. Prevent post-operative bed sore.

D. Makes changing of patient position easier.

Answer: B

1085) Which of the following needle's angle for insulin injection?

A. A

B. B ✅

C. C

Answer: B
1086) Mother want to ensure her fetus health is well. What is the most appropraite
method?

A. Measure fundal hight ✅

ِ‫يِاْلَ أرض‬
‫أ‬ ُ ‫َم‬
ُ ‫ك‬
‫ثِِف‬ َ ‫اس‬
‫ِِفي أ‬َ َّ ‫ع‬
‫ِالن‬ ُ ‫ف‬
َ ‫ماِيَ أن‬ َّ َ‫وأ‬
َ ِ‫ما‬ َ
{ 273 }

B. Ultrasonography

C. Check Mother heart rate


Answer: A

1087)Nurse administer digoxin to patient and dr order subsequent dose of digitalis


...nurse assess apical pulse before administering medication

When nurse should withholding it and notify dr?

A. heart rate 57✅

B. 120

C. 87

Answer: A

1088) Acute Respiratory distress syndrom is relates to?

A. Disease that cause chronic obstruction in airway

B. Disease that affect alveolar capilly permeability ✅

C. Disease caused due to excorine dysfunction

Answer: B
1089) Most appropraite method to explain ambulatory surgery for patient. which the
best method?

A. Lectury and video

B. Audio vision

C. Discussion ✅

D. Written instructions

Answer: C
1090) Newly promoted (nurse manager) ... what's priority for her?

A. Supervise patient care delivered by staff

ِ‫يِاْلَ أرض‬
‫أ‬ ُ ‫َم‬
ُ ‫ك‬
‫ثِِف‬ َ ‫اس‬
‫ِِفي أ‬َ َّ ‫ع‬
‫ِالن‬ ُ ‫ف‬
َ ‫ماِيَ أن‬ َّ َ‫وأ‬
َ ِ‫ما‬ َ
{ 274 }

B. Complete evaluation of nursing staff ✅

C. Make unit plan and operational plan

Answer: B
1091)Multicultural team work on saudi Arabia what's important thing for the all team
member to have?

A. Have the same value and belief

B. Have the same professional guidelines

C. Have the same goal and objectives toward work ✅

D. Have the same qualifications

Answer: C
1092) Patient fall and has injury ... what should nurse do to decrease liability to fall?

A. Just inform the coming nurse

B. Write incident report

C. Document what happened ... inform...and follow up patient ✅

Answer: C

1093) Quality team in meeting to set the Best measure to decrease liability for fall...What
is the best action for that?

A. Apply fall risk band around wrist

B. Lower bed level

C. Round observations on patient

D. Apply fall risk sticker on patient door ✅

Answer: D

1094) Action of comined oral contraceptives?

A. Affect lining of uterus making it unfavorable for implementation

B. Supress lutenizing hormone and follicle stimulating hormone

and prevent ovulation ✔️

ِ‫يِاْلَ أرض‬
‫أ‬ ُ ‫َم‬
ُ ‫ك‬
‫ثِِف‬ َ ‫اس‬
‫ِِفي أ‬َ َّ ‫ع‬
‫ِالن‬ ُ ‫ف‬
َ ‫ماِيَ أن‬ َّ َ‫وأ‬
َ ِ‫ما‬ َ
{ 275 }

Answer: B

1095) Women come to ER with her husband and the husband demonstrate she not talk
or voluntary eat since the son died in accident, She isolated since she has seen his son
body. What the long term care for her ?

A. Physiological needs

B. psychological Care ✅

C. spiritual support

D. family support

Answer: B

1096) The patient with head injury will be discharges. What is the best head injury
advice after discharge?

A. Repeat CT scan after 48 hours

B. Return immediately if drowsy ✅

Answer: B

1097) A 42 year old man with thalassemia received a packed cell transfusion. The nurse
assess the stability of his

condition after transfusion by monitoring the vital signs and general condition every two
hour. When should the

nurse immediately report the patient condition ?

A. Sever headache and raised blood pressure

B. Raised body temperature and flushed skin ✅

C. Joint pain, body ache listlessness

D. Restlessness & bradycardia

Answer: B

1098) Girl after eye surgery. She is irritable and try to remove sheld from eye. The nurse
observed sweeling, Redness and tears around eye. What is the Nursing diagnosis?

A. Anxiety

B. Pain, sweeling after surgery ✅

ِ‫يِاْلَ أرض‬
‫أ‬ ُ ‫َم‬
ُ ‫ك‬
‫ثِِف‬ َ ‫اس‬
‫ِِفي أ‬َ َّ ‫ع‬
‫ِالن‬ ُ ‫ف‬
َ ‫ماِيَ أن‬ َّ َ‫وأ‬
َ ِ‫ما‬ َ
{ 276 }

C. Risk for injury

Answer: B

1099) The baby has asthma and complaining with cough, fever and secretions . He is
lethargic and cyanosis around his dry and crackles lips and anorexic. Wahati the first
action?

A. Assess skin turger and dehydration

B. Assess Oxygen saturation and child behavior ✅

C. The last 24 hour intake and output

Answer: B

1100) A client diagnosed with somatization disorder visits multiple physicians because of
var- ious, vague symptoms involving many body systems. Which nursing diagnosis takes
priority?

A. Risk for injury R / T treatment from multiple physician. ✅

B. Anxiety R / T unexplained multiple somatic symptoms.

c. Ineffective coping R / T psychosocial distress.

D. Fear R / T multiple physiological complaints.

Answer: A

1101) What is the most common cause for peptic ulcer?

A. pylori

Answer: A

1102) When assessing the monitor tracing of a woman whose labor is being induced
with an infusion of oxytocin. her vital signs are stable and fall within a normal range.
Contractions are intense last 90 seconds. and occur every 1 1/2 to 2 minutes but with
moderate strength . The fetah heart rate drop from 140 to 130 b/m abd return to base
line within 15-20 second. The nurse's immediate action would be to:

A. Increase rate and continue

B. Slow rate and put the pt left lateral to assess fatal heart rate✅

C. Slow rate, give oxygen and inform the doctor about the fatal heartrate irregularities
and hypercontractlity

ِ‫يِاْلَ أرض‬
‫أ‬ ُ ‫َم‬
ُ ‫ك‬
‫ثِِف‬ َ ‫اس‬
‫ِِفي أ‬َ َّ ‫ع‬
‫ِالن‬ ُ ‫ف‬
َ ‫ماِيَ أن‬ َّ َ‫وأ‬
َ ِ‫ما‬ َ
{ 277 }

Answer: B

1103) 60-year-old man is being discharged from the post-operative Care Unit following
a transurethral resection of the

prostate. The nurse provides discharge information regarding the care of the bladder
catheter. Which method would

be most effective in bladder retraining for this patient?

A. Scheduled urination every 2-3 hours ✅

B. Limit fluid intake before sleeping time

C. Perform pelvic floor exercises daily

D. Increase fluid intake during the daytime

Answer: A

1104)Alteration in nutrition: More than body requirements for 14 months. The patient has
reached ideal body weight and reports

enjoying a new life style(see table)

*Alteration in nutrition, more than body requirements. Short term goals. The
Patient will :

*Lose 4 pounds each week.

*Eat a healthy well-balanced diet daily

*Exercise for 30 minutes 5 days each week.

Which of the following is the best action should the nurse take?

A. Continue the care plan as written for another year

B. Discontinue the care plan since the patient has met all goals

C. Change the long-term goal for altered nutrition to maintain current weight ✅

D. Discontinue the care plan and add new diagnosis of health seeking behavior

Answer: C

1105) 17 years old patient Newly diagnosed with type 1 diabetes melitus. He need to
inject insulin twice per day. What the most appropriate action?

A. Discharge instructions

ِ‫يِاْلَ أرض‬
‫أ‬ ُ ‫َم‬
ُ ‫ك‬
‫ثِِف‬ َ ‫اس‬
‫ِِفي أ‬َ َّ ‫ع‬
‫ِالن‬ ُ ‫ف‬
َ ‫ماِيَ أن‬ َّ َ‫وأ‬
َ ِ‫ما‬ َ
{ 278 }

B. Insulin injection ✅

C. life style change to mange disease

Answer: B

1106) Doctor order 1000 unit heparin in 5 ml. The available 5000 unit. How many ml
should the nurse give?

A. 1 ✅

B. 2

C. 3

Answer: A

1107) Nurse manger pull out (float) nurse from medical ward to CCU which of the
following Patients should be assigned to the nurse:

A. Patient just transferred from coronary artry surgery

B. Patient just received with unstable angina on heparin infusion ✅

C. Patient need discharge education about coronary stenting

D. Patient on admission need assessment

Answer: B

1108) A nurse is performing physical examination on a newborn. She that the baby has
developed cephalohematoma. This baby is at risk of which of the following?

A. Sudden death

B. Pathological jaundice ✅

C. Infected umbilical cot

D. Increased intracranial pressure

Answer: B

1109) Contraindication for oral contraceptives?

A. Hypotension

B. Hypertension ✅

C. Anemia

ِ‫يِاْلَ أرض‬
‫أ‬ ُ ‫َم‬
ُ ‫ك‬
‫ثِِف‬ َ ‫اس‬
‫ِِفي أ‬َ َّ ‫ع‬
‫ِالن‬ ُ ‫ف‬
َ ‫ماِيَ أن‬ َّ َ‫وأ‬
َ ِ‫ما‬ َ
{ 279 }

D. Infection

Answer: B

1110 ) Which of the following diagnostic studies is essential to differentiate

between renal failure and lower renal obstruction that cause urinary

retention?

A. Cholesterol level

B. Abdominal X-ray

C. Complete blood tests

D. Blood urea nitrogen and serum Creatinine. ✅

Answer: D

1111) Nurse is caring for a 58-year-old patient (See lab result).

Test Result Magnesium 2.8

Normal Value 0.7-1.2mmoI/L

Which ECG change is the nurse expected to note?

A. Prolonged QRS ✅

B. Multiple P waves

C. Prominent U waves

D. Depressed ST segment

Answer: A

1112) What is the goal for cardiac catheterization?

A. Obtain venous pressure

B. Assess Oxygen for heart chambers ✅

Answer: B

1113) The patient ask him self too much if he close the home door or no. What is the
most appropriate Diagnosis?

A. Obsessive compulsive disorder ✅

ِ‫يِاْلَ أرض‬
‫أ‬ ُ ‫َم‬
ُ ‫ك‬
‫ثِِف‬ َ ‫اس‬
‫ِِفي أ‬َ َّ ‫ع‬
‫ِالن‬ ُ ‫ف‬
َ ‫ماِيَ أن‬ َّ َ‫وأ‬
َ ِ‫ما‬ َ
{ 280 }

B. illusion

C. Hallucinations

Answer: A

1114) Gremaer measles isolation period after rash ?

A. 5 days

B. 1 week ✅

C. 2 weeks

D. 3 weeks

Answer: B

1115) Patient has given digoxin then the doctor ordered Digibind which of the following
vital signs is most likely the patient have:

A. Bp ….. HR 69

B. Bp ….. HR 80

C. Bp ….. HR 90

D. Bp ….. HR 120

Answer: D

1116) Which patient is contraindicated from MRI ?

A. Patient who has dye contract allergy

B Patient who have cardiac pacemaker ✅

Answer: B

1117) is the dose of vitamin k according to WHO 2017?

A. 0.2

B. 0.5

C. 1 ✅

D. 2

Answer: C

ِ‫يِاْلَ أرض‬
‫أ‬ ُ ‫َم‬
ُ ‫ك‬
‫ثِِف‬ َ ‫اس‬
‫ِِفي أ‬َ َّ ‫ع‬
‫ِالن‬ ُ ‫ف‬
َ ‫ماِيَ أن‬ َّ َ‫وأ‬
َ ِ‫ما‬ َ
{ 281 }

1118) Postpartum mother breastfeeding weight 46 kg height 164 cm daily intake 1800
kcal what is the Nursing diagnosis?

A. Nutritional imbalance less than body requirement ✅

B. Altered parent-infant relationship

C. Ineffective breastfeeding

Answer: A

1119) Which goal is an aim of public surveillance?

A. To rapidly detect the introduction and early cases of a pandemic disease ✅

B. To serve as an early warning system to detect increases in illness in the community

C. To monitor a pandemic's impact on health

D. To track trends in community disease activity and identify populations that are
severely affected

Answer: A

1120) Why would a hospital use surveillance?

a. To protect the public against isolated patients

b. To eliminate pathogens from the environment

c. To improve quality of care and outcomes ✅

d. To decrease the incidence of ventilator-acquired pneumonia

Answer: C

1121) What is the position during Chemotherapy session?

A. Supine

B. Semi fowler ✅

Answer: B

1122) Elderly patient is receiving chemotherapy. The next day while he is having the
dinner, He can't complete eating. What is the most appropriate cause?

A.. Fatigue

B.. Altered sensation related to inflamation in gum ✅

ِ‫يِاْلَ أرض‬
‫أ‬ ُ ‫َم‬
ُ ‫ك‬
‫ثِِف‬ َ ‫اس‬
‫ِِفي أ‬َ َّ ‫ع‬
‫ِالن‬ ُ ‫ف‬
َ ‫ماِيَ أن‬ َّ َ‫وأ‬
َ ِ‫ما‬ َ
{ 282 }

Answer: B

1123) The nurse is giving health education about cooper iud disadvantages. Which of
the following is consider disadvantage for CIUD ?

A. Cause Iron deficiency anemia

B. Short term cost

C. Painful Insertion ✅

Answer: C

1124) The nurse Assess Neonat after delivery using Apgar score.

Sponteanous respiration

Prompt respond

Limitted cry

98 pulse

Pinkish body color execpt hands

What is the expected score?

A. 8

B. 7 ✅

C. 6

D. 5

Answer: B

1125) The comatosed patient with NGT for feeding and positioned in low fowler. The
nurse enter the patient room. She found the patient in supine position. She auscultated
lung sound with diventure. What is the most appropriate nursing diagnosis?

A. Risk for injury

B. Risk for aspiration due to NGT ✅

Answer: B

1126) Pregnant 33 week came to ER with sudden vaginal beeding and

soft abdomen painless. What is the most appropriate Diagnosis?

ِ‫يِاْلَ أرض‬
‫أ‬ ُ ‫َم‬
ُ ‫ك‬
‫ثِِف‬ َ ‫اس‬
‫ِِفي أ‬َ َّ ‫ع‬
‫ِالن‬ ُ ‫ف‬
َ ‫ماِيَ أن‬ َّ َ‫وأ‬
َ ِ‫ما‬ َ
{ 283 }

A. Threatened Abortion

B. Abruptio placenta

C. Placenta previa ✅

Answer: C

1127) The patient has fever, nuasea, vomiting and rebound tenderness. He is
complaining diarrhea with blood and abdominal pain. What is the most appropriate
Diagnosis?

A. Lower right pain

B. Appendicitis

C. Gastroenteritis

D. Devirticulitis ✅

Answer: D

1128) 17 year old mother after normal delivery have anemia and bleeding

which of the following education the nurse do?

a. Mass media

b. Group education

c. Individual education ✅

Answer: c

1129) A newborn baby is being evaluated on the APGAR score immediately after his
birth. The score measures the quality of newborn's pulse rate, reflexes, muscles tone and
respiration. His APGAR score is graded as 7 out 10. What is the significance of measuring
the APGAR score immediately after the birth?

A. It helps to plan the treatment for congenital diseases

B. It serves as a permanent record for the newborn babies

C. It provides the bases for the comparison, a few minutes later ✅

D. It helps identifying the abnormalities related to muscular tone

Answer: c

ِ‫يِاْلَ أرض‬
‫أ‬ ُ ‫َم‬
ُ ‫ك‬
‫ثِِف‬ َ ‫اس‬
‫ِِفي أ‬َ َّ ‫ع‬
‫ِالن‬ ُ ‫ف‬
َ ‫ماِيَ أن‬ َّ َ‫وأ‬
َ ِ‫ما‬ َ
{ 284 }

1130) o As per of a neurological assessment, which of the following is

associated with the higher score on the Glasgow coma scale?

A. Eye opening to pain, no verbalization.

B. Confused, obey command. ✅

C. Localized pain, abnormal extension.

D. Eye opening to speech, confused

Answer: B

1131) Newly nurse checked v/s for postoperative pt 2 days ago. The patient had high
blood pressure. What should the nurse do next ?

A. Check medication list if contain anti hypertensive

B. Check patient medical record ✅

Answer: B

1132 ) Which of the following APGAR score is consider moderate risk at the first minutes
and after 5 minutes of assessment?

A. 3-4

B. 3-5

C. 5-7 ✅

D. 8-10

Answer: C

1133) Patient came to ER breath rapidly. What is the first action?

A. Notify the doctor

B. Adminster oxygen

C. Assess oxygen saturation ✅

Answer: C

1134) Pregnant woman 12 week after motor accident. She came to ER with vaginal
bleeding. What is the most appropriate Diagnosis?

A-Placenta previa

ِ‫يِاْلَ أرض‬
‫أ‬ ُ ‫َم‬
ُ ‫ك‬
‫ثِِف‬ َ ‫اس‬
‫ِِفي أ‬َ َّ ‫ع‬
‫ِالن‬ ُ ‫ف‬
َ ‫ماِيَ أن‬ َّ َ‫وأ‬
َ ِ‫ما‬ َ
{ 285 }

B-inevitable abortion ✅

C. Abruptio placenta

Answer: B

1135)Woman come to follow up in the third week after delivery ...nurse observe that
uterus is slight palpable

Perineal pad full soaked with blood ...Bp 100/60 temperature 39 what sould nurse
anticipate?

A. Perineal laceration

B. Retained placental fragment ✅

Answer: B

1136)Pregnant woman in labor. She has sever abdominal pain extend to symphysis
pubis. Contraction frequency every 3 minutes and Duration 40 - 50 seconds. Cervical
dilatation 3 cm. What is the most appropriate action?

A. Encourage her to walk ✅

B. Give nitro oxide

C. Give analgesic

Answer: A

1137) Mother with 11 month infant is asking for wrong action for her baby. Which of the
following is consider risk for him?

A. Giving him Cow milk ✅

B. Using Rear face car seat

Answer: A

1138) Psychiatric patient with depression. The doctor order antidepressant with low side
effect. Which of the following is the most appropriate type of antidepressant?

A. Anti depressant nor Adrenaline

B. Monoamine oxidase inhibitors (MAOIs)

C. Selective serotonin reuptake inhibitors (SSRIs) ✅

Answer: C

ِ‫يِاْلَ أرض‬
‫أ‬ ُ ‫َم‬
ُ ‫ك‬
‫ثِِف‬ َ ‫اس‬
‫ِِفي أ‬َ َّ ‫ع‬
‫ِالن‬ ُ ‫ف‬
َ ‫ماِيَ أن‬ َّ َ‫وأ‬
َ ِ‫ما‬ َ
{ 286 }

1139) Nurse manager provide evaluation for all staff. How can we detect that she
acheive good feed back during evaluation?

A. Apply with all staff same criteria during evaluation ✅

Answer: A

1140)Patient hospitalized for surgical procedure. The nurse informed the patient that she
will be discharged. On the next shift another nurse received her. The nurse has
discussed with patient and family and conflict is happened. What is the most
appropriate action?

A. Check the patient file ✅

B. Call supervisor inform her about previous nurse action

Answer: A

1141)Patient came to ER after motor accident with hip fracture. What is the first
important sign should be concerned?

A. Hematurea

B. Hypotension ✅

Answer: B

1142)When giving analgesic for pregnant mother during labor. What is the most
important consideration should the nurse taken?

A. Mother

B. Fetus

C. Contraction

D. It will affect and cause prolonged labor ✅

Answer: D

1143) Child diagnosed with appendicitis. He will be have appendectomy. He was


anxious and crying. The nurse was asking mother to calm him. His mother do behavior
that indicates there is misunderstanding for the procedure. Which of the following
statement should the nurse correct to wrong behavior for the mother?

A. You should not cry. You are older.

ِ‫يِاْلَ أرض‬
‫أ‬ ُ ‫َم‬
ُ ‫ك‬
‫ثِِف‬ َ ‫اس‬
‫ِِفي أ‬َ َّ ‫ع‬
‫ِالن‬ ُ ‫ف‬
َ ‫ماِيَ أن‬ َّ َ‫وأ‬
َ ِ‫ما‬ َ
{ 287 }

B. Because You will be punished, you have the surgery ✅

C. Explain to child pre-operative and postoperative procedure

Answer: B

1144)The nurse stated to Psychiatric patient " I will stay here with you". Which of the
following therapeutic communication thechniques that the nurse used.?

A. Accepting

B. Exploring

C. Offering self ✅

D. Reflecting

Answer: C

1145)New manager promoted to position. Which of the following is the most


appropriate action?

A. Assess patient care

B. Interview for new staff ✅

C. Ask help from head nurse

Answer: B

1146)Alzheimer's patient high risk for Fall and admitted with fractures hip during
hospitalization the patient fell twice. What should the nurse do?

A. Continue same plan

B. Impair skin integrity

C. Risk for pressure ulcer

D. Non compliance add new nursing diagnosis ✔️✔️

Answer: D

ِ‫يِاْلَ أرض‬
‫أ‬ ُ ‫َم‬
ُ ‫ك‬
‫ثِِف‬ َ ‫اس‬
‫ِِفي أ‬َ َّ ‫ع‬
‫ِالن‬ ُ ‫ف‬
َ ‫ماِيَ أن‬ َّ َ‫وأ‬
َ ِ‫ما‬ َ
{ 288 }

1147)The nurse is preparing patient with knee replacement surgery. During physical
examination for patient, The nurse observed abdominal pulsating mass in epigastrium .
The patient with high blood pressure. What is the first action for nurse?

A. Continue in surgery preperation

B. Make abdominal Ultrasound ✅

Answer: B

1148) Patient came to ER with lower back pain and numbness of leg

A. Multiple scelorosis

Answer: A

1149)Patient with vital signs.

Blood pressure 100/70., T 40 °c, HR 89, Hgb 11.7 g/dl.

What is the first action for the nurse?

A. Corticosteroid for inflammation

B. Antibiotics for infection ✅

C. I. V fluids for blood pressure

D. Blood transfusion for Hgb

Answer: B

1150)What is the initial sign should the nurse Assess indicates that ICP for patient not
increased?

A. Patient tolerate lower head of bed

B. Normal blood pressure

C. Absence of confusion and restlessness ✅

Answer: C

1151) Patient take a full course of treatment for pneumonia what confirmed that
treatment was effective?

A-decrease haemoglobin level

B- decrease wbc count ✅

ِ‫يِاْلَ أرض‬
‫أ‬ ُ ‫َم‬
ُ ‫ك‬
‫ثِِف‬ َ ‫اس‬
‫ِِفي أ‬َ َّ ‫ع‬
‫ِالن‬ ُ ‫ف‬
َ ‫ماِيَ أن‬ َّ َ‫وأ‬
َ ِ‫ما‬ َ
{ 289 }

C-low platelets count

D- high hematocrite

Answer: B

1152) What is the difference between Transational stool and meconium?

1153 ) Which of the following category for CTG?

A. Normal

B. Category I

C. Category II

D. Category III ✅

Answer: D

ِ‫يِاْلَ أرض‬
‫أ‬ ُ ‫َم‬
ُ ‫ك‬
‫ثِِف‬ َ ‫اس‬
‫ِِفي أ‬َ َّ ‫ع‬
‫ِالن‬ ُ ‫ف‬
َ ‫ماِيَ أن‬ َّ َ‫وأ‬
َ ِ‫ما‬ َ
{ 290 }

1154) Patient with somatization. The patient will experiencing which of the following?

A. With clinical symptoms ✅

B. No clinical symptoms

Answer: A

1155)Girl diagnosed with somatization. What should the nurse expect for this patient
high risk for ?

A. Anorexia nervosa

B. Anxiety disorder ✅

Answer: B

1156)Community health nurse visit diabetic patient. One of family member complaining
for her that patient unmoved. what should the nurse do?

A. Forced the patient to move

B. Walk with the patient and take round

C. Ask patient express feeling and the cause ✅

Answer: C

1157)The nurse found the patient fell down on the floor. After she helped the patient to
get the bed. Do assessment and the patient is good. What is the next first action?

A. Write incident report

B. Set bed alarm ✅

C. Call the Witness client to be side the patient

D. Inform the manager

Answer: B

1158)The nurse research ER need data for study. She called to another nurse in the
ward to ask about Information for DM patient. What should the nurse response ?

A. Give her information

B. This my patient not give you information

C. No problem to give you information but not on the phone

ِ‫يِاْلَ أرض‬
‫أ‬ ُ ‫َم‬
ُ ‫ك‬
‫ثِِف‬ َ ‫اس‬
‫ِِفي أ‬َ َّ ‫ع‬
‫ِالن‬ ُ ‫ف‬
َ ‫ماِيَ أن‬ َّ َ‫وأ‬
َ ِ‫ما‬ َ
{ 291 }

D. Ask patient to get informed consent to use this information ✅

Answer: D

1159)Physician came to the ward and he checked patient medical record. After
finishing the nurse find that the physician not related to the patient. What should the
nurse do?

A. Nothing to do that is patient and normal

B. Call patient to clarify for reasons and inform the patient

C. Inform nurse supervisor ✅

Answer: C

1160)What is chest assessment for patient with Vitamin D deficiency ?

A. Dull sound

B. Rosary ✅

C. Crackle

D. Chest deformity

Answer: B

1161)Pregnant woman had an accident....and come to ER with brown


bleeding...abdominal distension...cramps. In abdomen fetaus heart rate not heard

A. Inevitable

B. Threatened

C. Abruptio placenta

Answer: A

1162) Unit manager conducts a 6-month performance review session with a staff
member. Which of the following actions is appropriate?

A. She asks another nurse to attest the session as a witness.

B. She informs the staff that she may ask another nurse to read the appraisal before the
session is over.

C. She tells the staff that the session is manager-centered.

ِ‫يِاْلَ أرض‬
‫أ‬ ُ ‫َم‬
ُ ‫ك‬
‫ثِِف‬ َ ‫اس‬
‫ِِفي أ‬َ َّ ‫ع‬
‫ِالن‬ ُ ‫ف‬
َ ‫ماِيَ أن‬ َّ َ‫وأ‬
َ ِ‫ما‬ َ
{ 292 }

D. The session is private between the two members. ✅

Answer: D

1163) Nurse enter patient room ...found him crying ...she asked him question about why
he crying ....he didn't respond...?

A. Get out of room to maintain privacy

B. Sit with him and not say anything ✔️

C. Say don't cry

Answer: B

1164)What is the most common complication during TPN insertion?

A. Rebound hypoglycemia

B. Air emoblism

C. Pneumothorax ✅

Answer: C

ِ‫يِاْلَ أرض‬
‫أ‬ ُ ‫َم‬
ُ ‫ك‬
‫ثِِف‬ َ ‫اس‬
‫ِِفي أ‬َ َّ ‫ع‬
‫ِالن‬ ُ ‫ف‬
َ ‫ماِيَ أن‬ َّ َ‫وأ‬
َ ِ‫ما‬ َ

You might also like